SEARCH (Type the topic here)

General Surgery MCQ

1. Skeletal muscle breakdown produces predominantly liberation of which two amino acids?
A. Lysine.
B. Tyrosine.
C. Alanine.
D. Glutamine.
E. Arginine.
Answer: CD

DISCUSSION: Alanine is released from skeletal muscle and extracted by the liver, where it is converted to new glucose. Glutamine is also released from muscle and participates in renal acid-base homeostasis and serves as fuel for rapidly growing cells such as enterocytes, stimulated macrophages, and fibroblasts. Together, these two amino acids account for approximately two thirds of the nitrogen released from skeletal muscle.




2. In “catabolic” surgical patients, which of the following changes in body composition do not occur?
A. Lean body mass increases.
B. Total body water increases.
C. Adipose tissue decreases.
D. Body weight decreases.
Answer: A

DISCUSSION: Lean body mass represents the body compartment that contains protein. Because critical illness stimulates proteolysis and increased excretion of body nitrogen, this compartment is consistently reduced, not increased. The change in body composition is associated with a loss of body weight, an increase in total body water, and a decrease in body fat.


3. The hormonal alterations that follow operation and injury favor accelerated gluconeogenesis. This new glucose is consumed by which of the following tissues?
A. Central nervous system.
B. Skeletal muscle.
C. Bone.
D. Kidney.
E. Tissue in the healing wound.
Answer: ADE

DISCUSSION: Glucose is produced in increased amounts to satisfy the fuel requirements of the healing wound. In addition, nerve tissue and the renal medulla also utilize this substrate. Skeletal muscle primarily utilizes fatty acids, and bone utilizes mineral substrate.


4. Cytokines are endogenous signals that stimulate:
A. Local cell proliferation within the wound.
B. The central nervous system to initiate fever.
C. The production of “acute-phase proteins.”
D. Hypoferremia.
E. Septic shock.
Answer:ABCD

DISCUSSION: Although cytokines exert primarily autocrine and paracrine effects, they may also cause systemic effects.


5. The characteristic changes that follow a major operation or moderate to severe injury do not include the following:
A. Hypermetabolism.
B. Fever.
C. Tachypnea.
D. Hyperphagia.
E. Negative nitrogen balance.
Answer: D

DISCUSSION: The characteristic metabolic response to injury includes hypermetabolism, fever, accelerated gluconeogenesis, and increased proteolysis (creating a negative nitrogen balance). Food intake is generally impossible because of abdominal injury or ileus. With time, food intake increases, but the patient generally experiences anorexia, not hyperphagia.


6. Shock can best be defined as:
A. Hypotension.
B. Hypoperfusion of tissues.
C. Hypoxemia.
D. All of the above.
Answer: B

DISCUSSION: Shock, no matter what the cause, is a syndrome associated with tissue hypoperfusion. Tissue hypoperfusion leads to tissue hypoxia, which may or may not be due to hypoxemia. Hypotension is a late sign of shock and, therefore, is not a good clinical indicator of the presence of tissue hypoperfusion.


7. Which of the following statements about continuous cardiac output monitoring are true?
A. Continuous cardiac output monitoring may unmask events not detected by intermittent cardiac output measurements.
B. Continuous cardiac output monitoring by the thermodilution method requires continuous infusion of fluid injectate at a constant rate and temperature.
C. The major advantage of the Fick method over the thermodilution method of calculating cardiac output is that it is noninvasive, requiring only the determination of oxygen consumption by respiratory gas analysis.
D. The technique of thoracic electrical bioimpedance utilizes sensors to determine stroke volume by detecting changes in resistance to a small, applied alternating current.
Answer: AD

DISCUSSION: Various techniques are available to measure cardiac output continuously. The advantages of continuous cardiac output monitoring, as compared with intermittent methods, are (1) previously undetected events may be unmasked; (2) more prompt recognition of adverse events may be achieved; and (3) earlier therapeutic intervention may be possible. Continuous cardiac output monitoring using the thermodilution method appears to be as accurate as the “standard” intermittent bolus method, but it does not require fluid injectates. In this method, a modified pulmonary artery catheter incorporating a thermal filament heats blood in the right ventricle at pulsed intervals, and a distal thermistor detects the temperature change, which can be related mathematically to cardiac output. The Fick method combines respiratory gas analysis with oximetery to determine oxygen consumption (V(overdot)O 2) and to estimate mixed venous and arterial oxygen content differences, respectively. Cardiac output (CO) is then determined from the formula: CO = V(overdot)O 2/ {C(a-v)O 2 × 10} @ V(overdot)O 2/ {SaO 2 - SvO 2) × (Hb) × (1.39) × 10}. Thoracic electrical bioimpedance is a technique by which the resistance to a small-amplitude alternating current (i.e., the impedance) is measured using various electrodes. The impedance change induced by each cardiac ejection is a function of the stroke volume, which then can be used to calculate the cardiac output.


8. Which of the following statements regarding cytokines is incorrect?
A. Cytokines act directly on target cells and may potentiate the actions of one another.
B. Interleukin 1 (IL-1) is a major proinflammatory mediator with multiple effects, including regulation of skeletal muscle proteolysis in patients with sepsis or significant injury.
C. Platelet-activating factor (PAF) is a major cytokine that results in platelet aggregation, bronchoconstriction, and increased vascular permeability.
D. Tumor necrosis factor alpha (TNF-a), despite its short plasma half-life, appears to be a principal mediator in the evolution of sepsis and the multiple organ dysfunction syndrome because of its multiple actions and the secondary cascades that it stimulates.
Answer: C

DISCUSSION: Cytokines are soluble peptide molecules that are synthesized and secreted by a number of cell types in response to injury, inflammation, and infection. Cytokines, which include the interleukins, tumor necrosis factor, colony-stimulating factors, and the interferons, comprise only one category of inflammatory mediators involved in the host response. Endotoxin, complement fragments, eicosanoids, kinins, nitric oxide, oxidants, and PAF are noncytokine mediators that also have important roles in the systemic inflammatory response. IL-1 and TNF-a, like other cytokines, have multiple effects on target cells and potentiate the actions of other mediators to produce an amplified inflammatory response. TNF-a is thought to play a central role in the stress response, particularly in response to endotoxemia.


9. True statements concerning hypoadrenal shock include which of the following?
A. Adrenocortical insufficiency may manifest itself as severe shock refractory to volume and pressor therapy.
B. The presence of hyperglycemia and hypotension may suggest the diagnosis of shock due to adrenocortical insufficiency.
C. Hydrocortisone does not interfere with the serum cortisol assay and should be given to hemodynamically unstable patients suspected of having hypoadrenal shock.
D. The rapid adrenocorticotropic hormone (ACTH) stimulation test should be performed to help establish the diagnosis of acute adrenocortical insufficiency.
Answer: AD

DISCUSSION: Shock due to acute adrenocortical insufficiency is relatively uncommon but must be considered when shock refractory to volume replacement and pressor therapy is present. Hypoglycemia may be present. Patients with high metabolic stress may exhibit adrenal insufficiency only under conditions of severe stress; thus, a history of adrenal insufficiency or steroid dependency need not be elicited. When adrenocortical insufficiency is suspected, the rapid ACTH (cosyntropin) stimulation test should be performed. Serum cortisol levels should be drawn before intravenous administration of 250 mg. of cosyntropin, and 30 and 60 minutes afterward. A peak cortisol level of less than 20 mg./100 ml. suggests abnormal adrenal function. In a hemodynamically unstable patient therapy should be instituted before the test results become available. Dexamethasone does not interfere with the cortisol assay, and it is the corticosteroid of choice while the ACTH stimulation test is being performed.


10. All of the following are true about neurogenic shock except:
A. There is a decrease in systemic vascular resistance and an increase in venous capacitance.
B. Tachycardia or bradycardia may be observed, along with hypotension.
C. The use of an alpha agonist such as phenylephrine is the mainstay of treatment.
D. Severe head injury, spinal cord injury, and high spinal anesthesia may all cause neurogenic shock.
Answer: C

DISCUSSION: Neurogenic shock occurs when severe head injury, spinal cord injury, or pharmacologic sympathetic blockade leads to sympathetic denervation and loss of vasomotor tone. Both arteriolar and venous vessels dilate, causing reduced systemic vascular resistance and a great increase in venous capacitance. The patient's extremities appear warm and dry, in contrast to those of a patient in cardiogenic or hypovolemic shock. Tachycardia is frequently observed, though the classic description of neurogenic shock includes bradycardia and hypotension. Volume administration to fill the expanded intravascular compartment is the mainstay of treatment. The use of alpha-adrenergic agonist is infrequently necessary to treat neurogenic shock.


11. True statements regarding eicosanoids include which of the following?
A. Prostaglandins and thromboxanes are synthesized via the cyclo-oxygenase pathway.
B. The vasoconstricting, platelet-aggregating, and bronchoconstricting effects of thromboxane A 2 are balanced by the actions of prostacyclin, which produces the opposite effects.
C. Leukotriene synthesis is inhibited by the action of nonsteroidal anti-inflammatory drugs (NSAIDs).
D. The principal prostaglandins have a short circulation half-life and exert most of their effects locally.
Answer: ABD

DISCUSSION: The eicosanoids are a group of compounds arising from the metabolism of arachidonic acid. The prostaglandins and thromboxanes are synthesized via the cyclo-oxygenase pathway; thus, their synthesis is blocked by NSAIDs. Leukotrienes, on the other hand, are synthesized via the lipoxygenase pathway. Prostacyclin, produced largely by vascular endothelium, inhibits platelet aggregation and causes vasodilatation as well as bronchodilatation. Its effects are balanced by those of thromboxane A 2, which is produced by platelets and also local actions, including platelet aggregation, vasoconstriction, and bronchoconstriction. The leukotrienes also have pulmonary and hemodynamic effects and may be involved in the physiologic responses associated with anaphylactic and septic shock.


12. Which of the following statements about delivery-dependent oxygen consumption are true?
A. Below the critical oxygen delivery (D(overdot)O 2crit), one would expect to see a decrease in the lactate-pyruvate ratio.
B. D(overdot)O 2crit may be increased in patients with sepsis.
C. A desirable goal in the treatment of shock is to achieve delivery-independent oxygen consumption.
D. The oxygen extraction ratio remains constant as long as oxygen delivery remains above D(overdot)O 2crit.
Answer: BC

DISCUSSION: Oxygen consumption is said to be delivery dependent below a critical point, D(overdot)O 2crit, at which anaerobic metabolism supervenes. Above this point, oxygen consumption is relatively independent of oxygen delivery because the body's cells can compensate for falls in oxygen delivery by extracting more oxygen. In the delivery-dependent region, if cellular hypoxia is present, the lactate-pyruvate ratio rises, owing to the switch to anaerobic metabolism. Generally, it is desirable to achieve delivery-independent oxygen consumption, to avoid ongoing tissue hypoxia. There is considerable debate, however, about the nature of the oxygen consumption–oxygen delivery relationship in cases of established sepsis or multiple organ dysfunction syndrome. In such cases, D(overdot)O 2crit may be increased, although the therapeutic benefit of trying to achieve “supranormal” oxygen delivery has not been firmly established.


13. All of the following may be useful in the treatment of cardiogenic shock except:
A. Dobutamine.
B. Sodium nitroprusside.
C. Pneumatic antishock garment.
D. Intra-aortic balloon pump.
Answer: C

DISCUSSION: Cardiogenic shock occurs when the heart fails to generate adequate cardiac output to maintain tissue perfusion. Intrinsic causes such as myocardial dysfunction secondary to coronary artery disease, or extrinsic causes such as pulmonary embolism, tension pneumothorax, and pericardial tamponade, may produce cardiogenic shock. Principles of treatment of cardiogenic shock are aimed at optimizing preload, cardiac contractility, and afterload. Preload is usually adequate or high in cardiogenic shock. Dobutamine is a useful inotropic agent, particularly when filling pressures are high, because of its mild vasodilatory effect, as well as its effect to enhance cardiac contractility. Afterload-reducing agents, such as sodium nitroprusside, may be beneficial in cardiogenic shock in the setting of elevated filling pressures, low cardiac output, and elevated systemic vascular resistance. Cardiac output may improve with use of afterload-reducing agents by decreasing myocardial wall tension and optimizing the myocardial oxygen supply-demand ratio. The intra-aortic balloon pump (IABP), by providing diastolic augmentation, reducing left ventricular afterload, and reducing myocardial oxygen consumption, is sometimes useful in the treatment of cardiogenic shock. The IABP is especially useful in low–cardiac output postcardiotomy patients, in patients awaiting revascularization, and in patients with acute myocardial infarction complicated by mitral insufficiency or ventricular septal defect. The pneumatic antishock garment (PASG), which causes an increase in systemic vascular resistance, is contraindicated in cardiogenic shock.


14. Which of the following statements concerning monitoring techniques in the intensive care unit are true?
A. Pulmonary artery and pulmonary capillary wedge pressure readings should be made at end inspiration, to minimize ventilatory artifacts.
B. Continuous SvO 2 monitoring based on the technique of reflectance spectrophotometry has been shown to be accurate and reliable.
C. Direct measurement of gastric intramucosal pH can be provided by gastrointestinal tonometry.
D. Hyperlactatemia may be seen in a number of clinical conditions not associated with tissue hypoxia, including liver disease and hypermetabolic states.
Answer: BD

DISCUSSION: Many different monitoring techniques may be used to assess the adequacy of therapy for shock. The pulmonary artery catheter can provide important hemodynamic and oxygen transport data that are very useful in directing therapy aimed at optimizing cardiac function and oxygen delivery. Pulmonary artery and pulmonary capillary wedge pressure readings should be made at end-expiration to minimize ventilatory artifacts. Continuous SvO 2 monitoring, an accurate, reliable method that combines pulmonary artery catheterization with the technique of reflectance spectrophotometry, may provide early warning signs of hemodynamic compromise or inadequate oxygen delivery. Gastrointestinal tonometry provides information that allows one to infer the adequacy of splanchnic tissue perfusion. In this technique, intramucosal pH is calculated using the Henderson-Hasselbalch equation and measurements of gut intraluminal PCO 2 and arterial bicarbonate concentration. Serum lactate concentration may be monitored in shock to detect metabolic acidosis associated with anaerobic metabolism; however, mild to moderate hyperlactatemia may also be seen with liver disease, toxin ingestion, and hypermetabolic states not associated with shock.


15. An 18-year-old man shot once in the left chest has a blood pressure of 80/50 mm. Hg, a heart rate of 130 beats per minute, and distended neck veins. Immediate treatment might include:
A. Administration of one liter of Ringer's lactate solution.
B. Subxiphoid pericardiotomy.
C. Needle decompression of the left chest in the second intercostal space.
D. Emergency thoracotomy to cross-clamp the aorta.
Answer: AC

DISCUSSION: The finding of distended neck veins in conjunction with hypotension should suggest tension pneumothorax or pericardial tamponade. Absent ipsilateral breath sounds and a trachea deviated to the contralateral side may provide additional evidence for a tension pneumothorax, the immediate treatment of which is needle decompression of the chest in the second or third intercostal space in the midclavicular line. Pericardial tamponade may initially respond to volume administration by enhancing preload. Pericardiocentesis may need to be performed emergently if hemodynamic instability persists after an initial fluid bolus when signs of compressive cardiogenic shock are present. Subxiphoid pericardiotomy should be performed only in the operating room by experienced persons who are trained to deal with penetrating cardiac injuries. There is no role for aortic cross-clamping in this scenario of cardiogenic shock.


16. Which of the following statements are true of the multiple organ dysfunction syndrome (MODS)?
A. The “two-hit” model proposes that secondary MODS may be produced when even a relatively minor second insult reactivates, in a more amplified form, the systemic inflammatory response that was primed by an initial insult to the host.
B. The systemic inflammatory response syndrome (SIRS), shock due to sepsis or SIRS, and MODS may be regarded as a continuum of illness severity.
C. Prolonged stimulation or activation of Kupffer cells in the liver is thought to be a critical factor in the sustained, uncontrolled release of inflammatory mediators.
D. The incidence of MODS in intensive care units has decreased owing to increased awareness, prevention, and treatment of the syndrome.
Answer: ABC

DISCUSSION: MODS is part of a clinical continuum that begins with the systemic inflammatory response syndrome, which is the host's stress response to any major insult such as injury or infection. MODS may develop as a result of the initial insult, but more commonly, it develops following a second or subsequent insult to the host. The two-hit theory holds that the systemic inflammatory response is amplified following the second hit, such as nosocomial pneumonia, leading to exaggerated, persistent release of inflammatory mediators that contribute to the pathogenesis of MODS. The liver appears to be a pivotal organ in the progression and outcome of MODS, partly because of the activation and prolonged stimulation of the Kupffer cells, which comprise the majority of the body's macrophage population. Macrophages are known to play a critical role in the elaboration of numerous inflammatory mediators. Despite advances in critical care and in the understanding of the pathogenesis of MODS, the incidence of MODS continues to increase without a significant improvement in outcome.


17. All of the following statements about hemorrhagic shock are true except:
A. Following hemorrhagic shock, there is an initial interstitial fluid volume contraction.
B. Dopamine, or a similar inotropic agent, should be given immediately for resuscitation from hemorrhagic shock, to increase cardiac output and improve oxygen delivery to hypoperfused tissues.
C. The use of colloid solutions or hypertonic saline solutions is contraindicated for treatment of hemorrhagic shock.
D. In hemorrhagic shock, a narrowed pulse pressure is commonly seen before a fall in systolic blood pressure.
Answer: BC

DISCUSSION: Hemorrhagic shock is associated with a contraction of the interstitial fluid compartment because of precapillary vasoconstriction and reabsorption of interstitial fluid into the vascular compartment along hydrostatic pressure gradients. Systolic hypotension may not be evident in hemorrhagic shock until at least 30% or more of blood volume is exsanguinated. A decrease in the pulse pressure (the difference between systolic and diastolic pressures) may be observed with losses of 15% to 30% of blood volume. Treatment of hemorrhagic shock includes intravascular fluid administration and definitive control of the source of the hemorrhage. Inotropic agents should not be started before volume resuscitation but may be added to improve oxygen delivery to hypoxic tissues if volume administration alone does not produce resuscitative goals. Colloid or hypertonic saline solutions are not contraindicated in the treatment of hemorrhagic shock; however, definitive evidence that such solutions are better than standard crystalloid solutions is lacking.


18. Which of the following statements about septic shock are true?
A. A circulating myocardial depressant factor may account for the cardiac dysfunction sometimes seen with shock due to sepsis or SIRS.
B. A cardiac index (CI) of 6 liters per minute per square meter of body surface, a pulmonary capillary wedge pressure of 15 mm. Hg, and a systemic vascular resistance index (SVRI) of 800 dynes-sec/(cm 5-m 2) is a hemodynamic profile consistent with septic shock.
C. An increase in SvO 2 in septic patients may be explained by the finding of anatomic arteriovenous shunts.
D. Results of human trials employing antimediator therapy, such as antiendotoxin antibodies, IL-1 receptor antagonist, and tumor necrosis factor (TNF) antibodies, have confirmed animal studies that demonstrate a significant improvement in survival with the use of such agents.
Answer: AB

DISCUSSION: Shock due to sepsis or SIRS frequently manifests as a hyperdynamic cardiovascular response, consisting of an elevated CI and a decreased SVR or SVRI. Occasionally, myocardial depression may be seen, characterized by increased ventricular volumes and decreased ejection fractions. A circulating myocardial depressant factor, possibly TNF, may be responsible for the cardiac dysfunction in such instances. The cause of the increased SvO 2 frequently observed in septic patients is unclear, but it may be secondary to bioenergetic failure, metabolic downregulation, or microcirculatory maldistribution leading to physiologic shunting. True anatomic arteriovenous shunting has not been demonstrated in humans in septic shock. Treatment of septic shock consists of appropriate antibiotic use and supportive therapy. Experimental antimediator therapies have not been encouraging thus far in human clinical trials, despite the promising results from many animal studies.


19. Which of the following statements are true of oxidants?
A. In addition to their pathophysiologic roles in inflammation, injury, and infection, oxidants also have physiologic roles.
B. Oxidants may be generated from activated neutrophils and during reperfusion following a period of ischemia.
C. The deleterious effects of oxidants include lipid peroxidation and cell membrane damage, oxidative damage to DNA, and inhibition of adenosine triphosphate (ATP) synthesis.
D. The mechanism of ischemia-reperfusion injury involved the catalytic production of superoxide anion (O 2•) by the enzyme xanthine oxidase.
Answer: ABCD

DISCUSSION: Oxidants are reactive oxygen metabolites that have both physiologic and pathophysiologic roles. As potent oxidizing agents, oxidants are involved in cytochrome P 450–mediated oxidations, for example. In pathophysiologic processes associated with inflammation, injury, and infection, oxidants may be generated by activated neutrophils and in ischemia-reperfusion injury. During ischemia, the enzyme xanthine oxidase accumulates. When oxygen availability increases during reperfusion, O 2• is formed in a reaction catalyzed by xanthine oxidase. Further oxidant formation ensues, causing the production of H 2O 2 and the extremely reactive hydroxyl ion (OH•). Oxidants may cause direct cell damage by the mechanisms of lipid peroxidation and cell membrane disruption, inhibition of ATP synthesis, reduction of cellular nicotinamide adenine dinucleotide (NAD), and oxidative damage to DNA and amino acids. In addition, oxidants may have a chemotactic role, leading to leukocyte infiltration and activation, causing further tissue damage by the release of cytotoxic proteases.


20. Which of the following statements about the role of the gut in shock and sepsis are true?
A. Selective decontamination of the digestive tract with the use of oral antibiotics has been shown to reduce nosocomial pneumonias and to improve mortality rates.
B. Enteral nutrition, as compared with parenteral nutrition, preserves the villus architecture of the gut.
C. Gut dysfunction may be an effect of shock, but it may also contribute to the development of MODS by the mechanism of bacterial translocation.
D. As compared with parenteral nutrition, enteral nutrition is associated with a reduction in septic morbidity.
Answer: BCD

DISCUSSION: The gut has a vital role in the pathophysiology of shock. The splanchnic circulation is very vulnerable to the circulatory redistribution that occurs in shock, thus, gut ischemia may occur early in the various shock syndromes. Gut injury, as a result of ischemia or reperfusion injury, leads to disruption in the intestinal mucosal barrier and increased gut permeability. Translocation of enteric flora or bacterial toxins across the gut wall may then occur, resulting in amplification of the systemic inflammatory response and the development of multiple organ dysfunction. Gut dysfunction, therefore, may perpetuate the inflammatory process. Various methods have been tried to modulate the deleterious effects of gut dysfunction. Selective decontamination of the digestive tract by oral antibiotics has been shown to reduce the incidence of nosocomial pneumonias, but no improvement in mortality has been demonstrated thus far with this controversial technique. Early enteral nutrition probably has the biggest impact on the preservation of gut architecture and function. When compared to parenteral nutrition, enteral feeding is more cost effective and is associated with a lower rate of septic morbidity.


21. Which of the following statements about head injury and concomitant hyponatremia are true?
A. There are no primary alterations in cardiovascular signs.
B. Signs of increased intracranial pressure may be masked by the hyponatremia.
C. Oliguric renal failure is an unlikely complication.
D. Rapid correction of the hyponatremia may prevent central pontine injury.
E. This patient is best treated by restriction of water intake.
Answer: A

DISCUSSION: Acute symptomatic hyponatremia is characterized by central nervous system signs of increased intracranial pressure. Changes in blood pressure and pulse are secondary to increased intracranial pressure. In the absence of hypovolemia, asymptomatic patients may be treated by restriction of water intake; however, in such patients, hyponatremia should be partially corrected by parenteral sodium administration. Rapid correction, particularly to hypernatremia, may lead to central pontine myelinolysis. Oliguric renal failure may rapidly develop in severe hyponatremia.


22. Which of the following statements about total body water composition are correct?
A. Females and obese persons have an increased percentage of body water.
B. Increased muscle mass is associated with decreased total body water.
C. Newborn infants have the greatest proportion of total body water.
D. Total body water decreases steadily with age.
E. Any person's percentage of body water is subject to wide physiologic variation.
Answer: CD

DISCUSSION: Since fat contains little water, lean persons with a proportionately greater muscle mass have a greater than expected volume of total body water. Likewise, the female body habitus and obesity contribute to decreased total body water percentage. The highest proportion of total body water is found in newborn infants, and total body water decreases steadily and significantly with age. The actual figure for a healthy person is remarkably constant.


23. Which of the following statements about extracellular fluid are true?
A. The total extracellular fluid volume represents 40% of the body weight.
B. The plasma volume constitutes one fourth of the total extracellular fluid volume.
C. Potassium is the principal cation in extracellular fluid.
D. The protein content of the plasma produces a lower concentration of cations than in the interstitial fluid.
E. The interstitial fluid equilibrates slowly with the other body compartments.
Answer: B

DISCUSSION: The total extracellular fluid volume represents 20% of body weight. The plasma volume is approximately 5% of body weight. Sodium is the principal cation. The Gibbs-Donan equilibrium equation explains the higher total concentration of cations in plasma. Except for joint fluid and cerebrospinal fluid, the majority of the interstitial fluid exists as a rapidly equilibrating component.


24. Which of the following statements are true of a patient with hyperglycemia and hyponatremia?
A. The sodium concentration must be corrected by 5 mEq. per 100 mg. per 100 ml. elevation in blood glucose.
B. With normal renal function, this patient is likely to be volume overloaded.
C. Proper fluid therapy would be unlikely to include potassium administration.
D. Insulin administration will increase the potassium content of cells.
E. Early in treatment adequate urine output is a reliable measure of adequate volume resuscitation.
Answer: D

DISCUSSION: Each 100-mg. per 100 ml. elevation in blood glucose causes a fall in serum sodium concentration of approximately 2 mEq. per liter. Excess serum glucose acts as an osmotic diuretic, producing increased urine flow, which can lead to volume depletion. Insulin therapy and the correction of the patient's associated acidosis produce movement of potassium ions into the intracellular compartment.


25. Which of the following statements about respiratory acidosis are true?
A. Compensation occurs by a shift of chloride out of the red blood cells.
B. Renal compensation occurs rapidly.
C. Retention of bicarbonate and increased ammonia formation are normal compensatory mechanisms.
D. Narcotic administration is an unusual cause of respiratory acidosis.
E. The ratio of bicarbonate to carbonic acid is less than 20:1.
Answer: CE

DISCUSSION: Renal compensation for acute hypoventilation is relatively slow. Depression of the respiratory center by morphine can lead to respiratory acidosis. Renal retention of bicarbonate, ammonia formation, and shift of chloride into red cells combine to increase the ratio of bicarbonate to carbonic acid to 20:1.


26. Which of the following statements are true of elevated–anion gap metabolic acidosis?
A. Hypoperfusion from the shock state rarely produces an elevated anion gap.
B. Retention of sulfuric and phosphoric acids may lead to this form of acidosis.
C. Copious diarrhea does not produce this condition.
D. Rapid volume expansion may produce this form of acidosis.
E. Use of lactated Ringer's solution is inappropriate in the treatment of lactic acidosis.
Answer: BC

DISCUSSION: An elevated anion gap may be produced by lactic acidosis from shock or by retention of inorganic acids from uremia. Lactated Ringer's solution rapidly corrects the lactic acidosis from hypovolemia, as lactate is converted to bicarbonate with hepatic reperfusion. Bicarbonate loss from diarrhea and “dilutional acidosis” are non–anion gap types of metabolic acidosis.


27. Which of the following is true of loss of gastrointestinal secretions?
A. Gastric losses are best replaced with a balanced salt solution.
B. Potassium supplementation is unnecessary in replacement of gastric secretions.
C. Bicarbonate wasting is an unusual complication of a high-volume pancreatic fistula.
D. Balanced salt solution is a reasonable replacement fluid for a small bowel fistula.
E. A patient with persistent vomiting usually requires hyperchloremic replacement fluids.
Answer: DE

DISCUSSION: Gastric secretions are relatively high in chloride and potassium. Other than an isolated pancreatic fistula, gastrointestinal tract losses below the pylorus are best replaced by a balanced salt solution. Although potassium concentrations are low, copious losses require potassium supplementation to prevent hypokalemia.


28. Which of the following statements regarding hypercalcemia are true?
A. The symptoms of hypercalcemia may mimic some symptoms of hyperglycemia.
B. Metastatic breast cancer is an unusual cause of hypercalcemia.
C. Calcitonin is a satisfactory long-term therapy for hypercalcemia.
D. Severely hypercalcemic patients exhibit the signs of extracellular fluid volume deficit.
E. Urinary calcium excretion may be increased by vigorous volume repletion.
Answer: ADE

DISCUSSION: Markedly elevated serum calcium levels produce polydipsia, polyuria, and thirst. Vigorous volume repletion and saline diuresis correct the extracellular fluid volume deficit and promote the urinary excretion of calcium. Metastatic breast cancer is the most common cause of hypercalcemia, from bony metastasis. The calcitonin effect on calcium is diminished with repeat administrations.


29. Which of the following statements about normal salt and water balance are true?
A. The products of catabolism may be excreted by as little as 300 ml. of urine per day.
B. The lungs represent the primary source of insensible water loss.
C. The normal daily insensible water loss is 600 to 900 ml.
D. Excessive cell catabolism causes significant loss of total body water.
E. In normal humans, urine represents the greatest source of daily water loss.
Answer: CE

DISCUSSION: The skin is the primary source of insensible water loss. Including losses from the lungs, this averages 600 to 900 ml. per day. Catabolism liberates “water of solution.” In normal humans, urine represents the greatest source of water loss. The patient deprived of external access to water must still excrete a minimum of 500 to 800 ml. of urine per day to expel the products of catabolism.


30. Which of the following is/are not associated with increased likelihood of infection after major elective surgery?
A. Age over 70 years.
B. Chronic malnutrition.
C. Controlled diabetes mellitus.
D. Long-term steroid use.
E. Infection at a remote body site.
Answer: C

DISCUSSION: Controlled diabetes mellitus has been shown repeatedly not to be associated with increased likelihood of incisional infection provided one avoids operations on body parts that may be ischemic or neuropathic. Uncontrolled diabetes mellitus, such as ketoacidosis, is associated with a dramatic increase in surgical infection. The other parameters noted—age over 70, chronic malnutrition, regular steroid use, and an infection at a remote body site—are well-recognized adverse predictive factors and are identified in tables within the chapter.


31. Which of the following are not determinants of a postoperative cardiac complication?
A. Myocardial infarct 4 months previously.
B. Clinical evidence of congestive heart failure in a patient with 8.5 gm. per dl. hemoglobin.
C. Premature atrial or ventricular contractions on electrocardiogram.
D. A harsh aortic systolic murmur.
E. Age over 70 years.
Answer: B

DISCUSSION: Clinical evidence of congestive heart failure in a patient with 8.5 gm. per dl. hemoglobin concentration is a misleading sign. Evidence of congestive failure is ordinarily a major risk factor, but in this particular patient the anemia lends itself to correction by preoperative transfusion with packed red blood cells, and often it is found that congestive failure and the associated increased risks disappear when the hemoglobin concentration is returned to the 12 gm. per dl. or higher ratio. All other factors are overt signs of increased likelihood of a postoperative cardiac event, the most ominous being a myocardial infarction 4 months preoperatively or the presence of a harsh aortic systolic murmur suggesting the presence of aortic stenosis. Age over 70 years and the presence of premature atrial or ventricular contractions on the electrocardiogram are less strong determinants of a postoperative cardiac complication.


32. Rank the clinical scenarios in order of greatest likelihood of serious postoperative pulmonary complications.
A. Transabdominal hysterectomy in an obese woman that requires 3 hours of anesthesia time.
B. Right middle lobectomy for bronchogenic cancer in a 65-year-old smoker.
C. Vagotomy and pyloroplasty for chronic duodenal ulcer disease in a 50-year-old who had chest film findings of old, healed tuberculosis.
D. Right hemicolectomy in an obese 60-year-old smoker.
E. Modified radical mastectomy in a 58-year-old woman who is obese.
Answer: BDCAE

DISCUSSION: If one considers the constellation of risk factors for pulmonary complications that is provided in tabular form in the accompanying chapter, one should readily recognize B, right middle lobectomy for bronchogenic cancer in a 65-year-old smoker, as the highest risk of a clinical situation for the likelihood of serious pulmonary complications. The next in rank may be properly debated between answer D and answer C. D, right hemicolectomy, is judged to have somewhat greater likelihood of complications since the patient is older, smokes, and is obese, although the procedure may be done through a transverse or lower abdominal incision. C, vagotomy and pyloroplasty, is viewed as being somewhat less serious since it is an upper abdominal operation on an elective basis in a 50-year-old whose only abnormalities include old, healed tuberculosis on a chest film. A very low risk of pulmonary complication should follow a transabdominal hysterectomy done through a lower abdominal incision in a woman whose only risk factors are obesity and a 3-hour anesthesia time. The lowest risk probably resides with the younger patient undergoing modified radical mastectomy, whose only risk factor is obesity. This is particularly true since this operation is conducted on the surface of the body, is associated with relatively little postoperative pain, and provides free and unrestricted respiratory function.


33. Rank the following laboratory tests and procedures in terms of their relative value to a 65-year-old woman who is to undergo elective resection of a sigmoid cancer.
A. Carcinoembryonic antigen (CEA).
B. Blood urea nitrogen (BUN).
C. Electrocardiogram (ECG).
D. Hemoglobin concentration (Hgb).
E. Serum creatinine (Cr).
F. Arterial blood oxygen tension (PaO 2) on room air.
G. Serum sodium concentration (Na+).
Answer: CDFEBAG

DISCUSSION: The most important test by far is the electrocardiogram, with its capacity to indicate signs of occult heart disease. The second most important evaluation is the hemoglobin concentration, which in this patient may show an anemia related to chronic alimentary tract blood loss that would require correction prior to safe induction of a general anesthetic. Arterial blood gases vary from individual to individual depending primarily on smoking habits and age. Accordingly, each older person should have a resting baseline determination prior to operation. Serum creatinine may show evidence of occult renal disease and is substantially more useful than blood urea nitrogen, which is more vulnerable to transient volume changes. Carcinoembryonic antigen is important to know in many patients with cancer with respect to postoperative follow-up since in some cases it may be an early herald of recurrent disease. However, it has little to do with the patient's preoperative assessment in terms of risk and preparation for an elective operation. The presence of liver metastases, for example, can be discovered with significant accuracy by palpation at the time of operation, and an elevated carcinoembryonic antigen in no set of circumstances would lead one to withhold colon resection with its relief of potential obstruction and bleeding. Finally, serum sodium concentration in a 65-year-old woman who is admitted electively for resection of the colon is always normal and would be of least value among these tests.


34. Which of the following statements regarding whole blood transfusion is/are correct?
A. Whole blood is the most commonly used red cell preparation for transfusion in the United States.
B. Whole blood is effective in the replacement of acute blood loss.
C. Most blood banks in the United States have large supplies of whole blood available.
D. The use of whole blood produces higher rates of disease transmission than the use of individual component therapies.
Answer: B

DISCUSSION: Whole blood is effective as a replacement fluid for acute blood loss because it provides both volume and oxygen-carrying capacity (red blood cells). It is rarely used in the United States nowadays, and most blood banks do not provide whole blood transfusions. It is significantly more efficient to separate donated blood into its components. In this manner, the red blood cell mass can be used to provide oxygen-carrying capacity, the plasma can be used for factor replacement, and the platelets and white cells can be used for patients deficient in these components. The use of whole blood to replace acute blood loss is associated with lower disease transmission rates than the use of packed red blood cells, fresh frozen plasma, and platelets, each from a different donor.


35. Which of the following statements about the preparation and storage of blood components is/are true?
A. Solutions containing citrate prevent coagulation by binding calcium.
B. The shelf life of packed red blood cells preserved with CPDA-1 is approximately 35 days at 1؛ to 6؛ C.
C. There are normal numbers of platelets in packed red blood cells stored at 1؛ to 6؛ C for more than 2 days.
D. The storage lesion affecting refrigerated packed red blood cells includes development of acidosis, hyperkalemia, and decreased intracellular 2,3DPG (diphosphoglycerate).
Answer: ABD

DISCUSSION: After blood has been collected from a donor, it is anticoagulated with a solution containing citrate, which acts by binding calcium. Blood is then separated into its components. Packed red blood cells stored at 1؛ to 6؛ C using CPDA-1 preservative have a shelf life of 35 days. There are essentially no functional platelets in refrigerated blood stored at 1؛ to 6؛ C after approximately 48 hours in storage. Refrigerated packed red blood cells undergo progressive changes termed a storage lesion. Such changes include acidosis, hyperkalemia, and decreased levels of 2,3-DPG, which are reversed after transfusion or produce effects other than those predicted based on the content of the unit of blood.


36. Which of the following is/are acceptable reasons for the transfusion of red blood cells based on currently available data?
A. Rapid, acute blood loss with unstable vital signs but no available hematocrit or hemoglobin determination.
B. Symptomatic anemia: orthostatic hypotension, severe tachycardia (greater than 120 beats per minute), evidence of myocardial ischemia, including angina.
C. To increase wound healing.
D. A hematocrit of 26% in an otherwise stable, asymptomatic patient.
Answer: AB

DISCUSSION: Currently accepted guidelines for the transfusion of packed red blood cells include acute ongoing blood loss, as might occur in an injured patient, and the development of symptomatic anemia with manifestations of decreased tissue perfusion related to decreased oxygen-carrying capacity of the blood. This includes situations in which the patient is unable to compensate for a decreased oxygen-carrying capacity by the usual mechanisms, such as increased cardiac output. Such patients develop myocardial dysfunction if an excessive demand is placed on the heart. The patient should be transfused with packed red blood cells, which afford added oxygen-carrying capacity. This decreases the workload on the myocardium while providing the necessary oxygen-delivery capability. The use of packed red blood cells to improve wound healing or to improve the patient's sense of well-being is highly questionable. No data support such a practice. In general, the use of a transfusion trigger such as a hematocrit of 30% or hemoglobin of 10 gm. per dl. constitutes a questionable indication for transfusion. If a patient is asymptomatic and stable and has no risk of myocardial ischemia, packed red blood cell transfusion should not be given based solely or predominantly on a numerical value such as a hematocrit of 28%.


37. The transfusion of fresh frozen plasma (FFP) is indicated for which of the following reasons?
A. Volume replacement.
B. As a nutritional supplement.
C. Specific coagulation factor deficiency with an abnormal prothrombin time (PT) and/or an abnormal activated partial thromboplastin time (APTT).
D. For the correction of abnormal PT secondary to warfarin therapy, vitamin K deficiency, or liver disease.
Answer: CD

DISCUSSION: The use of FFP as a volume expander is not indicated. There are currently several preparations (both crystalloid and colloid) that are equally effective and do not carry the infectious and other risks associated with the use of FFP. The use of FFP as a “nutritional” supplement is to be condemned. Patients with specific deficiencies of coagulation factors generally benefit greatly from the infusion of FFP. In cases of specific factor deficiency, other preparations may be more appropriate, but FFP is generally immediately available and is effective in most patients. Patients receiving warfarin therapy, those who have vitamin K deficiency, and those with liver dysfunction have abnormalities of the vitamin K–dependent factors II, VII, IX, and X, as well as protein C and protein S.


38. In patients receiving massive blood transfusion for acute blood loss, which of the following is/are correct?
A. Packed red blood cells and crystalloid solution should be infused to restore oxygen-carrying capacity and intravascular volume.
B. Two units of FFP should be given with every 5 units of packed red blood cells in most cases.
C. A “six pack” of platelets should be administered with every 10 units of packed red blood cells in most cases.
D. One to two ampules of sodium bicarbonate should be administered with every 5 units of packed red blood cells to avoid acidosis.
E. One ampule of calcium chloride should be administered with every 5 units of packed red blood cells to avoid hypocalcemia.
Answer: A

DISCUSSION: Patients who are suffering from acute blood loss require crystalloid resuscitation as the initial maneuver to restore intravascular volume and re-establish vital signs. If 2 to 3 liters of crystalloid solution is inadequate to restore intravascular volume status, packed red blood cells should be infused as soon as possible. There is no role for “prophylactic infusion” of FFP, platelets, bicarbonate, or calcium to patients receiving massive blood transfusion. If specific indications exist patients should receive these supplemental components. In particular, patients who have abnormal coagulation tests and have ongoing bleeding should receive FFP. Patients who have depressed platelet counts along with clinical evidence of oozing (microvascular bleeding) benefit from platelet infusion. Sodium bicarbonate is not necessary, since most patients who receive blood transfusion ultimately develop alkalosis from the citrate contained in stored red blood cells. The use of calcium chloride is usually unnecessary unless the patient has depressed liver function, ongoing prolonged shock associated with hypothermia, or, rarely, when the infusion of blood proceeds at a rate exceeding 1 to 2 units every 5 minutes.


39. Hemostasis and the cessation of bleeding require which of the following processes?
A. Adherence of platelets to exposed subendothelial glycoproteins and collagen with subsequent aggregation of platelets and formation of a hemostatic plug.
B. Interaction of tissue factor with factor VII circulating in the plasma.
C. The production of thrombin via the coagulation cascade with conversion of fibrinogen to fibrin.
D. Cross-linking of fibrin by factor XIII.
Answer: ABCD

DISCUSSION: Hemostasis requires the interaction of platelets with the exposed subendothelial structures at the site of injury followed by aggregation of more platelets in that area. Interactions between endothelial cell and subendothelial tissue factor with factor VII activate the coagulation cascade. The end product is large amounts of thrombin that catalyze the conversion of fibrinogen into fibrin. Fibrin thus formed is cross-linked by factor XIII to form a stable clot that incorporates the platelet plug and fibrin thrombus into a stable clot.


40. Which of the statements listed below about bleeding disorders is/are correct?
A. Acquired bleeding disorders are more common than congenital defects.
B. Deficiencies of vitamin K decrease production of factors II, VII, IX, and X, protein C, and protein S.
C. Hypothermia below 32؛C rarely causes a bleeding disorder.
D. Von Willebrand's disease is a very uncommon congenital bleeding disorder.
Answer: AB

DISCUSSION: Acquired bleeding disorders are significantly more common than congenital bleeding defects. Vitamin K deficiency may be related to malnutrition or competitive inhibition of the production of the vitamin K–dependent factors II, VII, IX, X, protein C, and protein S by warfarin (Coumadin). Hypothermia causes significant platelet dysfunction with a significant bleeding disorder in many patients. It is among the least recognized causes of altered coagulation in surgical patients. Von Willebrand's disease is a relatively common disorder of bleeding and is generally undetectable by routine screening methods.


41. The evaluation of a patient scheduled for elective surgery should always include the following as tests of hemostasis and coagulation:
A. History and physical examination.
B. Complete blood count (CBC), including platelet count.
C. Prothrombin time (PT) and activated partial thromboplastin time (APTT).
D. Studies of platelet aggregation with adenosine diphosphate (ADP) and epinephrine.
Answer: A

DISCUSSION: The evaluation of most patients scheduled for elective surgery who do not have a history of significant bleeding disorders is somewhat controversial. An adequate history and physical examination screen out most patients with bleeding problems. For patients who are scheduled to undergo a major surgical procedure, it is advisable to obtain a CBC and platelet count, as well as a PT and APTT level. This detects a large number of bleeding disorders but does not rule out all possible causes of perioperative bleeding. Studies of platelet aggregation are indicated only for patients who are suspected of having qualitative defects of platelet function (e.g., von Willebrand's disease).


42. Which of the following statements regarding the transmission of infectious agents through blood transfusions is/are true?
A. The transmission rates for human immunodeficiency virus (HIV) have been decreasing progressively since the early 1980s.
B. The transmission rates of hepatitis have been decreasing steadily since the 1980s.
C. Cytomegalovirus (CMV) is the infectious agent most commonly transmitted in blood.
D. Severely immunocompromised patients (such as patients undergoing transplantation) should receive specially screened blood products.
Answer: ABCD

DISCUSSION: The incidence of both HIV and hepatitis transmitted via blood transfusions has been steadily decreasing since the 1980s. This is related to improved methods for detection and increased awareness of surrogate markers of disease. The currently available techniques for the detection of HIV are highly effective, provided the donor is not in the “window” before the formation of specific antibody. The surrogate markers for hepatitis C, as well as the specific assays for the organism, are now sufficiently refined to allow the detection of a large percentage of hepatitis C infection in donated blood. Screening for hepatitis B surface antigen has effectively eliminated the transmission of hepatitis B through blood products in most cases. CMV is the most commonly transmitted infectious agent in blood. Since a large percentage of the population carry the virus, routine screening is not performed for this organism; however, severely compromised patients such as those undergoing transplantation should receive CMV-negative blood products.


43. The most common cause of fatal transfusion reactions is:
A. An allergic reaction.
B. An anaphylactoid reaction.
C. A clerical error.
D. An acute bacterial infection transmitted in blood.
Answer: C

DISCUSSION: The most common cause of fatalities related to transfusion reactions result from ABO-incompatible transfusion related to clerical error. Most such reactions occur if a type O person receives type A red cells owing to a clerical error that occurs either at the time the blood sample was drawn, during processing in the laboratory, or at the time a unit is administered. The importance of extremely careful labeling, transfer, and handling of specimens and of cross-matched blood products cannot be overemphasized. Allergic and other reactions are common but rarely fatal. The transmission of bacterial organisms (e.g., Staphylococcus aureus) has been reported especially with platelet concentrates maintained at or near room temperature. Fortunately, such reactions are rare.


44. Which of the following statements about the coagulation cascade is/are true?
A. The intrinsic pathway of coagulation is the predominant pathway in vivo for hemostasis and coagulation.
B. The intrinsic pathway beginning with the activation of factor XII is the predominant in vivo mechanism for activation of the coagulation cascade.
C. Deficiencies of factor VIII and IX cause highly significant coagulation abnormalities.
D. Deficiencies of factor XII cause severe clinical bleeding syndromes.
Answer: AC

DISCUSSION: Although it was previously held that two somewhat distinct pathways existed for the activation of the coagulation cascade, it is now recognized that the predominant mechanism for coagulation in vivo is the “extrinsic pathway.” Tissue factor is exposed in the subendothelial tissues when endothelial cell injury occurs. Tissue factor then tightly binds factor VII circulating in the plasma and activates the coagulation cascade. Factor VIII and factor IX deficiency cause the clinical syndromes of hemophilia A and hemophilia B, respectively. Both of these disorders involve very severe clinical bleeding disorders, whereas deficiencies of factor XII do not generally cause clinically significant bleeding. This further emphasizes the secondary role that the “intrinsic pathway” plays in coagulation.


45. A major problem in nutritional support is identifying patients at risk. Recent studies suggest that these patients can be identified. Which of the following findings identify the patient at risk?
A. Weight loss of greater than 10% over 2 to 4 months.
B. Serum albumin of less than 3 gm. per 100 ml. in the hydrated state.
C. Malnutrition as identified by global assessment.
D. Serum transferrin of less than 220 mg. per 100 ml.
E. Functional impairment by history.
Answer: ABCDE

DISCUSSION: All of these are at least partially correct. It is not clear whether weight loss of 10% or 15% is the required threshold, but it certainly is close. Serum albumin of less than 3 gm per 100 ml. remains the most constant identifier of patients at risk in the literature and has been so for years. Global assessment in the hands of an experienced investigator is quite efficacious at identifying persons at risk. Serum transferrin is certainly a confirmatory identifier of patients with malnutrition—and may be even a primary one. Graham Hill and his co-workers have pioneered the concept of global assessment using functional parameters, and in the hands of an experienced observer is quite a reasonable way of approaching and identifying patients at risk.


46. Essential fatty acid deficiency may complicate total parenteral nutrition (TPN). Which of the following statements are true?
A. Essential fatty acid deficiency may be prevented by the administration of 1% to 2% of total calories as fat emulsion.
B. Fat-free parenteral nutrition results in the appearance of plasma abnormalities, indicating essential fatty acid deficiency, within 7 to 10 days of initiation.
C. An abnormal plasma eicosatrienoic-arachidonic acid ratio is always associated with essential fatty acid deficiency.
D. Following initiation of fat-free parenteral nutrition, dry, scaly skin associated with a maculopapular rash indicates essential fatty acid deficiency.
Answer: BD

DISCUSSION: Biochemical evidence of essential fatty acid deficiency may occur as early as 7 to 10 days following initiation of fat-free parenteral nutrition. The decrease in arachidonic acid in plasma and the appearance of the abnormal eicosatrienoic acid may yield the earliest indication of prostaglandin deficiency; it is not absolute. Decreased intraocular pressure, another early indication of prostaglandin deficiency, may appear as soon as 7 days following initiation of fat-free parenteral nutrition. While my current practice is to give at least 500 ml. of 10% lipid emulsion daily to provide 20% to 25% of total calories to support hepatic protein synthesis, as little as 4% to 6% of total daily calories as fat prevents essential fatty acid deficiency. Practically, this may be undertaken by the administration of 500 ml. of 10% lipid three times weekly. The appearance of eicosatrienoic acid and a decrease in arachidonic acid, and a change in ratio, is not essential to the diagnosis of essential fatty acid deficiency, but this plasma abnormality is often present.


47. It is stated that enteral nutrition is safer than parenteral nutrition. Which of the following may be complications of enteral nutrition?
A. Hyperosmolar, nonketotic coma.
B. Vomiting and aspiration.
C. Pneumatosis cystoides intestinalis.
D. Perforation and peritonitis.
Answer: ABCD

DISCUSSION: It is not necessarily true that enteral nutrition is safer than parenteral nutrition, and it may in fact be associated with a higher risk of death than parenteral nutrition. Specifically, a well-run parenteral nutrition service should not be associated with significant mortality, except for the occasional death due to undetected yeast infection. On the other hand, enteral nutrition, especially if not carried out safely, can result in significant mortality. The most common of the severe complications of enteral nutrition result from the gastrostomy, or tube feedings into the stomach. Sudden changes in gastric motility, such as those associated with sepsis, may result in aspiration. Nasoenteric or nasoduodenal tubes help prevent this complication, as does shutting off enteral feedings between the hours of 11 P.M. and 7 A.M. It is also essential to keep the patient's head elevated 30 degrees. Also necessary is the use of extreme care when initiating enteral nutrition. If hypertonic material is given into the stomach, one can increase osmolality followed by an increase in volume. If, however, the material is given into the small bowel, volume must be increased first and then tonicity, with the expectation that osmolality greater than 400 or 500 mOsm per liter may never be achieved without provoking severe diarrhea. If care is not taken with the initiation of enteral nutrition, massive diarrhea may result, including fluid loss, the absorption of enormous amounts of carbohydrate into the circulation with inadequate fluid to support it, and the development of hyperosmolar, nonketotic coma. Alternatively, severe unremitting diarrhea may result in necrosis of the intestinal wall, the appearance of pneumatosis cystoides intestinalis, and, finally, perforation and death. All of these complications may be prevented by judicious use of enteral nutrition with the same care one uses for parenteral nutrition.


48. It has been suggested that enterocyte-specific fuels be utilized for all patients receiving parenteral nutrition. Theoretically, the benefits of such fuels include:
A. Glutamine increases gut mucosal protein content and wall thickness.
B. Butyrate increases jejunal mucosal protein content and wall thickness.
C. The short-chain fatty acids—butyrate, propionate, and acetate—are useful in supporting ileal mucosal protein content and thickness.
D. The use of glutamine-enriched solutions for parenteral nutrition for patients with chemotherapy toxicity or radiation enteritis is without hazards.
Answer: NONE IS ENTIRELY TRUE

DISCUSSION: The use of enterocyte-specific fuels is part of a new and potentially exciting phase of “nutritional pharmacology” in parenteral nutrition; however, exciting as the research may be, the use of such fuels is by no means acceptable for indiscriminate use at present. Though some studies have shown that the provision of glutamine in amounts up to 2% in standard parenteral nutrition solutions increases both jejunal and ileal mucosal protein content, cell wall thickness, and DNA content, this has not been the case in all studies, and this reported effect seems very dependent on experimental design. In many of the studies that have shown such an effect, 2% glutamine has been used to replace virtually all nonessential amino acids, probably initiating a deficiency state. The beneficial effects seen with glutamine are far less impressive than those seen with epidermal growth factor, for example, and disappear entirely when a different experimental design is used in which 2% glutamine is added to an adequate amino acid formulation in which glutamine does not replace nonessential amino acids but is added to them. Nonetheless, the use of enterocyte-specific fuels, specifically glutamine, is potentially exciting and should be carefully investigated. More striking are the results that follow massive bowel resection, radiation enteritis, and chemotherapy toxicity. Glutamine may help the small bowel regenerate more quickly, enabling more rapid use of the small bowel for nutrition. It should be pointed out, however, that glutamine is a fuel utilized by many tumors and, thus, one runs the risk of stimulating the growth of the tumor with excessive glutamine. The short-chain fatty acids, produced from bacterial fermentation of soluble pectin, may be useful in both the maintenance of colonocyte-specific nutrition and, in the case of butyrate, ileal enterocyte nutrition.


49. Essential amino acids have been advocated as standard therapy for renal failure. Which of the following statements are true?
A. Increased survival from acute renal failure has been reported with both essential and nonessential amino acid therapy of patients in renal failure.
B. Essential amino acids retard the rise of blood urea nitrogen (BUN) secondary to decreased urea appearance.
C. Essential amino acids and hypertonic dextrose are a convenient form of therapy for hyperkalemia.
D. Essential amino acids decrease BUN and creatinine to the same degree as solutions containing excessive nonessential amino acids.
Answer: BC

DISCUSSION: Essential amino acids and hypertonic dextrose, as opposed to hypertonic dextrose alone, was reported by Abel and co-workers to be associated with a decreased mortality rate in mostly surgical patients with acute tubular necrosis. The most significant improvement in mortality, as compared with the control group receiving hypertonic dextrose, was among patients who required dialysis (i.e., the more severely affected patients). Another group responding favorably to treatment includes patients with nonoliguric renal failure whose need for dialysis is not clearly established. The effect of essential amino acids in preventing a rise in BUN, as well as its beneficial effect in preventing hyperkalemia, may obviate dialysis in such patients. With increasing amounts of nonessential amino acids, BUN increases, and thus, dialysis is required. Prospective randomized studies comparing the use of essential versus nonessential amino acids in patients with acute renal failure have not been carried out in sufficient numbers to yield answers to this question.


50. A modified amino acid solution with increased equimolar branched-chain amino acids and decreased aromatic amino acids has been proposed for patients with hepatic insufficiency. Which of the following statements is/are true?
A. This formulation is proposed for the use of patients with fulminant hepatitis.
B. Nitrogen balance is achieved in such patients with amounts of 40 gm. of amino acids per 24 hours.
C. The use of 80 to 100 gm. of such solutions is associated with hepatic encephalopathy.
D. In some studies of surgical patients, improvements in mortality have been reported.
Answer: D

DISCUSSION: The use of modified amino acid solutions is based on the false neurotransmitter hypothesis of the cause of hepatic coma. According to this hypothesis, the imbalance between aromatic and branched-chain amino acids in the plasma results in abnormally high levels of the toxic aromatic amino acids in the brain, thus provoking hepatic encephalopathy. The use of modified amino acid mixtures, with glucose as the calorie base, has been associated in a number of studies with improvement in encephalopathy. Meta-analysis has concluded that the use of such solutions is indicated as therapy for hepatic encephalopathy but has been proposed only for hepatic encephalopathy complicating acute exacerbation of chronic liver disease. Although there are a few anecdotal reports of beneficial effects on hepatic encephalopathy of acute fulminant hepatitis, the use of such a solution has not been advocated, but such a modified solution is tolerated better than standard amino acid mixtures in patients requiring TPN. In some studies, particularly in complicated surgical cases, the use of a high–branched-chain, low–aromatic amino acid solution has been associated with lower mortality. These statements are true only for studies in which the modified solutions are given with hypertonic glucose as a calorie base. Studies in which lipid was the principal calorie source have not revealed such improvements in mortality. In recent studies, giving an aromatic amino acid–deficient, branched-chain amino acid–enriched solution to patients about to undergo resection of the liver has proved particularly efficacious in a group of patients with cirrhosis, decreasing morbidity and showing a trend toward decreased mortality.


51. In the nutritional support of patients with cancer, which of the following statements is/are true?
A. Nutritional support benefits the patient's lean body mass but does not enable the tumor to grow.
B. In experimental animals, the growth of implanted tumors is directly proportional to the amount of calories and protein supplied.
C. Prospective randomized trials of nutritional support utilizing chemotherapy and radiation therapy have revealed benefits to patients receiving total parenteral nutrition.
D. Studies of nutritional support for patients with cancer about to undergo surgery revealed decreased morbidity and mortality, especially morbidity from sepsis.
Answer: B

DISCUSSION: The problem with the patient with cancer is a very vexing one. Clearly, one of the metabolic effects of cancer, cachexia, affects patients in the last quartile of their disease and makes such patients intolerant of chemotherapy, radiation therapy, and, in many cases, operative procedures. Total parenteral nutrition (TPN) has been proposed as a means of reversing cachexia and enabling patients to better tolerate surgery, chemotherapy, and radiation therapy. In experimental animals, it is clear that the provision of calories and protein, especially in excessive amounts, is associated with the more rapid growth of tumors and decreased survival, especially in the group that is overfed in the extreme. There is also evidence suggesting that overfeeding, or at least TPN, may result in increased growth (or at least change cell kinetics) in patients who are overnourished with TPN. Of the randomized prospective trials that have been carried out, no trial utilizing chemotherapy or radiation therapy has revealed a survival advantage for patients receiving TPN. Indeed, in Shamberger's study, there is a suggestion that the tumor-free interval following treatment of lymphoma may be shorter in patients receiving TPN. In patients undergoing surgery, however, especially those who are severely malnourished (as recently revealed in the VA study) or in patients with major procedures such as esophagogastrectomy (as in Muller's study), evidence suggests that TPN is beneficial. In a late follow-up in Muller's study, there was no apparent increase in recurrence, and the survival rate was the same, despite much higher mortality in the non-TPN group. This suggests that any improved survival following operation may have been offset by an increased late recurrence rate, although it is difficult to reach this conclusion. In summary, for patients with cancer TPN probably nourishes the tumor as well as the host. Nonetheless, in severely malnourished patients provision of TPN from 5 to 10 days preoperatively may increase survival and decrease morbidity. Overfeeding must be avoided. Future studies will undoubtedly reveal that there are certain nutrients that tumors require, which probably should be best avoided.


52. Glucose overload results in increased CO 2 production. Which of the following statements are true?
A. In patients with respiratory insufficiency, administration of glucose as a principal calorie source is contraindicated.
B. In patients with pulmonary infection and sepsis, calorie support should consist of 95% fat and 5% glucose.
C. In Askanazi's study, increased CO 2 production and difficulty in weaning was associated only with pronounced overfeeding.
D. CO 2 production should be measured in most patients who are supported by respirators in intensive care units and are receiving nutritional support.
Answer: C

DISCUSSION: Few papers have excited as much interest as that by Askanazi, Kinney, and co-workers, which demonstrated that glucose calories given to patients with severe respiratory impairment may result in difficulty in weaning from a respirator. Subsequent research has suggested, however, that this occurs only with severe overfeeding, when the respiratory quotient is greater than 1 and calories are excessive. If one examines the conditions under which Askanazi's patients were studied, these were a group of septic, depleted patients who were taken from almost no nutritional support to a caloric supply of 2.25 times their caloric requirement, most of the calories consisting of glucose. Suffice it to say that, in patients with impaired respiratory function, one should measure VCO2 and, when VCO2 is significantly elevated and appears to interfere with weaning, decrease the amount of glucose calories and increase the amount of fat. If one measures or estimates calorie requirements and does not overfeed, lipid can be utilized for 25% of the caloric requirement and glucose for the remainder, without much fear of excessive CO 2 production.


53. Hepatic abnormalities have been noted in adults since the beginning of hyperalimentation. Which of the following statements are true?
A. Hepatic steatosis appears to be associated with an overload of glucose.
B. Hepatic steatosis is usually associated with abnormalities in hepatic enzymes.
C. Hyperbilirubinemia is inevitably associated with hepatic steatosis.
D. Abnormalities in the portal insulin-glucagon ratio are thought to be causative of hepatic steatosis in experimental animals.
Answer: AD

DISCUSSION: The most common metabolic complication of TPN in adults is hepatic steatosis. Unlike the hepatic abnormalities in children, which may progress to cholestasis, liver damage, and in some cases death, hepatic steatosis, or fatty infiltration of the liver with triglycerides, appears to be a rather benign complication. It may be, but is not necessarily, associated with hepatic enzymatic abnormalities, which usually occur in the first week, peak at the third week, and generally disappear by the sixth week of parenteral nutrition. Abnormalities in the transaminases are most common, with alkaline phosphatase also being elevated, but there is no correlation between the degree of fatty infiltration and enzymatic abnormalities. Fatty infiltration appears to be largely vacuolization with increased storage of triglycerides. Hepatic steatosis is almost always associated with an overload of glucose. Recent studies in experimental animals have suggested that the portal insulin-glucagon ratio, which is elevated under these circumstances, may be causally related to hepatic steatosis. Insulin is the leading storage enzyme and is responsible for lipogenesis. The presence of insulin inhibits lipolysis. Glucagon, on the other hand, results in the mobilization of hepatic lipid. The liver “sees” the portal vein insulin-glucagon ratio. Excesses of insulin elicited by hypertonic dextrose increase lipid deposition in the liver, whereas glucagon, which is elicited by certain amino acids, results in the mobilization of hepatic lipid.


54. Which of the following statements about the presence of gallstones in diabetes patients is/are correct?
A. Gallstones occur with the same frequency in diabetes patients as in the healthy population.
B. The presence of gallstones, regardless of the presence of symptoms, is an indication for cholecystectomy in a diabetes patient.
C. Diabetes patients with gallstones and chronic biliary pain should be managed nonoperatively with chemical dissolution and/or lithotripsy because of severe complicating medical conditions and a high operative risk.
D. The presence of diabetes and gallstones places the patient at high risk for pancreatic cancer.
E. Diabetes patients with symptomatic gallstones should have prompt elective cholecystectomy, to avoid the complications of acute cholecystitis and gallbladder necrosis.
Answer: E

DISCUSSION: Gallstones have been found to be very prevalent in patients with type II (non–insulin-dependent) diabetes mellitus, perhaps related to the dyslipoproteinemia in such patients. Although the complications of acute cholecystitis (infection, sepsis, gangrene of the gallbladder) are more common in diabetics, a decision-analysis study has shown that prophylactic cholecystectomy cannot be justified since the risk of morbidity and/or mortality from the cholecystectomy procedure is as great as that of complications or death from acute cholecystitis. Patients who become symptomatic should be promptly prepared and should undergo elective cholecystectomy, because an emergency operation in these patients with comorbid conditions, especially coronary artery disease, has substantial added mortality associated with it. There is no causal relationship between diabetes and pancreatic cancer.


55. Intensive insulin therapy:
A. Prevents the aggressive development of atherosclerosis in diabetic patients.
B. Is not associated with unawareness of hypoglycemia.
C. Improves peripheral neuropathy.
D. Improves established retinopathy and nephropathy.
E. Is indicated in all patients with non–insulin-dependent diabetes mellitus (NIDDM).
Answer: C

DISCUSSION: Intensive insulin therapy is indicated in patients with IDDM who can actively participate in their own management and the attainment of the goals set for their blood glucose and glycosylated hemoglobin (HgA1 c) levels. Because the main complication of intensive therapy is iatrogenic hypoglycemia, this mode of treatment is not indicated for patients with NIDDM, who often have coexisting medical conditions such as coronary artery disease and who tolerate hypoglycemia poorly. There is little or no evidence that macrovascular disease is affected by intensive insulin therapy, and the added weight gain and hyperinsulinemia associated with the therapy may worsen atherosclerosis. Unawareness of hypoglycemia is directly related to a recent hypoglycemia episode, so patients treated intensively are often unaware of the problem. Intensive therapy does not improve established retinopathy or nephropathy but slows or prevents progression of these complications; however, better glucose control may improve peripheral neuropathy.


56. Which of the following statements about hypertension in diabetes patients is/are correct?
A. Hypertension worsens the macrovascular disease of diabetes patients.
B. Hypertension accelerates the progression of diabetic nephropathy.
C. Hypertension is associated with sodium retention in diabetes patients.
D. Angiotensin-converting enzyme (ACE) inhibitors should be used in all patients with chronic hyperglycemia, regardless of the presence of hypertension.
E. Diuretics, as single-drug therapy, are not indicated in the treatment of hypertension in diabetes patients.
Answer: ABCDE

DISCUSSION: All of the answers listed are correct. By damaging endothelial cells, hypertension worsens macrovascular disease in all patients but especially in diabetics. Hypertension dramatically accelerates the onset and progression of diabetic renal disease and proteinuria, and this phenomenon can be slowed or prevented by a combination of treatment modalities, including ACE inhibitors, which dilate efferent glomerular vessels and lower intraglomerular pressure. Despite sodium retention in diabetes patients, single-drug therapy with a diuretic is not indicated because the chronic state of dehydration in such patients may become worse.


57. What is the major determinant in an individual patient's risk for perioperative complications?
A. The surgical procedure.
B. The length of the surgical procedure.
C. The anesthetic technique (e.g., general, regional).
D. The length of anesthesia.
E. All of the above.
Answer: A

DISCUSSION: The planned surgical procedure is the major determining factor in assessing an individual patient's risk for perioperative complications and in deciding which anesthetic technique will be most appropriate. Good communication between the surgeon and the anesthesiologist is vital, as the surgeon knows better than anyone else the extent of the operation and the length of time it will require.


58. Which of the following are considered routine intraoperative monitors?
A. Temperature probe.
B. Electrocardiogram.
C. Capnograph.
D. Blood pressure cuff.
E. Foley catheter.
Answer: ABD

DISCUSSION: The American Society of Anesthesiologists requires that the patient's ventilation, circulation, oxygenation, and temperature be continually monitored during all anesthetics. Routine monitors are considered to be a temperature probe, electrocardiogram, pulse oximetry, and blood pressure cuff.


59. Muscle relaxants can be used for which of the following?
A. To facilitate intubation.
B. To provide optimal surgical conditions.
C. To optimize ventilator support.
D. To provide sedation.
Answer: ABC

DISCUSSION: Muscle relaxants are administered to facilitate endotracheal intubation, to provide the surgeon with optimal working conditions during anesthesia, and to optimize mechanical ventilator support in some patients. They do not produce analgesia, sedation, or amnesia. Therefore, muscle paralysis should not be performed without sedation or general anesthesia.


60. Local anesthetics:
A. Inhibit transmission of nerve impulses by increasing sodium membrane permeability and the displacement of ionized calcium.
B. Are classified as amides or esters.
C. Produce peripheral vasodilation.
D. Are weak acids.
Answer: BC

DISCUSSION: Local anesthetics act within the nerve membrane, where they inhibit transmission of nerve impulses by reducing sodium membrane permeability and the displacement of ionized calcium. All local anesthetics consist of a hydrophilic region and a hydrophobic region separated by an alkyl chain. The bond of the alkyl chain is either an ester or an amide, and these drugs are classified based on this bond. All local anesthetics except cocaine produce vasodilatation and are weak bases.


61. Absolute indications for a double-lumen endotracheal tube during thoracic surgery are:
A. Massive hemorrhage from one lung.
B. Unilateral lung infection.
C. Facilitation of surgical exposure.
D. Unilateral bronchopulmonary lavage.
E. All of the above.
Answer: ABD

DISCUSSION: The absolute indications for a double-lumen tube are for the purposes of protecting one lung from the other. These indications include ventilation with a bronchopleural fistula, massive hemorrhage from one lung, pulmonary air cyst resection, unilateral lung infection, and unilateral bronchopulmonary lavage. Relative indications include facilitation of surgical exposure, for pneumonectomy, upper lobectomy, and thoracic aneurysm repair.


62. Determinants of cerebral blood flow include:
A. Preoperative neurologic dysfunction.
B. Arterial CO 2 tension.
C. Arterial O 2 tension.
D. Systemic arterial pressure.
E. All of the above.
Answer: BCD

DISCUSSION: Determinants of cerebral blood flow include arterial CO 2 and O 2 tensions, systemic arterial pressure, and temperature. Other factors that may affect cerebral blood flow and intracranial pressure are head position, jugular venous obstruction, and positive end-expiratory pressure.


63. Discharge criteria following ambulatory surgery include:
A. Ability to eat solid food.
B. Stable vital signs.
C. Ability to ambulate.
D. Ability to have protective airway reflexes.
Answer: BCD

DISCUSSION: Discharge criteria following ambulatory surgery include the patient's being fully awake and oriented, the ability to have protective airway reflexes, stable vital signs, adequate hydration with the ability to hold down oral intake, the ability to ambulate, and adequate pain control. All patients must have a competent person with them to transport them—and ideally to stay with them on the first postoperative night.


64. Advantages of patient-controlled analgesia (PCA) include:
A. Immediate medication delivery.
B. Less contact with nursing staff.
C. Rapid onset of analgesia.
D. Patient control over pain medication.
E. All of the above.
Answer: ACD

DISCUSSION: Advantages of PCA are immediate medication delivery, rapid onset of analgesia, and patient control over pain medication. Disadvantages of PCA are less contact with nursing staff and patients' fears that they could inadvertently administer an overdose or possibly become addicted to the opioid.


65. Advantages of epidural analgesia include:
A. Earlier mobilization after surgery.
B. Earlier return of bowel function.
C. Shorter hospitalizations.
D. Decreased stress response to surgery.
E. All of the above.
Answer: E

DISCUSSION: Epidural analgesia include excellent pain relief, decreased sedation with more rapid recovery to presurgical levels of consciousness, earlier mobilization after surgery with increased ability to co-operate with respiratory therapy and physical therapy. Following vascular surgery epidural analgesia may also improve graft flow through mild sympathetic blockade. Earlier return of bowel function, decreased stress response, shorter hospitalizations, and decreased morbidity have all been associated with epidural analgesia.


66. Ketorolac:
A. Is a nonsteroidal anti-inflammatory drug (NSAID) approved for intravenous, intramuscular, and oral administration.
B. Can be used indefinitely for postoperative analgesia.
C. Can cause renal dysfunction.
D. May decrease surgical blood loss.
Answer: AC

DISCUSSION: Ketorolac tromethamine, an NSAID, is approved by the FDA for intravenous, intramuscular, and oral administration. The agent is an effective analgesic with minimal side effects; however, ketorolac, like all NSAIDs, can enhance surgical bleeding and cause renal and platelet dysfunction. Additionally, it is recommended that ketorolac should not be used for more than 5 consecutive days.


67. Factors that decrease collagen synthesis include all of the following except:
A. Protein depletion.
B. Infection.
C. Anemia.
D. Advanced age.
E. Hypoxia.
Answer: C

DISCUSSION: Collagen synthesis, an integral part of wound healing, is affected by many local and systemic factors. Protein depletion impairs fibroplasia. Hypoproteinemia leads to diminution of fibroblast proliferation, proteoglycan and collagen synthesis, angiogenesis, and wound remodeling. Although anemia was once believed to be a significant cause of wound disruption, studies have shown that, in the absence of malnutrition or hypovolemia, anemia with a hematocrit greater than 15% does not interfere with wound healing. In contrast, molecular oxygen is critical for collagen synthesis because it is one of the factors required for the hydroxylation of lysine and proline. Also, hypoxia favors wound infection. The role of age in collagen synthesis is not clear, but the incidence of wound failure and incisional hernias is greater in patients older than 60. Fibroplasia occurs at a slower rate in older animals. Perhaps more than any other factor, wound infection is associated with the risk of wound failure.


68. Wound contraction and ultimate contracture may be controlled by which of the following drugs?
A. Colchicine.
B. D-Penicillamine.
C. Thiphenamil (Trocinate).
D. Glucocorticoids.
E. Ibuprofen (Motrin).
Answer: AC

DISCUSSION: Wound contraction is carried out by highly specialized cells called myofibroblasts, which, as their name implies, have histologic characteristics of fibroblasts and smooth muscle cells. The activity of these cells, and therefore wound contraction, can be influenced by topical application of smooth muscle inhibitors such as thiphenamil. Inhibitors of microtubule formation in myofibroblasts, such as colchicine and vinblastine, also inhibit wound contraction under experimental conditions. Glucocorticoids and NSAIDs do not affect the wound contraction process.


69. Which of the following is/are true of the actions of transforming growth factor beta (TGF-b) during wound repair?
A. Increased matrix and proteoglycan synthesis.
B. Inhibition of proteases.
C. Stimulation of plasminogen inhibitor.
D. Chemotaxis for fibroblasts and macrophages.
E. Autoinduction of TGF-b.
Answer: ABDE

DISCUSSION: Through autocrine and paracrine mechanisms TGF-b stimulates the deposition of collagen and other matrix components by fibroblasts, inhibits proteases, blocks plasminogen inhibitor, enhances angiogenesis, and is chemotactic for fibroblasts, monocytes, and macrophages. TGF-b modulates the expression of cell-surface integrins in a manner that enhances cell-matrix interaction and matrix assembly. TGF-b also induces cell production by cells, thus amplifying its biologic effects. The sustained production of TGF-b at the wound site leads to tissue fibrosis.


70. In contrast to adult wound healing with scar formation, which of the following are characteristic of scarless fetal skin repair?
A. Matrix rich in hyaluronic acid.
B. Increased inflammatory response.
C. Increased production of TGF-b.
D. No collagen deposition.
E. Minimal angiogenesis.
Answer: AE

DISCUSSION: The ability of a fetus to heal without scar formation depends on its gestational age at the time of injury and the size of the wound defect. In general, linear incisions heal without scar until late in gestation, whereas excisional wounds heal with scar at an earlier gestational age. The profiles of fetal proteoglycans, collagens, and growth factors are different from those in adult wounds. The less differentiated state of fetal skin is probably an important characteristic responsible for scarless repair. There is minimal inflammation and angiogenesis in fetal wounds. Fetal wounds are characterized by high levels of hyaluronic acid and its stimulator(s) with more rapid, highly organized collagen deposition. The roles of peptide growth factors such as TGF-b and basic fibroblast growth factor are less prominent in fetal than in adult wound healing. An understanding of scarless tissue repair has possible clinical applications in the modulation of adult fibrotic diseases and abnormal scar-forming conditions.


71. Which of the following cell types are not crucial for healing a clean, incisional wound?
A. Macrophage.
B. Platelet.
C. Fibroblast.
D. Polymorphonuclear leukocyte.
E. Myofibroblast.
Answer: DE

DISCUSSION: Experimental studies have shown that healing may progress normally in the absence of polymorphonuclear leukocytes in an uninfected wound. In contrast, depletion of monocytes and macrophages causes a severe alteration in wound healing with poor débridement, delayed fibroblast proliferation, and inadequate angiogenesis. Platelets carry a cadre of biologically active substances that are important for wound repair, including peptide growth factors like platelet-derived growth factor (PDGF) and TGF-b. Fibroblasts are the principal cell for matrix synthesis and deposition. Myofibroblasts are important for wound contraction in open defects but have little if any role in clean, incisional wounds.


72. Which of the following is/are not a substrate or cofactor for prolyl hydroxylase?
A. Alpha-ketoglutarate.
B. Ascorbate.
C. Biotin.
D. Oxygen.
E. Copper.
Answer: CE

DISCUSSION: Prolyl hydroxylase is one of the rate-limiting enzymes in collagen synthesis. Substrates and cofactors such as iron, alpha-ketoglutarate, ascorbate, and oxygen are important participants in this process. If insufficient prolines are hydroxylated, then the alpha-peptide collagen chains cannot assume a stable triple helix, the collagen cannot be exported from the fibroblasts, and the incomplete, unassociated alpha chains are broken down. Thus, ascorbate deficiency (scurvy) and hypoxia have similar effects on collagen synthesis.


73. Which of the following is an adhesion glycoprotein?
A. Fibronectin.
B. Tenascin.
C. Laminin.
D. Hyaluronic acid.
E. Collagen type IV.
Answer: ABC

DISCUSSION: Cell adhesion glycoproteins such as fibronectin, vitronectin, laminin, and tenascin provide a “railroad track” to facilitate epithelial and mesenchymal cell migration over the wound matrix. Hyaluronic acid is a glycosaminoglycan, and collagen type IV is a protein that is a crucial component of basement membrane.


74. Which of the following is/are true concerning wound fibroblasts?
A. Fibroblasts synthesize and secrete collagen molecules.
B. Wound fibroblasts are derived from blood-borne precursor cells.
C. Fibroblasts migrate to a wound along fibrin strands, which are used as a scaffold.
D. Large amounts of fibrin or blood clot can act as a physical barrier to fibroblast penetration, which delays normal wound healing.
Answer: ACD

DISCUSSION: Fibroblasts appear in the wound on about the third day of healing and begin to synthesize and secrete collagen molecules. Wound fibroblasts arrive from cells surrounding the wound (e.g., the adventitia of blood vessels), change their phenotype(s), and become mobile during the process of replication. Fibroblasts migrate into a wound using the provisional fibronectin and fibrin matrix as a scaffold. Fibroblasts do not have fibrinolytic enzymes, and large amounts of fibrin and blood clot prevent fibroblasts from entering the wound.


75. Which of the following is/are true?
A. Because of its thickness, the tensile strength of a healing wound on the eyelid is much less than one on the thick skin of the back.
B. By 2 days, the experimental burst strength of skin is minimal since collagen has been formed in the wound but has not yet cross-linked.
C. Wound strength reaches a plateau by 3 weeks.
D. Wounds rarely, if ever, regain the strength of intact tissues.
Answer: BD

DISCUSSION: Tensile strength measures load per cross-section area at rupture, whereas burst strength measures load required to break a wound, regardless of dimension. Therefore, skin wounds have comparable tensile strength, regardless of thickness. Collagen appears in the wound by 3 to 4 days. Minimal wound strength on day 2 is due to fibrin polymerization and adhesion of globular proteins. Wounds rapidly gain strength for about 4 months and then continue to gain strength at a slower rate for more than a year. Wounds do not regain the strength of normal tissue.


76. Which of the following interfere with normal collagen formation or cross-linking?
A. Beta-aminopropionitrile.
B. Iron chelators.
C. Vitamin C depletion.
D. Proline analogs (e.g., cis-hydroxyproline).
E. D-Penicillamine.
Answer: ABCDE

DISCUSSION: Intramolecular and intermolecular cross-links are crucial for collagen structural stability. Formation of cross-links can be inhibited by two pharmacologic agents: beta-aminopropionitrile inhibits the enzyme lysyl oxidase, and D-penicillamine binds to collagen substrate directly to prevent collagen cross-link formation. Iron is a cofactor for prolyl hydroxylase, which is important for collagen synthesis. In high enough concentration, proline analogs prevent collagen formation with minimal effects on noncollagenase protein synthesis.


77 Which of the following statement(s) is/are true concerning the cell plasma membrane?
a. The plasma membrane is composed of amphipathic molecules
b. The hydrophobic core of the lipid bilayer of the cell membrane contains specialized transport proteins which maintain the intracellular ionic milieu different from the extracellular fluid
c. Plasma membrane proteins extend externally and bear phospholipid moieties which contribute to the cell coat
d. The membrane proteins of nerve cells are highly voltage-dependent
Answer: a, b, d

The plasma membrane defines the boundary of the cell and serves to contain and concentrate enzymes and other macromolecule constituents. The plasma membrane is composed of amphipathic molecules, mainly phospholipids and proteins that contain distinct regions that are either insoluble in water (hydrophobic) or soluble in water (hydrophilic). The plasma membrane forms a continuous barrier between the aqueous extracellular and intracellular fluids. Transport proteins in the membrane act as regulated channels or transporters to maintain the intracellular ionic milieu that is clearly different from the extracellular milieu. In some cells, membrane proteins are diversified such as in nerve cells where the ion channels are highly voltage-dependent, providing the basis for information transmission in the form of electrical impulses. Most plasma membrane proteins extend externally and bear carbohydrate moieties primarily as oligosaccharide chains that contribute to the cell coat or glycocalyx.

78 Which of the following statement(s) is/are true concerning water movement across cell membranes?
a. Water moves only actively through cell membrane transport proteins
b. For most cells of the body, the transmembrane hydrostatic pressure is 0
c. Water distribution is determined entirely by solute distribution
d. Specialized cells such as the glomerulus of the kidney actively transport water to maintain hydrostatic pressure
Answer: b, c

The energetics of water transport across cell membranes is simplified by the fact that water moves only passively due to gradients of hydrostatic pressure or water concentration. Hydrostatic pressure is an important driving force only for certain specialized cells—the capillary endothelium and the glomerulus of the kidney. For most cells of the body, the transmembrane hydrostatic pressure is 0 and water moves only in response to water concentration gradients. Because the concentration of water is determined by the amount of dissolved solute, the difference in water concentration is typically expressed as a function of the difference in solute concentration or osmotic pressure difference. Because there are no specialized, energy-converting transport mechanisms for water, water is distributed at equilibrium. Water distribution is determined entirely by solute to solute distribution.


79 The transport of proteins out of the cell is termed exocytosis. Which of the following statement(s) is/are true concerning this process?
a. Secretory vesicles fuse with the plasma membrane
b. The process can occur in either a constitutive or regulated process
c. A regulated secretion is triggered by a stimulus, most likely a hormone or a neurotransmitter
d. A decrease in cytoplasmic calcium occurs as part of the secretion process
Answer: a, b, c

Transport vesicles that bud off the Golgi network carry both material to be secreted from the cell and protein destined to become components of the plasma membrane. These vesicles can fuse with the plasma membrane in a process termed exocytosis. Vesicular transport to the cell surface can be divided into two components, constitutive and regulated secretion. Regulated secretion occurs in cells secreting digestive enzymes, hormones and other regulatory molecules, and neurotransmitters. In regulated secretion, the material to be secreted is sorted in a storage vesicle or granule; fusion with the plasma membrane in exocytosis then takes place in response to external stimulation. Regulated secretion is triggered in most cases by a hormone or neurotransmitter. The ensuing process is termed stimulus-secretion coupling. In most cases the coupling involves an increase in cytoplasmic concentration of Ca++, but may also involve generation of diacylglycerol or production of cyclic AMP which activate kinases or phosphatases.

80 Which of the following statement(s) is/are true concerning the cell function of phagocytosis?
a. Phagocytosis is a mechanistically distinct process of endocytosis performed by special cells to take up larger particles such as bacteria or erythrocytes
b. Lymphocytes are the primary blood cell involved with this process
c. The process involves a coating of the cytoplasmic surface known as clathrin
d. Phagocytosis is performed only by white blood cells and tissue macrophages
Answer: a

Phagocytosis is a specialized form of endocytosis by which large particles are internalized by specialized cells primarily macrophages and neutrophils. To be phagocytosed, particles must bind to the surface of the phagocytic cell, usually as the result of specific antibody coating the particle. The phagocytic cell then extends pseudopods which engulf the particle. This event is followed by membrane fusion and a pinching off. As opposed to endocytosis, this process does not involve the membrane protein, clathrin, but rather actin. A physiologically relevant site of phagocytosis is the thyroid gland, where thyroid follicular cells phagocytose and digest thyroglobulin from the lumen of the thyroid follicle, thereby releasing the thyroid hormones, thyroxine triiodothyronine.

81 A striking feature of living cells is a marked difference between the composition of the cytosol and the extracellular milieu. Which of the following statement(s) concerning the mechanisms of maintenance of these differences is/are true?
a. The cell membrane is able to maintain a 10,000 fold gradient between the extracellular concentration of ionized calcium and the intracellular concentration
b. The key to these differences is the fact that the plasma membrane is normally impermeable to sodium, potassium and calcium
c. The selectivity of biologic membranes is highly consistent and seldom changes
d. The selectivity of cell membranes relates only to ions and not organic compounds

Answer: a

The survival of the cell requires that cytosolic composition be maintained within narrow limits, despite the constant influx of nutrients and the simultaneous outflow of waste. A familiar example of the distribution of ions across the cell membrane is that of sodium and potassium. Cells are typically rich in potassium and contain very little sodium. Despite the fact that they are constantly bathed by fluid that is precisely the opposite composition. Even more impressive is the distribution of ionized calcium. The extracellular concentration of this ion is typically of the order of 10–3M, whereas that of cytosol is typically 10–7M, a 10,000-fold gradient. Such nonequilibrium ion distributions are even more remarkable in light of the fact that the plasma membrane is, to varying degrees, leaky to ions such as sodium, potassium and calcium. The plasma membrane is leaky to a variety of substances, but it exhibits an astonishing ability to discriminate or select one substance over another. This selectivity relates to not only ions but also for organic compounds such as glucose. Finally, the selectivity of biologic membranes can be altered drastically as a result of regulatory or signaling processes that occur within the cell.


82 Which of the following statement(s) is/are true concerning DNA?
a. DNA is contained only in the nucleus of the cell
b. DNA strands are encoded by the sequence of four bases—adenine, guanine, cytosine and uridine
c. The basic unit of information of DNA is the intron, a sequence of three bases
d. There are an infinite number of possible codons
Answer: a

The genetic blueprint of an organism is carried in the nucleus of every cell, encoded by the sequence of four bases—adenine, guanine, cytosine and thymine, which together make up two long chains bound together by hydrogen bonds to form a DNA double helix. A gene is a segment of DNA that is transcribed into a corresponding RNA molecule that either codes for a protein or forms a structural RNA molecule. Genes are commonly between 10,000 and 100,000 base pairs in length and include, in addition to the coding sequence, flanking regions and intervening sequences, termed introns. Introns are removed from the primary RNA transcript by a process called splicing. The basic unit of information is the codon, a sequence of three bases or triplet. The four nucleotide bases arranged as triplets lead to 64 possible codons. Sixty-one of these code for amino acids and three are termination signals called stop codons.

83 Which of the following statement(s) is/are true concerning cell membrane receptors?
a. The largest family of cell surface receptors are the G-protein-linked receptors
b. Activities of the G-protein involve binding and hydrolysis of ATP
c. The G protein receptor generates an intracellular messenger commonly through the use adenylate cyclase
d. Tyrosine kinase receptors are considered G-protein-linked receptors
Answer: a, c

All water-soluble regulatory molecules bind to the cell surface receptor proteins. Binding of the appropriate ligand evokes an intracellular signal which usually regulates enzyme activity, membrane transport, or in some cases gene expression. Most cell surface receptors belong to one of three functional classes—these are ion channel receptors, catalytic receptors, and G-protein-linked receptors. Ion channel receptors are multisubunit assemblies which, with each subunit, have a multiple membrane spanning segment. Together these subunits form an ion-selected pore that can be gated by a change in transmembrane electrical potential or binding of a ligand to one of the subunits. Catalytic receptors are membrane proteins that possess enzymatic activity. The best understood receptors of this class are the tyrosine kinases. The largest family of cell surface receptors are the G-protein-linked receptors. G-proteins are a family of proteins that bind and hydrolyze GTP. The final component of single transduction by G-protein-linked cell surface receptors is the effector that generates the intracellular messenger. The two best understood effectors are adenylate cyclase, which converts ATP to cAMP, and the polyphosphoinositide-specific phospholipase C.


84 Which of the following statement(s) is/are true concerning cellular ion channels?
a. Ion channels are transmembrane proteins that form pores that can conduct ions across the plasma membrane
b. Ion channels are formed by membrane-spanning peptides that are arranged so that polar moieties line a central core
c. Ion channel proteins undergo conformational changes between open states and closed states
d. Ion channels can be blocked
Answer: a, b, c, d

Ion channels are transmembrane proteins that form pores that can conduct ions across the plasma membrane. Ion channels are formed by membrane-spanning peptides that are arranged so that polar moieties line a central pore. These polar groups take the place of the water of hydration, which stabilizes an ion in an aqueous solution creating, in essence, a water-like environment into which the ion can partition and move in the presence of the appropriate driving force. Ion channels are permissive transport elements. Ions flow through a channel only through the presence of an appropriate driving force. Ion channels do not conduct all the time, rather the channel protein undergoes conformational changes between a conducting (open) state and nonconducting (closed) state. These conformational changes are collectively referred to as gating. The conduction process can also be blocked by ions or organic compounds that enter the channel, bind there, and occlude the pore.

85 Which of the following statement(s) is/are true concerning carrier proteins?
a. Carrier proteins are distinguished by three types of mechanisms: carrier-type, channel-type, and conduction-type
b. Conformational changes in the membrane protein occur between the conducting and the nonconducting states
c. A channel-type carrier protein has two states—closed and open
d. Carrier-type transport proteins are equally accessible from either side of the membrane
Answer: b, c

Most transport proteins appear to function as carriers, rather than channels. Important distinctions can be made between types of carrier proteins on the basis of transport kinetics. Two primary types can be distinctly identified based on carrier-type and channel-type mechanisms. The most important difference between the channel mechanism and the carrier mechanism is the role in the transport event played by conformational changes in the membrane protein. The channel is depicted as having two states, closed and open, so that it operates like a switch. In contrast, carrier transport is envisioned as requiring a cycle of conformational changes. The transport of one molecule of substrate requires one complete cycle of the protein. In a channel mechanism, binding sites within the open pore are equally accessible from either side of the membrane, whereas in a carrier mechanism, the binding site is available only one side of the membrane at any instant.

86 Which of the following statement(s) is/are true concerning translation of the mRNA message to protein synthesis?
a. An adaptor molecule, tRNA, recognizes specific nucleic acid bases and unites them with specific amino acids
b. Covalent attachment of tRNA to amino acids is energy dependent
c. The formation of a peptide bond between the growing peptide chain and the free amino acid occurs in the free cytoplasm
d. Complete protein synthesis takes hours
Answer: a, b

The synthesis of protein involves conversion from a four-letter nucleotide language to one of 20 chemically distinct amino acids. This process is referred to as translation. There is no mechanism for direct chemical recognition between specific nucleic acid bases and specific amino acids. Instead, an adaptor molecule, tRNA, is used. Each tRNA carries only one amino acid and must be recognized by a distinct enzyme which catalyzes the covalent attachment of the carboxyl end of the amino acid to the end of the tRNA in a process using ATP. Protein synthesis occurs by the formation of a peptide bond between the carboxyl terminal of the growing peptide chain and the free amino acid of deactivated amino acid tRNA. This event does not occur in free solution, but within ribosomes. Ribosomes are protein synthesizing machines that bring all of the necessary components together in the correct sequence and spacial orientation. Protein synthesis consumes a great deal of energy because four high-energy phosphate bonds must be split to make each peptide bond. Complete synthesis of a single protein takes 30 seconds to a few minutes, but multiple ribosomes can initiate translation and be moving down the mRNA molecules simultaneously, thus increasing the rate of protein synthesis.

87 Cell regulation can be thought of as the effector side of cell communication. Most commonly cell regulation occurs by means of extracellular chemical messengers. Which of the following statement(s) is/are true concerning these messengers?
a. Paracrine regulation involves a messenger which is produced and acts systemically
b. The extracellular signal or stimulus is received by a receptor on or in the target cell
c. Neurocrine regulation depends on a physical connection between the neuron and the target cell
d. Most hormones, local mediators, and neurotransmitters readily cross the cell plasma membrane
Answer: b, c

Depending on how the extracellular messenger arrives, cell regulation can be classified as paracrine, endocrine, or neurocrine. In paracrine regulation, a chemical messenger or mediator is produced and acts locally. In endocrine regulation, the extracellular messengers (hormones) are released into the blood and act on target cells anywhere on the body that has appropriate receptors. In neurocrine regulation, neurons secrete transmitters into highly localized regions, the synaptic cleft, so that the regulation depends on a physical connection between the neuron and the target cell as well as the presence of a specific receptor. In almost all cases of cell regulation, the extracellular signal or stimulus is restricted to being an informational molecule. This information is received by receptor on or in the target cell, which generally has an affinity for the signal molecule. Most hormones, local mediators, and neurotransmitters are water-soluble and cannot readily cross the plasma membrane.

88 Proteins that are destined to be secreted from the cells must pass through a series of organelles. These organelles include:
a. Endoplasmic reticulum
b. Golgi apparatus
c. Mitochondria
d. Lysosomes
Answer: a, b, d

Proteins targeted for the secretory pathway most commonly begin with translocation from the cytoplasm across the lipid bilayer into the lumen of the endoplasmic reticulum. It must then pass through a number of compartments including the Golgi apparatus where they are further processed and sorted and end up in a secretory vesicle or lysosome.

89 The best understood intracellular messenger is cyclic AMP (cAMP). Which of the following statement(s) concerning this intracellular messenger is/are correct?
a. Intracellular cyclic AMP is constantly degraded by a specific enzyme, cAMP phosphodiesterase
b. Most of the actions of cAMP are mediated by activation of protein kinase A
c. Intracellular levels of cAMP are relatively stable and change solely in response to activation of adenylate cyclase
d. cAMP is the only cyclic nucleotide active as an intracellular messenger
Answer: a, b

The prototypic intracellular messenger is cAMP. To function as a mediator, the concentration of cAMP must change rapidly. In resting cells, cAMP is continuously being degraded by a specific enzyme, cAMP phosphodiesterase. cAMP levels can increase 10-fold or more within seconds of receptor binding through activation of adenylate cyclase. cAMP acts as an allosteric regulator, and most, if not all, of its actions are mediated by activation of cAMP-dependent protein kinase A. cAMP is not the only cyclic nucleotide active as an intracellular messenger. Most animal cells also produce cGMP. Intracellular calcium ions also serve as second messengers in a large number of cells.

90 The activities of the cytoskeleton is dependent on which of the following types of filaments?
a. Microtubules
b. Intermediate filaments
c. Actin filaments
d. None of the above
Answer: a, b, c

The cytoskeleton is a collection of filamentous protein structures that allow cells to assume and maintain a variety of shapes, to produce directed movement of organelles within the cell, and to affect movement of the entire cell relative to other cells. These multiple activities depend upon three main types of filaments: actin filaments, intermediate filaments, and microtubules.

91 Intracellular organelles involved with protein synthesis include:
a. Mitochondria
b. Endoplasmic reticulum
c. Golgi complex
d. Lysosomes
Answer: b, c

Mitochondria are the major source of energy production in eukaryotic cells. The endoplasmic reticulum is the network of interconnected membranes forming closed vesicles, tubules, and saccules. The endoplasmic reticulum has a number of functions and is primarily involved in the synthesis of proteins and lipids. Adjacent to the rough endoplasmic reticulum and functionally involved in the sorting and package of secreted protein is the Golgi complex. Lysosomes are membrane-limited organelles containing acid hydrolytic enzymes that degrade polymers such as proteins, carbohydrates, and nucleic acids.


92 An important step in protein synthesis is transcription. Which of the following statement(s) is/are true concerning this process?
a. The first step in gene transcription involves separating the double helix of DNA by an enzyme known as DNA polymerase
b. The initial product of DNA transcription is called heterogeneous nuclear RNA which codes directly for proteins
c. After processing is complete, the mRNA is exported from the nucleus to the cytoplasm
d. Only one protein can be produced from an initial mRNA strand
Answer: c

Transcription of a gene begins at an initiation site associated with a specific DNA sequence, termed a promoter region. After binding to DNA, the RNA polymerase opens up a short region of the double helix to expose the nucleotides. Once the two strands of DNA are separated, the strand containing the promoter acts as a template to which ribonucleoside triphosphates base pair by hydrogen bonds. The initial products of transcription are known as heterogeneous nuclear RNA because of their large size variation. These primary transcripts are then processed to form mRNA. RNA splicing accounts for mature RNA being much shorter than nuclear RNA. Moreover, alternative splicing can lead to the production of different mRNA molecules and in some cases different proteins from the same gene. mRNA is exported from the nucleus only after processing is complete.

93 There are two properties of the cell necessary to maintain nonequilibrium cellular composition; the first is selectivity and the second is energy conversion. Which of the following statement(s) is/are true concerning energy converting transport?
a. The site of energy conversion and transport in the plasma membrane involves the phospholipid component
b. The Na+-K++-ATPase derives energy from hydrolysis of extracellular ATP
c. In some systems, energy inherent in the transmembrane ion gradient can be used to drive transport of a second species
d. Examples of species transported via secondary active transport include hydrogen ions, calcium, amino acids and glucose
Answer: c, d

The selectivity of the plasma membrane, although impressive, cannot account for the nonequilibrium composition of living cells. A cell can be maintained in a nonequilibrium state only by continual expenditure of energy. The maintenance of a steady-state, nonequilibrium cellular composition is possible because the plasma membrane is the site of energy converters, membrane proteins that function as biologic transport machines using energy derived from metabolic processes to perform transport work. The archetype for the biologic transport machine is the Na+-K+-ATPase, a membrane protein that hydrolyses cytosolic ATP and couples the resulting free energy to transport of Na+ and K+. A second equally important type of energy-converting transporter is one in which the energy inherent in a transmembrane ion gradient, usually that of Na+ can be used to drive the transport of a second species such as protons, calcium, amino acids, or glucose.

94 Which of the following statement(s) is/are correct concerning cell junctions?
a. The major occluding junction is the tight junction or zonula occludens
b. Tight junctions are usually located near the basal pole of the cell
c. Desmosomes are button-like points of attachment which serve to weld together adjacent cells
d. Gap junctions are a type of cell junction specialized for cell communication
Answer: a, c, d

Cell junctions are classified as occluding, anchoring, and communicating. The major occluding junction is the tight junction or zonula occludens which connects cells in epithelia and thereby allows epithelia to serve as selective permeability barriers. Tight junctions are normally located near the apical pool of the cell and form a belt that completely encircles the cell. Anchoring junctions connect the cytoskeleton of the cell to the extracellular matrix or neighboring cells. Morphologically these are adherens junctions or desmosomes. Desmosomes are button-like points of attachment with a prominent intracellular plaque that serves to weld together adjacent cells by serving as anchoring sites for intermediate filaments within the cell. The third functional type of cell junction is a gap junction which is specialized for communication. This junction mediates both electrical and chemical coupling.

95 Examples of ion channel blockers include:
a. Tetrodotoxin
b. Amiloride
c. Xylocaine
d. None of the above
Answer: a, b, c

Channel blockade is an important mechanism of action for toxins and some therapeutic agents. The deadly toxin of the puffer fish, tetrodotoxin, acts by blocking the Na+ channels that are responsible for the conduction of nerve impulse. The diuretic, amiloride, acts by blocking the Na+ channels that inhabit the apical membrane of the epithelial cells of the distal nephron. Local anesthetics such xylocaine also act by blocking ion channels.

96 Most hormone receptors are localized on the cell membrane and transduce hormone binding into altered levels of intracellular messengers. A limited number of intracellular receptors do exist. Which of the following statement(s) is/are true concerning intracellular receptors?
a. The messengers or hormones must by lipophilic
b. These intracellular receptors generally regulate protein synthesis
c. The intracellular receptors are located entirely in the nucleus of the cell
d. A heat-shock protein serves as an inhibitor protein blocking the DNA-binding domain of the steroid receptor
Answer: a, d

Although most hormone and other messenger receptors are extracellular, intracellular receptors have been identified. The hormone messengers involved for these receptors are primarily steroid and thyroid hormones and are lipophilic. By virtue of their hydrophobic nature, they are able to readily penetrate the lipid portion of the cell membrane. Receptors for these hormones exist intracellularly in the cytoplasm or in the nucleus and generally act as regulators of gene expression. These hydrophobic signaling molecules exist in plasma bound to protein, so that the concentration of this class of regulators does not fluctuate rapidly in plasma and their actions are generally slower in onset and more prolonged than those of water-soluble class. Some types of steroid receptors, particularly for glucocorticoids, are located in the cytosol in the inactive state. Once the ligand binds, the receptor undergoes a conformational change, termed activation. This allows cytoplasmic receptors to move into the nucleus and bind to DNA. Receptors already in the nucleus increase their affinity for DNA. In the case of glucocorticoid receptors and probably others of this class, the inactive receptor is associated with another protein, the heat-shock protein. They block the DNA-binding domain of the receptor. Activation involves the dissociation of the inhibitor protein.

97 Altering the amino acid profile in total parenteral nutrition solutions can be of benefit in certain conditions. Which of the following conditions are associated with a benefit by supplementation with the amino acid type listed?
a. Acute renal failure and essential amino acids
b. Hepatic failure and aromatic amino acids
c. Short gut syndrome and glutamine
d. Chronic renal failure and essential amino acids
Answer: a, c

In a number of conditions, altering the amino acid profile of the total parenteral nutrition solution can be of benefit. TPN with amino acids of high biologic value may decrease the mortality in patients with acute renal failure. These solutions, containing high quality amino acids, can improve nitrogen balance and diminish urea nitrogen. Provision of essential amino acids only allows the body to maximally utilize nitrogen for the synthesis of non-essential amino acids and thereby helps prevent rapid rises in blood urea nitrogen. There appears to be no advantages to using essential amino acids if the patient is already being dialyzed every other day and therefore a balanced standard amino acid solution is recommended. Because of liver damage and portasystemic shunting, patients with hepatic failure develop derangements in circulating levels of amino acids. The plasma aromatic/branch chain amino acid ratio is increased favoring the transport of aromatic amino acids across the blood brain barrier. These amino acids are precursors of false transmitters which contribute to lethargy and encephalopathy. Treatment of individuals with liver failure with solutions enriched in branch chain amino acids and deficient in aromatic amino acids results in improved tolerance to administration of protein and clinical improvement in encephalopathic states. Glutamine-enriched TPN partially attenuates villous atrophy and may be useful in treatment of short gut syndrome.

98 Under certain circumstances, the gut may become a source of sepsis and serve as the motor of systemic inflammatory response syndrome. Microbial translocation is the process by which microorganisms migrate across the mucosal barrier to invade the host. Which of the following mechanisms can promote bacterial translocation?
a. An increased number of gut bacteria
b. Altered intestinal mucosal permeability
c. Decreased host defense mechanisms
d. Lack of enteral feeding

Answer: a, b, c, d

99 Translocation is promoted in three general ways: 1) altered permeability of the intestinal mucosa as caused by shock, sepsis, distant injury, or cell toxins; 2) decreased host defense (secondary to glucocorticoid administration, immunosuppression, or protein depletion; and 3) an increased number of bacteria within the intestine. Because many factors that facilitate bacteria translocation occur simultaneously in surgical patients, these effects may be either additive or cumulative. In addition, many patients in Surgical Intensive Care Units do not generally receive enteral feedings and therefore current parenteral therapy results in gut atrophy which further promotes translocation.
Which of the following statement(s) is/are true concerning nutritional support of the injured patient?
a. The goal of nutritional support is maintenance of body cell mass and limitation of weight loss to less than 25% of preinjury weight
b. Under-nutrition may compromise the patient’s available defense mechanisms
c. Nutritional support is an immediate priority for the trauma patient
d. Fifty percent of non-nitrogen caloric requirements should be provided in the form of fat
Answer: b

Metabolic response to injury results in increased energy expenditure. If energy intake is less than expenditure, oxidation of body fat stores and erosion of lean body mass will occur with resultant loss of weight. When weight loss exceeds 10–15% of body weight, the complications of malnutrition interact with disease processes, with increased morbidity and mortality rates. The goal of nutritional support is maintenance of body cell mass and limitation of weight loss to less than 10% preinjury. The major impact of nutritional support in the trauma patient is to aid host defense. Under-nutrition may compromise the available host defense mechanism and may thus increase the likelihood of invasive sepsis, multiple organ system failure, and death. Resuscitation, oxygenation and arrest of hemorrhage are immediate priorities for survival. Nutritional support is an essential part of the metabolic care of the critically ill patient and should be instituted after resuscitation before significant weight loss occurs. The nutritional requirements of a trauma patient can be determined by determining basal metabolic rate with appropriate increases based on extent of injury and hospital activity. After initial determination of nitrogen requirements, caloric requirements should be distributed at a ratio of 70% as glucose and 30% as fat.

100 Which of the following statement(s) is/are true concerning body fuel reserves?
a. The largest fuel reserve in the body is skeletal muscle
b. Fat provides about 9 calories/gram
c. Free glucose and glycogen stores are a trivial fuel reserve
d. Body protein is a valuable storage form of energy
Answer: b, c

The body contains fuel reserves which it can mobilize and utilize during times of starvation or stress. By far the greatest energy component is fat, which is calorically dense since it provides about 9 calories/gram. Body protein comprises the next largest mass of utilizable energy, but amino acids yield only about 4 kcal/gram. Unlike fat reserves, body protein is not a storage form of energy but rather serves as a structural functional component of the body; loss of body protein, if severe, is associated with functional consequences. Glycogen stored in muscle and liver and free glucose have a trivial caloric value of less than 1000 kcal for a 70 kg male.
101 Which of the following statement(s) is/are true concerning the indications and administration of nutritional support to cancer patients?
a. Preoperative nutritional support should be provided to all patients with cancer
b. To be effective, preoperative nutrition must be given for at least two weeks preoperatively
c. Parenteral nutrition is the preferred route of feeding for all cancer patients
d. Standard total parenteral nutrition solutions maintain integrity of the small bowel
e. None of the above
Answer: e

The role of nutritional support in the cancer patient remains an important component of overall therapy. Preoperative nutritional support should be given only to those patients who do not require an emergency operation and who have severe weight loss (> 15% of pre-illness body weight) and a serum albumen < 2.9 mg%. Preoperative nutrition (enteral or parenteral) should not be given for longer than 7 to 10 days. Enteral nutrition is always the preferred route of feeding cancer patients if the GI tract is functional. There are several benefits of using the bowel lumen for nutrient delivery. The trophic effects of enteral feeding on small bowel mucosa have been well described. The integrity of the mucosal lining is maintained and it may provide an effective barrier to intraluminal enteric organisms which might otherwise translocate into the systemic circulation. Atrophic changes may be seen in the intestinal epithelium after several days of bowel rest; this atrophy is not reversed by currently available total parenteral nutrition solutions. 102 Which of the following hormones can be expected to be released as part of the stress response? a. Antidiuretic hormone (ADH) b. Aldosterone c. Insulin d. Epinephrine nswer: a, b, d Several important responses occur in response to stress. The body immediately attempts to compensate for a reduction in circulating blood volume in order to maintain adequate organ perfusion. Afferent nerve signals are also initiated which stimulate the release of both antidiuretic hormone (ADH) and aldosterone. The pain and fear associated with the stress response lead to excessive production to catecholamines which also increase metabolic rate, stimulate lipolysis, hepatic glycolysis, and gluconeogenesis. Glucagon, which has a potent glycogenolytic and gluconeogenic effect in the liver, is also released. This hormone has the exact opposite effect of insulin, which promotes glucose storage and uptake by the cells. 103 Cytokines which play an important role in the metabolic response to injury include: a. Tumor necrosis factor—a (TNF) b. Interleukin-1 (IL-1) c. Interleukin-6 (IL-6) d. Interferon-g Answer: a, b, c, d TNF or cachetin is considered the primary mediator of the systemic effects of endotoxin, producing anorexia, fever, tachypnea, and tachycardia at low doses and hypotension, organ failure, and death at higher doses. TNF is produced primarily by macrophages, but lymphocytes, Kupffer cells, and a number of other cell types have been identified as sources of TNF. IL-1, like TNF, has a variety of pro-inflammatory activities. IL-6 is now recognized as a primary mediator of altered hepatic protein synthesis known as acute-phase protein synthetic response. Glucocorticoid hormones augment the cytokine effects on acute phase protein synthesis. Interferons are a family of proteins which are readily identified for their ability to inhibit viral replication in infected sells. IFN-g has the ability to upregulate the number of TNF receptors on various cell types. 104 A 16-year-old boy suffers a mid-gut volvulus with massive loss of small intestine. Which of the following statement(s) is/are true concerning his nutritional requirements and management? a. If at least 18 inches of residual small intestine survives, the patient may tolerate some form of enteral nutrition b. A nutritional regimen consisting of supplemental glutamine, growth hormone, and a modified high carbohydrate, low fat diet may be beneficial in this patient c. The regimen described above may decrease the cost of care d. TPN needs will increase after discontinuation of growth hormone Answer: a, b, c Prior to the availability of TPN, most patients developing short bowel syndrome from either surgery or catastrophic event died. In selected patients, however, with residual small intestine (at least 18 inches), post-resectional hyperplasia may develop with time such that they can tolerate enteral feeds. Recent studies have demonstrated the requirement for TPN could be decreased or even eliminated in patients with short-gut syndrome by providing a nutritional regimen consisting of supplemental glutamine, growth hormone, and a modified high carbohydrate, low fat diet. There was a marked improvement in absorption of nutrients with this combination of therapy and a decrease in stool output. In addition, TPN requirements were reduced by 50% as were costs associated with the care of these individuals. Discontinuation of the growth hormone did not increase TPN needs in patients once they had undergone successful gut rehabilitation. 105 A number of changes in trace mineral metabolism are noted during sepsis. Which of the following change(s) may be observed in a septic or trauma patient? a. Plasma iron levels are noted to decrease b. Plasma copper levels are noted to decrease c. Plasma serum zinc levels may decrease d. Administration of iron is appropriate Answer: a, c Changes in the balance of magnesium, inorganic phosphate, zinc, and potassium generally follow alterations in nitrogen balance. Although the iron-binding capacity of transferrin is usually unchanged in early infection, iron disappears from the plasma, especially during severe pyogenic infection; similar alterations are observed in serum zinc levels. The administration of iron to the infected host, especially early into the disease, is contraindicated, however, because increased serum iron concentrations may impair resistance. Unlike iron and zinc, copper levels generally rise, and the increased plasma concentrations can be ascribed almost entirely to the levels of the ceruloplasmin produced by the liver. 106 A 17-year-old patient involved in an automobile accident is paralyzed with multiple peripheral extremity injuries. Nutritional support is instituted with a transnasal feeding catheter. Which of the following statement(s) is/are true concerning the patient’s management? a. Feeding into the stomach results in stimulation of the biliary/pancreatic axis which is probably trophic for small bowel b. Gastric secretions will dilute the feedings increasing the risk of diarrhea c. The major risk in this patient is tracheobronchial aspiration d. Placement of the feeding catheter through the pylorus into the first portion of the duodenum reduces the risk of regurgitation and aspiration Answer: a, c, d The use of transnasal feeding catheters for intragastric feeding or for duodenal intubation are popular adjuncts for providing nutritional support by the enteral route. The stomach is easily accessed by passage of a soft flexible feeding tube. Intragastric feeding provides several advantages for the patient. The stomach has the capacity and reservoir for bolus feedings. Feeding into the stomach results in stimulation of the biliary/pancreatic axis which is probably trophic for the small bowel. Gastric secretions will have a dilutional effect on the osmolarity of the feedings, reducing the risk of diarrhea. The major risk of intragastric feeding is the regurgitation of gastric contents resulting in aspiration into the tracheobronchial tree. This risk is highest in patients who have an altered sensorium or who are paralyzed. The placement of the feeding tube through the pylorus into the fourth portion of the duodenum reduces the risk of regurgitation and aspiration of feeding formulas. 107 Although TPN has major beneficial effects to the patient and specific organ systems, TPN has a downside which is related to intestinal disuse. Which of the following statement(s) is/are true concerning the effects of TPN on the GI tract? a. Patients receiving TPN have an accentuated systemic response to endotoxin challenge compared to enterally fed volunteers b. TPN can result in disruption of intestinal microflora c. In experimental models, bacterial translocation from the gut is increased d. Effects of TPN on the gut may lead to multiple organ failure nswer: a, b, c, d A number of studies have examined the effects of TPN on intestinal function and immunity. Although most of these studies have been done in animal models, TPN has consistently been shown to have some detrimental effects. In rats, TPN results in significant disruption of the intestinal microflora and bacterial translocation of the gut to the mesenteric lymph nodes. In addition, when stresses such as a burn injury, chemotherapy, or radiation are introduced into these models, animals on TPN have a much higher mortality. The body of literature suggests that TPN under certain circumstances may predispose patients to an increase in gut-derived infectious complications. In a study in human volunteers, individuals receiving TPN had an accentuated systemic response to endotoxin challenge compared to enterally fed volunteers. This study is consistent with impairment of gut barrier function during parenteral feedings which may promote the release of bacteria and/or cytokines leading to pronounced systemic responses and possibly multiple organ failure. 108 Total body mass is composed of an aqueous component and a nonaqueous component. The nonaqueous component is made up of which of the following? a. Liver b. Tendons c. Skeletal muscle d. Extracellular fluid e. Adipose tissue Answer: b, e The nonaqueous portion of total body mass is made up of bones, tendons, and mineral mass as well as adipose tissue. The aqueous component contains the body cell mass which is made up of skeletal muscle, intraabdominal and intrathoracic organs, skin, and circulating blood cells. Also contributing to the aqueous portion is the interstitial fluid and intravascular volume. 109 Fatty acids are a major energy source for the body. Which of the following statement(s) is/are true concerning the use of fatty acids as an energy source? a. Fatty acids are stored in adipocytes as triglycerides b. Hormone-sensitive lipase is present only in adipose tissue c. Fatty acids are released into the circulation traveling freely in plasma d. Approximately 25% of total nonprotein caloric needs supplied via total parenteral nutrition should be in the form of fat Answer: a, b, d In most tissues, fatty acids are readily oxidized for energy. They are especially important energy sources for the heart, liver and skeletal muscle. In adipose tissue, fatty acids may be re-esterified with glycerol and stored as triglycerides in adipocytes. Stored fat is mobilized during starvation and stress. Hormone-sensitive lipase, present only in adipose tissue, catalyzes the breakdown of stored triglycerides into glycerol and fatty acids. The fatty acids that are produced are released in the circulation. The major lipids in plasma do not circulate in a free form, thus free fatty acids must be bound to albumin. During stress, the activity of hormone-sensitive lipase is increased which leads to mobilization of fat stores. However, fat remains an important fuel source for critically ill patients and as a rule the amount of fat administered to patients receiving total parenteral nutrition should comprise about 5–30% of total nonprotein caloric needs. 110 Which of the following metabolic effects may be observed in patients with sepsis? a. Increased gluconeogenesis b. Accelerated proteolysis c. Increased lipolysis d. Impaired gut metabolism of glutamine Answer: a, b, c, d A number of metabolic responses to sepsis have been defined. Glucose production is increased in infected patients which appears to be additive to the augmented gluconeogenesis that occurs following injury. Accelerated proteolysis, increased nitrogen excretion and prolonged negative nitrogen balance also occur following infection with a response pattern similar to that described with injury. Severe infection is often associated with a hypercatabolic state that initiates marked changes in interorgan glutamine metabolism. This process results in accelerated muscle proteolysis and net skeletal muscle glutamine release. The bulk of glutamine is taken up by the liver at the expense of the gut. It appears that sepsis can impair gut metabolism of glutamine. Fat is a major fuel oxidized in infected patients, and increased metabolism of lipids from peripheral fat stores is especially prominent during a period of inadequate nutritional support. 111 Which of the following statement(s) is/are true concerning protein/amino acid metabolism in man? a. The major source of amino acids is breakdown of circulating proteins b. The recommended daily allowance for protein may triple in critically ill patients c. Urinary nitrogen losses will approach 0 in the face of protein starvation d. Negative nitrogen balance refers to a decrease in nitrogen taken into the body versus the amount of nitrogen lost Answer: b, d About 15% of the total body weight is made up of proteins, about half of which are intracellular and half extracellular. In man and other animals, dietary protein is the source of most amino acids. Intestinal absorption is the only physiological pathway by which the body obtains exogenous amino acids. Digestion of ingested protein provides free amino acids that are absorbed by the small intestine and transported to the liver where they can be incorporated into new proteins or other biosynthetic products. Excess amino acids are degraded and their carbon skeleton is oxidized to produce energy or it is incorporated into glycogen or into free fatty acids. In addition to the metabolism of dietary amino acids, the existing proteins in the cell are continuously recycled, such that total protein turnover in the body is about 300 g/day. Vertebrates cannot reutilize nitrogen with 100% efficiency; therefore, obligatory nitrogen losses occur, mainly in the urine. Urinary nitrogen losses will diminish when individuals are fed a protein-free diet, but will never become 0 because of the body’s inability to completely reutilize nitrogen. In stressed patients, this ability to adapt to starvation is compromised such that proteolysis of body proteins continues at a substantial rate. This increases the amount of obligatory nitrogen losses which are accentuated by the catabolic disease states. This results in a negative nitrogen balance in which the amount of nitrogen taken in by the patient is exceeded by the amount of nitrogen lost in the urine, stool, skin, wounds, and fistula drainage. 112 Which of the following statement(s) concerning intravenous nutritional support is/are true? a. Concentrations of glucose no higher than 5% should be used to avoid peripheral vein sclerosis b. A major disadvantage of the peripheral technique is limited caloric delivery c. If total parenteral nutrition is required, access to the superior vena cava via the external jugular vein is the most suitable site d. Venous thrombosis is an uncommon complication for long-term central vein catheterization Answer: b Although peripheral access can be used for intravenous nutrition, the major disadvantage of this technique is limited caloric delivery to meet catabolic demands within tolerated fluid limits. Infusion of glucose (up to 10%), amino acid solutions, and fat emulsions can be administered peripherally but these solutions must be nearly isotonic to avoid peripheral vein sclerosis. The preferred method of access for total parenteral nutrition is into the superior vena cava by cutaneous cannulation of the subclavian vein. Alternative sites include the internal and external jugular vein but the catheter exiting from the neck region makes it more difficult to secure and maintain a sterile dressing. Complications from long-term central venous catheterization include venous thrombosis and venous catheter-related infection. Thrombosis of central vessels is a complication which is often overlooked. The clinical suspicion of subclavian vein thrombosis is only about 3%, whereas studies that use phlebography or radionucleotide venography indicate the incidence is as high as 35%. 113 Sepsis causes a marked metabolic response. Which of the following statement(s) is/are true concerning the metabolic response to sepsis? a. Oxygen consumption is increased in the face of infection b. In a patient with a maximal metabolic rate secondary to trauma, the presence of infection will increase the rate further c. Metabolic rate increases at a rate of approximately 10% for each increase of 1°C in central temperature d. The extent of increase in oxygen consumption relates to the severity of the infection Answer: a, c, d Oxygen consumption is usually elevated in the infected patient. The extent of this increase is related to the severity of the infection, with peak elevations reaching 50% to 60% above normal. If the patient’s metabolic rate is already elevated to a maximal extent because of severe injury, no further increase will be observed. In patients with only a slightly accelerated rate of oxygen consumption, the presence of infection will cause a rise in metabolic rate added to the preexisting state. A portion of the increase in metabolism may be ascribed to increase in reaction rate associated with fever. Calculations suggest that the metabolic rate increases 10% to 13% for each elevation of 1°C in central temperature. 114 Interleukin-6 is recognized as the cytokine primarily responsible for the alteration in hepatic protein synthesis recognized as the acute phase response. Which of the following statement(s) is/are true concerning acute phase protein response to surgical stress? a. Glucocorticoid hormones inhibit this response b. Proteins such as albumin and transferrin which serve in serum transport are generally increased in this response c. Examples of acute phase proteins include fibrinogen and C-reactive protein d. In general, the physiologic role of acute phase proteins are to reduce the systemic effects of tissue damage Answer: c, d IL-6 is now recognized at the cytokine primarily responsible for the alteration in hepatic synthesis recognized as the acute phase response. Glucocorticoid hormones augment this response. The primary metabolic component of the acute phase response is a qualitative alteration in hepatic protein synthesis with resulting alteration in plasma protein composition. Characteristically, proteins which act as serum transport in binding molecules, (albumin, transferrin) are reduced in quantity and acute phase proteins (fibrinogen, C-reactive proteins) are increased. Acute phase proteins are elaborated for the purpose of reducing the systemic effects of tissue damage. Many act as anti-proteases, opsonins, or coagulation and wound healing factors that generally inhibit the tissue destruction that is associated with the local initiation of inflammation. 115 A 59-year-old trauma patient has suffered multiple septic complications including severe pneumonia, intraabdominal abscess, and major wound infection. He has now developed signs of multisystem organ failure. Which of the following statement(s) is/are true concerning necessary changes to be made in his nutritional management? a. Carbohydrate load should be reduced in the face of respiratory failure b. In patients with renal failure, protein intake should be diminished c. During hemodialysis protein intake should be limited to the same extent d. In patients with hepatic failure, carbohydrate load should be increased Answer: a, b The most severe complication of sepsis is multiple system organ dysfunction syndrome, which may result in death. The development of organ failure requires changes in the nutritional requirements and creates special feeding problems. A problem associated with systemic infection is oxygenation and elimination of carbon dioxide. Most of the enteral and parenteral formulas used to provide nutritional support for critically ill patients contain large amounts of carbohydrate, which generate large amounts of carbon dioxide following oxygenation. Such a large CO2 load may worsen pulmonary function or may delay weaning from the respirator. If this factor becomes a problem, the carbohydrate load should be reduced to 50% of metabolic requirements and fat emulsion administered to provide additional calories. When renal failure becomes progressive, the use of hemodialysis minimizes the effect of uremia superimposed on the metabolism of sepsis. Metabolic studies in patients with acute and chronic renal failure have limited the intake of nonessential amino acids, in an attempt to lower urea production. Proteins of high biologic value, but in much smaller quantities than usually given, are administered along with adequate calories, usually in the form of glucose. When enteral feedings are not feasible, a central venous infusion of an essential amino acid solution and hypertonic dextrose provides calories and a small quantity of nitrogen to reduce protein catabolism while simultaneously controlling the rise in BUN. During dialysis, protein intake is liberalized, but the BUN should still be maintained below 100 mg/dl. Hepatic dysfunction is a common manifestation of septicemia. The carbohydrate load is usually reduced to consist of no more than 5% of metabolic requirements, and the additional calories should be provided as fat emulsion. If encephalopathy develops, protein load should also be reduced0. 116 Which of the following statement(s) is/are true concerning the role of glutamine in total parenteral nutrition? a. Glutamine is an essential amino acid b. Glutamine appears to be of primary benefit in critical illness c. Glutamine is included in most standard TPN solutions d. Glutamine is the primary energy source for intestinal mucosal cells of the small bowel and colon Answer: b Glutamine is the most studied gut-specific nutrient. Glutamine has been classified as a nonessential or nutritionally dispensable amino acid since glutamine can be synthesized in adequate quantities from other amino acids and precursors. Glutamine is not included in most nutritional formulas and has been eliminated from TPN solutions because of its relative instability and short half life compared to other amino acids. With few exceptions, glutamine is present in oral enteral diets but only at relatively low levels characteristic of the concentration in most animal and plant stores (about 7% of total amino acids). Several recent studies, however, have demonstrated that glutamine may be an essential amino acid during critical illness, particularly as it relates to supporting the metabolic requirements of the intestinal mucosa. These studies demonstrate that dietary glutamine is not required during states of health but appears to be beneficial when glutamine depletion is severe and/or when intestinal mucosa is damaged by insults such as chemotherapy or radiation therapy. The addition of glutamine to enteral diet reduces the incidence of gut translocation but these improvements are dependent upon the amount of supplemental glutamine and the type of insult studied. Glutamine-enriched TPN partially attenuates villous atrophy that develops during parenteral nutrition. The use of intravenous glutamine in patients appears to be safe and effective in its ability to maintain muscle glutamine stores and improve nitrogen balance. In contrast to glutamine, short chain fatty acids are primary energy source for colonocytes. 117 Which of the following are determinants of the host response to surgical stress? a. Gender b. Age c. Nutritional status d. Body composition Answer: a, b, c, d The pattern of physiologic changes elicited in response to surgical stress results from the specific interaction of an individual patient with a stressful stimulus. Several factors specific to the patient may determine the nature of the host response to stress. Body composition is a major determinant of the metabolic responses observed during surgical illness. Post-traumatic nitrogen excretion is directly related to the size of the body protein mass. A strong relationship between protein depletion and postoperative complications has been demonstrated in nonseptic, nonimmunocompromised patients undergoing elective major gastrointestinal surgery. Protein-depleted patients have significantly lower preoperative respiratory muscle strength and vital capacity, increased incidence of postoperative pneumonia, and longer postoperative hospital stay. Impaired wound healing and respiratory, hepatic, and muscle function in protein-depleted patients awaiting surgery has also been reported. Many of the changes in the metabolic responses to surgical illnesses that occur with aging can be attributed to alterations in body composition and to long-standing patterns of physical activity. Fat mass tends to increase with age and muscle mass tends to decrease. Loss of strength that accompanies immobility, starvation and acute surgical illness may have marked functional consequences. Furthermore, the prevalence of cardiovascular and pulmonary diseases increase with age. Thus, the delivery of oxygen to tissues may be impaired in the elderly. Finally, observed differences in metabolic responses of men and women generally reflect differences in body composition. Lean body mass is lower in women than in men; and this difference is thought to account for the net loss of nitrogen after major elective abdominal surgery generally being lower in women than in men. 118 In contrast to a patient undergoing an elective operation, which of the following statement(s) is/are true concerning a patient who has suffered a multiple trauma? a. Basal metabolic rates are similar b. The patient is highly sensitive to insulin c. Utilization of the amino acids, glutamine and alanine, is similar to their composition in skeletal muscle d. Fat and protein stores are rapidly depleted Answer: b, d The degree of hypermetabolism is generally related to the severity of injury. Patients with long-bone fractures have a 15–25% increase in metabolic rate, whereas metabolic rates in patients with multiple injuries increases by 50%. These metabolic rates in trauma patients are contrasted with those in postoperative patients, who rarely increase their BMR by more than 10–15% following operation. Studies have shown that uninjured volunteers are able to dispose of exogenous glucose load much more readily than injured patients. Other studies have demonstrated a failure to suppress hepatic glucose production in trauma patients during glucose loading or insulin infusion. Thus, profound insulin resistance occurs in injured patients. Skeletal muscle is the major source of nitrogen that is lost in the urine following extensive injury. Although it is recognized that amino acids are released by muscle in increased quantities following injuries, it has only been recently appreciated that the composition of amino acid reflux does not reflect the composition of muscle protein. The release is skewed towards glutamine and alanine, each of which comprise about one-third of the total amino acids released by skeletal muscle. To support hypermetabolism, stored triglyceride is mobilized at an accelerated rate. Although mobilization and use of free fatty acids are accelerated in injured subjects, if unfed, severely injured patients rapidly deplete their fat and protein stores. 119 A 47-year-old patient undergoing a complicated laparotomy for bowel obstruction develops a postoperative enterocutaneous fistula. Which of the following statement(s) is/are true concerning parenteral nutritional support in the postoperative period? a. Oral intake can result in severe dehydration, electrolyte abnormalities, and perifistula skin injury b. Total parenteral nutrition increases the spontaneous closure rate of intestinal fistula c. Total parenteral nutrition decreases mortality rate in patients with intestinal fistulas d. The use of TPN better prepares the patient for surgery if surgical intervention proves necessary Answer: a, b, d Patients with gastrointestinal-cutaneous fistulas represent the classical indication for TPN. In such patients, oral intake of food almost invariably results in increased fistula output with associated metabolic disturbances, dehydration, skin breakdown, and death. Several comprehensive reviews have concluded that TPN clearly impacts on the treatment course of the disease in patients with GI fistulas. The following conclusions can be drawn from studies evaluating the use of TPN in patients with enterocutaneous fistula. First, TPN increases spontaneous closure rate of enterocutaneous fistulas but does not markedly decrease the mortality rate in patients with fistulas. Second, if spontaneous closure of the fistula does not occur, patients are better prepared for operative intervention because of the nutritional support they have received. Finally, certain fistulas are associated with a lower rate of spontaneous closure than others and should be treated more aggressively surgically after a defined period of nutritional support (unless closure occurs). 120 Appropriate guidelines for the use of TPN in cancer patients include: a. Long-term TPN in patients with rapid progressive tumor growth unresponsive to other therapy b. Mildly malnourished patients undergoing surgery for a curable cancer c. Preoperatively administered TPN prior to surgery or other therapy in patients with severe malnutrition d. Patients in whom treatment toxicity precludes the use of enteral nutrition Answer: c, d As a general rule, the most important factor to consider when making decisions about the use of TPN in patients with cancer is the response of the tumor to antineoplastic therapy. Appropriate guidelines would include the following: Short-term TPN is indicated in severely malnourished patients or in those in whom gastrointestinal or other toxicities preclude adequate enteral intake for seven days or a longer period. TPN is not indicated in well nourished or mildly malnourished patients undergoing therapy or surgery who would be expected to be able to resume adequate nutrition in approximately seven days. Long-term TPN is indicated in patients in whom treatment associated toxicities preclude the use of enteral nutrition and represent the primary impediment to the restoration of performance status. These patients should be expected to be responding to anti-tumor therapy. Long-term TPN is not indicated with rapidly progressive tumor growth which is unresponsive to such therapy. 121 Which of the following statements(s) is/are true concerning human energy requirement? a. In normal subjects, less than 5% of basal energy requirement is spent on cardiac output and the work of breathing b. Mechanical ventilation can decrease the energy expenditure for normal respiration c. For a 70 kg male, average resting energy consumption is almost 1500 kcal/day d. Similar increases in energy expenditures are associated with elective surgery and trauma or thermal injury Answer: a, c Basal energy requirements are measured with the subject at rest when no external work is being done; the energy is used mainly for transport and synthetic work within cells. A surprisingly small percentage (< 5%) of this energy is spent on cardiac output and the work of breathing in normal subjects. In contrast, the work of breathing in individuals with chronic obstructive lung disease or in patients on a ventilator may account for 15–20% of caloric expenditure. The average resting post-absorptive 70 kg male consumes about 1500 kcal/day. Energy needs increase as severity of illness increases. The expenditure of kcal is only minimally increased after elective surgery. The largest increase in energy expenditure occurs in patients with severe multiple trauma or major thermal injury. The average-sized adult who sustains a major burn rarely may require more than 3500 kcal/day for maintenance. 122 Which of the following complications of TPN are appropriately managed with the listed treatment? a. Air embolism—place patient in reverse Trendelenburg and the left lateral decubitus position and aspirate venous air b. Hyperchloremic metabolic acidosis—give sodium and potassium as acetate salts c. Carbon dioxide retention—decrease glucose calories and replace with fat d. Line sepsis—intravenous antibiotics Answer: b, c A number of complications of TPN can occur which can be divided into three types: mechanical, metabolic, and infectious. 123 A 55-year-old male undergoes a total abdominal colectomy. Which of the following statement(s) is/are true concerning the hormonal response to the surgical procedure? a. Adrenocorticotropic hormone (ACTH) is secreted from the anterior pituitary gland b. ACTH stimulation results in elevation of serum cortisol levels for up to a week after the operation c. An increased secretion of aldosterone and ADH may contribute to postoperative fluid retention d. An increase in serum insulin and a fall in glucagon accelerate hepatic glucose production and maintain gluconeogenesis Answer: a, c One of the earliest consequence of a surgical procedure is the rise in levels of circulating cortisol that occur in response to a sudden outpouring of ACTH from the anterior pituitary. The rise in ACTH stimulates the adrenal cortex to elaborate cortisol which remains elevated for 24–48 hours after operation. The neuroendocrine responses to operation also modify the various mechanisms that regulate salt and water excretion. Alterations in serum osmolarity and tonicity of body fluids secondary to anesthesia and operative stress, stimulate the secretion of aldosterone and ADH. Thus, the ability to excrete a water load after elective surgical procedures is restricted, and weight gain secondary to salt and water retention is usual following an operation. Alterations occur in response to the endocrine pancreas following elective operation. Insulin elaboration is diminished and glucagon concentrations rise. The rise in glucagon and the corresponding fall in insulin are important signals to accelerate hepatic glucose production, and, with other hormones (epinephrine and glucocorticoids), gluconeogenesis is maintained. 124 A number of prospective clinical trials have addressed the role of total parenteral nutrition in the cancer patient. The results have been somewhat conflicting. Which of the following statement(s) have been proven correct by prospective trials? a. Preoperative TPN is beneficial in surgical patients with severe preoperative nutrition b. Postoperative TPN is of value following pancreatic resection c. Routine use of perioperative (including prior to the procedure) TPN is of benefit in patients undergoing hepatectomy for hepatoma d. TPN is of no benefit in patients undergoing bone marrow transplant Answer: a, c Numerous clinical trials have failed to yield a consensus with regard to the efficacy of TPN in cancer patients. In 1991, a multicenter VA cooperative trial demonstrated that preoperative TPN is of benefit in surgical patients (many of whom had cancer) with severe preoperative malnutrition. Another study examined the use of routine postoperative TPN following major pancreatic resection. Patients randomized to receive TPN starting on postoperative day 1 were noted to have an increased incidence of intra-abdominal abscesses as well as a tendency towards increased incidence in peritonitis and bowel obstruction. These investigators concluded that routine use of postoperative TPN was not indicated and may, in fact, be harmful following pancreatic resection. In another study, however, perioperative (starting 7 days prior to the planned procedure) TPN for patients undergoing hepatectomy for hepatocellular carcinoma demonstrated that this regimen statistically reduced infectious complications compared to patients who did not receive TPN. This was one of the few studies that demonstrated that routine TPN (without the requirement of severe preoperative malnutrition) was of benefit. The use of TPN in patients receiving bone marrow transplantation has also been shown to be a valuable component of overall care. 125 Which of the following statements concerning perioperative nutrition is true concerning the above-described patient? a. Since the patient’s weight had been stable with no preoperative nutritional deficit, 5% dextrose intravenous solutions are adequate for the initial postoperative source of nutrition b. Preoperative immunologic status should be determined including total peripheral lymphocyte count and delayed hypersensitivity reaction to determine skin-test response to common antigens c. Routine postoperative fluid administration with intravenous 5% glucose solutions can provide the calories to meet basal energy requirements d. A jejunal feeding catheter should be placed at the time of surgery for postoperative enteral feeding Answer: a Most patients undergoing elective operations are adequately nourished. Unless the patient has suffered significant preoperative malnutrition, characterized by weight loss greater than 10–15%, or has major intraoperative or postoperative complications, solutions containing 5% dextrose may be administered for five to seven days before initiation of enteral nutrition, with no detrimental effect on outcome. The usual postoperative surgical patient is given intravenous glucose at 125 cc/hour receives about 500 kcal/day, far less than the actual number of kcal needed to meet energy requirements. The increased cost of feedings and potential complications associated with intravenous nutrition cannot be justified. Although the use of jejunal feedings in the postoperative period may be useful in some patients, especially those undergoing extensive gastrointestinal surgery, this technique would not appear indicated in the patient described above. 126 The neurohormonal arm of the stress response is well defined. Less is known about the inflammatory arm mediated primarily by cytokines. Which of the following statement(s) is/are true concerning this arm of the surgical stress response? a. Cytokines primarily work locally via direct cell-to-cell communication b. Cytokines are never detectable in the systemic bloodstream c. Cytokines are produced only by immune cells attracted to the site of injury d. Cytokine release may stimulate the release of other cytokines leading to an important cascade of events Answer: a, d Cytokines, which are produced at the site of injury by endothelial cells and by diverse immune cells throughout the body, also occupy a pivotal position in the stress response. Cytokines differ from classic endocrine hormones in that they are produced by a variety of cell types and in that they have the capacity to exert their tissue effects locally via direct cell-to-cell communications in a paracrine and/or autocrine fashion. Cytokines can stimulate the production of other cytokines, leading to important cascades which both amplify and diversify the effects of the proximal cytokine. Occasionally, when in excess, cytokines act as hormones and “spill over” into the systemic circulation and become detectable in the bloodstream. 127 Which of the following tissues contain significant collagen useful for placing sutures to allow the prolonged tension necessary to maintain tissue approximation? a. Dermis b. Intestinal submucosa c. Muscular fascia d. Blood vessel wall Answer: a, b, c, d It takes at least three weeks for collagen to undergo sufficient remodeling and cross linking to attain moderate strength. Since most skin sutures are removed at one to two weeks, the wound has only a small fraction of its eventual strength and may therefore disrupt with even modest stress. Therefore, deep sutures are placed in collagen containing structures to maintain the prolonged tension necessary. Dermis, intestinal submucosa, muscular fascia, tendon, ligament, Scarpa’s fascia, and blood vessel wall represent a partial list of tissues with high collagen content. 128 Products of platelet degranulation include: a. Tumor necrosis factor b. Interleukin-1 c. Transforming growth factor b d. Platelet-derived growth factor Answer: c, d The initial response to injury and disruption of a blood vessel is bleeding. The hemostatic response to this is clot formation to stop hemorrhage. Platelet plug formation initiates the hemostatic process along with clotting factors activated by collagen and the basement membrane proteins exposed by the injury. Platelets then degranulate, releasing the contents of their alpha granules and dense granules, most notably platelet derived growth factor and transforming growth factor b. These substances initiate chemotaxis and proliferation of inflammatory cells, beginning the inflammatory response that will ultimately heal the wound. Tumor necrosis factor and interleukin-1 also stimulate fibroblast proliferation, however are produced by macrophages. 129 A patient with gross fecal contamination and peritonitis from a ruptured sigmoid diverticulum has his midline wound left open to heal by secondary intention. Which of the following statement(s) describes this healing process? a. Wounds healing in this fashion have an altered sequence of healing compared to a primarily closed wound b. A bed of granulation tissue forms over exposed subcutaneous tissue c. Epithelialization is enhanced in the face of bacterial colonization d. The ability of a wound to form granulation tissue is dependent on the blood supply of the tissue Answer: b, d Open wounds, whether they be ulcers or open surgical incisions closing by secondary intention, heal with the same sequence of inflammation, matrix deposition, epithelialization, and scar maturation as in all wounds. The major difference is in the healing incisional wound, the healing process progresses in an orderly temporal sequence. In an open wound, the healing events are spatially separated. In the healing wound, a bed of granulation tissue forms over the exposed subcutaneous tissue. Granulation tissue is composed of new capillaries, proliferating fibroblasts, an immature matrix of collagen, proteoglycans, substrate adhesion molecules, and acute and chronic inflammatory cells. Granulation tissue is the cobblestone pink surface of the healthy new tissue in an open wound. The ability of an open wound to form granulation tissue is governed by the blood supply to the tissue and the relative absence of devitalized tissue and bacteria. Epithelialization is therefore enhanced by limiting bacterial growth which presumably interferes via bacterial and phagocytic cell products such as proteases, collagenases, elastases, and other enzymes. 130 Which of the following factors can be associated with impaired wound healing? a. Chemotherapy b. Chronic steroid use c. Peripheral vascular disease d. Radiation therapy e. Diabetes mellitus Answer: a, b, c, d, e Bone marrow suppression, a common consequence of chemotherapy, is detrimental to wound healing. Quantitative and qualitative lymphocyte and monocyte deficiency impairs cellular proliferation in the inflammatory phase of wound healing. Any chemotherapeutic agent that suppresses the bone marrow will impair healing. Glucocorticoids inhibit wound healing based on their anti-inflammatory and immunosuppressive effects. The anti-inflammatory effect of steroids is, in part, the result of inhibiting arachidonic acid metabolism by impairing macrophage migration, and by altering neutrophil function. Glucocorticoids also inhibit the synthesis of procollagen by fibroblasts, thus delaying wound contraction. Radiation injury leads to arteriolar fibrosis and impaired oxygen delivery. In addition, there is progressive obliteration of blood vessels in the radiated area over time. Radiation also causes intranuclear and cytoplasmic damage to fibroblasts, and this appears to limit their proliferative potential. Diabetes mellitus is often associated with decreased healing of open wounds and increased susceptibility of infection. Many factors contribute to poor healing in diabetic patients and most of them reflect local wound ischemia. However, healing is not impaired in a normally perfused area in a well-controlled diabetic. Peripheral arterial occlusive disease secondary to atherosclerosis can be a primary cause of impaired healing, and may be also a cofactor with other conditions. 131 Which of the following cells or blood elements play a role in the initial phases of wound healing? a. Polymorphonuclear leukocytes (PMNs) b. Platelets c. Monocytes d. Lymphocytes Answer: a, b, c, d Shortly after the initial injury, the wound is full of debris which is cleared over the next several days by recruited and activated phagocytic cells. PMNs begin to arrive immediately, reaching large numbers within 24 hours. The PMNs are followed by macrophages which appear in wounds in significant numbers within two to three days. Macrophages are mononuclear phagocytic cells derived from circulating monocytes or resident tissue macrophages. They complete the process of removing all material not necessary for the ensuing steps of wound healing. Lymphocytes also appear in wounds in small numbers during the inflammatory response. The role of lymphocytes in the wound healing process remains to be clarified, but they are thought to be more related to the chronic inflammatory processes than the initial response to wounding. Platelets are anuclear discoid blood elements derived from bone marrow megakarocytes which play a role in the initial hemostatic process as well as releasing chemotactic factors and factors leading to fibroblast proliferation. 132 Which of the following surgical techniques lead to improved wound healing? a. Atraumatic handling of tissue b. Approximation of underlying fatty tissue to obliterate dead space c. Protecting the wound from water for at least one week d. Meticulous hemostasis Answer: a, d There are numerous practical implications for the care of wounds and surgical incisions. Meticulous hemostasis reduces the inflammation of phagocytosis necessary to clear the wound of blood. Atraumatic handling of tissue decreases the load of necrotic or nonviable cells at the wound margin. Deep sutures are best placed only into collagen laden structures that will hold tension, i.e., fascia and dermis. These tissues have a tensile strength to hold sutures under tension. Fat does not contain collagen and will not hold tension. Therefore, fatty tissue should not be sutured as a separate layer. Given that epithelialization of an incision is normally complete within 24–48 hours, there is no reason to protect the incision from water beyond this time period. Allowing the patient to wash or shower one or two days after surgery actually serves useful purpose in debriding the wound. 133 Which of the following statement(s) is/are true concerning the clinical management of an open wound? a. A wet-to-dry dressing is the most optimal form of wound management b. A moist occlusive dressing promotes epithelialization and reduces pain c. The protein rich plasma exudate covering the open wound facilitates healing d. Irrigation of the wound disrupts epithelialization therefore inhibiting the healing process Answer: b Epithelialization is more rapid under moist conditions than dry conditions. Without dressings, a superficial wound, or one with minimal devitalized tissue forms a scab or crust, meaning that the blood and serum will coagulate, dry, and form a protective moisture barrier over the open wound. If a wound is kept moist with an occlusive dressing, then epithelial migration is optimized. In addition, the pain of an open wound is dramatically reduced under an occlusive dressing. The traditional wet-to-dry dressing if truly left to dry, simply produces desiccation and necrosis of the surface layer of the wound which delays epithelialization. Although wet-to-dry dressings can be effective for debridement of wound exudate, they are generally less desirable than a moist healing environment combined with effective cleaning of the wound (i.e. water irrigation). Any open wound will leak plasma. With more inflammation, the plasma capillary permeability is further increased. This exudate of serum proteins and inflammatory cells serves as a rich culture medium. This, in turn, will continue to cycle bacterial proliferation and lead to further exudate formation. The net result of this cycle is delayed or absent wound healing. In addition, the edema that results from capillary dysfunction, increases the distance for diffusion from oxygen and nutrient sources to their metabolic targets. 134 Which of the following statement(s) is/are correct concerning the management of an open wound? a. Frequent surgical debridement is usually necessary b. Water irrigation can effectively debride most wounds c. Hydrogen peroxide is particularly useful in the management of open wounds d. A number of the newer dressing products have clearly been shown to promote wound healing compared to simple moist occlusive dressing Answer: b Although there are numerous dressing products commercially available at present, no treatment has been demonstrated to improve healing beyond that of standard treatment which adheres to basic principles. In the absence of large amounts of necrotic tissue, wound debridement does not need to be accomplished surgically. Simple water irrigation either with whirlpool or by water from a hand held shower spray can generate enough power to effectively debride most wounds. Frequent moist dressing changes can accomplish this as well, and in some cases, occlusive absorptive dressings can generate enough tissue proteases to effectively degrade proteins which the absorptive dressings remove. Deeper portions of a wound may accumulate exudate and bacteria. In such cases, water irrigation may be particularly useful. Commonly used agents such as hydrogen peroxide actually may be harmful to normal tissue and are weak oxidants and do a poor job of debriding. Enzymatic debriding agents can be effective when used properly. Most of the newer dressing products have been designed to be more absorptive and achieve moist healing without infection from excess exudate. However, it must be emphasized that as long as moist healing is achieved, there has been no evidence that one product is better than another. 135 Which of the following statement(s) is/are true concerning the proliferative phase of wound healing? a. The macrophage is the predominant cell type b. The pink or purple-red appearance of a wound is due to ingrowth and proliferation of endothelial cells c. Collagen, the dominant structural molecule of the wound matrix, contains two unique amino acids, hydroxyproline and hydroxylysine d. The predominant collagen type in a scar is type 3 Answer: b, c The proliferative phase of wound healing begins with the formation of a provisional matrix of fibrin and fibronectin as part of the initial clot formation. Initially, the provisional matrix is populated by macrophages; however, by day three fibroblasts appear in the fibronectin-fibrin framework and initiate collagen synthesis. Fibroblasts proliferate in response to growth factors become the dominant cell type during this phase. Growth factors produced by macrophages simultaneously induce angiogenesis which results in the ingrowth and proliferation of endothelial cells, forming new capillaries. This neovascularity is visible through the epithelium and gives the wound a pink or purple-red appearance. Collagen is the dominant structural molecule in the wound matrix and in the final scar. Collagen is synthesized into an organized cable-like network in a multi-step process with both intra- and intercellular components. The collagen molecule has quantities of two unique amino acids, hydroxyproline and hydroxylysine. The hydroxylization processes which form these amino acids require ascorbic acid (vitamin C) and is necessary for the subsequent stabilization and cross linkage of collagen. The principal collagen type scar is type 1, with lesser amounts of type 3 collagen also present. 136 Which of the following statement(s) is/are true about the role of macrophages in the wound healing process? a. Macrophages are the dominant cell type during the inflammatory phase of wound healing b. Macrophages are not essential for wound healing c. The macrophage role in wound healing is limited to phagocytosis d. Macrophages are a source of a number of humoral factors essential for wound healing Answer: a, d Within three or four days after injury, macrophages become the dominant cell type in the inflammatory phase of wound healing. The role of macrophages is not limited only to phagocytosis. In addition, macrophages are the source of more than 30 different growth factors and cytokines. These growth factors induce fibroblast proliferation, endothelial cell proliferation (angiogenesis), extracellular matrix production, and recruit and activate additional macrophages. The result is the induction of a wound healing amplification cycle as growth factors recruit macrophages and elicit additional growth factor release. Experimental studies in which antibodies, which either destroy PMNs or block certain aspects of their function, have shown that wounds heal normally, but that healing is significantly impaired without functional macrophages. These studies confirm the dominant role of the macrophage and the inflammatory phase of wound healing. 137 Which of the following statement(s) is/are true concerning the role of antibiotics in wound care? a. Systemic antibiotics are indicated for all open wounds b. Bacterial resistance can occur with systemic but not topical antibiotics c. An indication for systemic antibiotic administration is a granulation tissue bacterial count in excess of greater than 105 organisms/gram of tissue on quantitative analysis d. Silver sulfadiazine is useful only for the management of burns Answer: c The role of antibiotics in wound care is controversial. All open wounds are colonized with bacteria. Only when surrounding tissue is invaded (cellulitis) are systemic antibiotics clearly indicated. Antibiotics may also be useful in other situations such as when granulation tissue has a high bacterial count (> 105 organisms/gram tissue), or in the case of reduced resistance to bacteria such as in a diabetic foot ulcer. The routine use of systemic antibiotics for chronic wounds should be avoided to reduce the development of resistant bacterial strains within the wound. Topical ointments are frequently used and can be useful. The topical vehicle may help keep the wound moist and the bacterial count in the wound may be lowered as the result. However, as with most antibiotics, resistant organisms quickly emerge. Silver sulfadiazine, frequently used for burn care, is also useful for chronic wounds. Its broad spectrum of activity, lack of relevant drug-resistant plasmids in bacteria, and its low cost make it a good choice.

138 Which of the following statement(s) is/are true concerning wound contraction?
a. Wound contraction accounts for similar rates of reduction of wound size regardless of their location
b. The fibroblast, at the cellular level, is the primary force driving wound contraction
c. Excessive wound contraction, when occurring over a joint, may lead to disability
d. Actin microfillaments are found in fibroblasts and may play a role in wound contracture
Answer: b, c, d

Wound contraction is an important event which contrasts healing open wounds and closed incisions. When open wounds contract, the surrounding skin is pulled over the open wound to reduce its size. This can occur much faster than epithelialization. As opposed to other animals, human skin does not have a significant degree of mobility in most sites and specifically on the lower leg, the skin is tightly adherent and less elastic. Therefore, although contraction may account for 90% of reduction of wound size on the perineum, it accounts for, at most, 30–40% of healing of a lower leg ulcer. All healing wounds generate a strong contractile force. When this force is exerted across a joint, it may result in scar contracture which may limit the functional range of motion. At the cellular level, the force which drives wound contraction comes from fibroblasts. Fibroblasts, like muscle cells, contain actin microfilaments. When these filaments increase in number, the cells take a morphologic appearance of myofibroblasts. Myofibroblasts are seen in an increased number in contracting wounds and are felt to play an active role in the process of wound contraction.

139 There are a multitude of various dressings available. Which of the following statement(s) is/are true concerning options for surgical dressings?
a. Hydrocolloids, such as karaya compounds, offer the primary advantage of increased absorptive ability
b. Films, such as Op-site, provide a water impermeable environment to achieve a dry wound
c. Impregnates are fine gauze impregnated with a variety of substances such as antibiotics or moisturizing agents that adhere tightly to the wound and do not require a secondary dressing
d. Absorptive powders and paste are highly useful in debriding necrotic and fibrous material from wounds and absorbing wound serum
Answer: a, d

Although the simplest dressing of gauze and tape combined with the use of antibacterial ointment can achieve moist wound healing in most patients. A multitude of other products are available. These can be classified into films, foams, hydrocolloids, hydrogels, and absorptive powders. Films are semipermeable to water, generally made of polyurethane, and are nonabsorptive. They are useful to achieve a moist wound healing environment over a minimally exudative wound such as split thickness skin graft donor sites. The hydrocolloids deserve special mention because they have achieved widespread use. These agents contain hydrophilic materials such as karaya or carboxymethyl cellulose with an adhesive material and are covered by a semipermeable polyurethane film. The material adheres to the skin surrounding the wound, is highly absorptive, and achieves a moist healing environment. Impregnants are generally fine mesh gauze impregnated with either moisturizing, antibacterial, or bactericidal compounds. They are generally not adherent and require a secondary dressing. They do promote reepithelialization and have a antiinfective effect when combined with antibacterial or bactericidal agents. A variety of absorptive powders and pastes are available which consist of starch copolymers or colloidal hydrophilic particles. These agents have high absorbency for tissue wound fluid and debride necrotic and fibrous material from the wound.

140 Which of the following statement(s) is/are true concerning the remodeling phase of wound healing?
a. Total collagen content increases steadily through this phase
b. The normal adult ratio of collagen is approximately 4:1 of type I to type III collagen.
c. Eventually a scar will achieve the strength of unwounded skin
d. The proteoglycans are responsible for the ground substance of the extracellular matrix
Answer: b, d

The transition from the proliferative phase to the remodeling phase of wound healing is defined by reaching collagen equilibrium. Collagen accumulation within the wound becomes maximal by two to three weeks after wounding. Although supramaximal rates of synthesis and degradation continue throughout remodeling, there is no further change in total collagen content. During the initial phase of wound healing, there is a relative abundance of type III collagen in the wound. With remodeling, the normal adult ratio of 4:1 (type I to type III) collagen is restored. The other important component of the extracellular matrix is the ground substance or proteoglycans. These substances are composed of a protein background with long hydrophilic carbohydrate side chains. The hydrophilic nature of these molecules accounts for much of the water content of scar.
Scars never achieve the degree of order advanced by collagen in normal skin or tendons, but they do increase in strength for six months or more, eventually reaching 70% of the strength of unwounded skin.

141 Which of the following statement(s) is/are true concerning pharmacologic agents used to accelerate wound healing?
a. A number of these agents are now currently approved for use in this country
b. PDGF (platelet-derived growth factor) promotes fibroblast proliferation, chemotaxis, and collagenase synthesis
c. PDGF has been demonstrated in a number of clinical trials to promote healing in chronic wounds
d. Growth hormone functions by promoting fibroblast proliferation and collagen synthesis
Answer: b, c

Currently there are no approved clinical agents that accelerate normal healing. Although a number of clinical trials are in progress, no agents are currently approved. PDGF (platelet-derived growth factor) accelerates wound healing by promoting fibroblast proliferation and chemotaxis and collagenase synthesis. Clinical trials have demonstrated that PDGF has accelerated healing in patients with chronic wounds such as pressure sores and diabetic ulcers. Growth hormone has been successfully used in some situations to reverse the catabolic effect of severe injuries. Wound healing is fundamentally an anabolic event, and in the setting of a severe burn, growth hormone administration significantly accelerates donor site healing, presumably due to its effects in minimizing catabolism.

142 Which of the following statement(s) describe the effects of aging on wound healing?
a. A finer, more cosmetic scar might be expected
b. In vitro studies demonstrate decreased proliferative potential of fibroblasts and epithelial cells
c. Skin sutures should be left in for a longer period of time
d. Wound infection occurs more frequently in elderly patients due to diminished ability to fight infection
Answer: a, b, c

There are important age-dependent aspects of wound healing. The elderly heal more slowly and with less scarring. There is a gradual attenuation of the inflammatory response with age, and decreased wound healing is one of the consequences. In vitro studies have documented an age-dependent decrease in proliferative potential of fibroblasts and epithelial cells. Clinically this will account for the formation of finer scars and improved cosmetic appearance in the elderly. Sutures should be left in place longer to allow for the slow regain of tensile strength in the aged. This can also be done without concern for formation of suture marks as slower epithelialization occurs along the sutures. There is no evidence to suggest that wound infections occur more commonly in elderly patients.

143 Reconstitution of the epithelial barrier (epithelialization) begins within hours of the initial injury. Which of the following statement(s) is/are true concerning the process of epithelialization?
a. Bacteria, protein exudate, and necrotic tissue all will compromise this process
b. Epithelial cells exhibit contact proliferation
c. Epithelialization occurs only from the margins of the wound
d. Visible scarring can occur only when the injury extends deeper than the superficial dermis
Answer: a, d

The initial step of epithelialization involves epithelial cells from the basal layer of the wound edge flattening and migrating across the wound, completing wound coverage within 24–48 hours in a co-opted surgical wound. Epithelial cells exhibit contact inhibition. That is, they will continue to migrate across an appropriate bed until a single continuous layer is formed. Epithelial cell migration occurs by a process in which the epithelial cells send out pseudopods, attaching to the underlying extracellular matrix by integrin receptors. Bacteria, large amounts of protein exudate from leaky capillaries, and necrotic tissue all compromise this process delaying epithelialization. In the case of open wounds, epithelialization results from migration of epithelial cells from remaining dermal appendages, sweat glands, and hair follicles, if the dermis is not completely destroyed. In a full thickness injury, the entire dermis is destroyed or removed. Epithelialization therefore occurs only at the margins of a wound, at a dermal rate of 1–2 mm/day.
Visible scarring occurs only when the injury extends deeper than the superficial dermis. Superficial abrasions and burns usually heal without scar, while deeper abrasions and burns may scar significantly. Whenever the dermis is incised, a scar will form.

144 Scar formation is part of the normal healing process following injury. Which of the following tissues has the ability to heal without scar formation?
a. Liver
b. Skin
c. Bone
d. Muscle
Answer: c

Every tissue in the body undergoes reparative processes after injury. Bone has the unique ability to heal without scar and liver has the potential to regenerate parenchyma, the only organ that has maintained that ability in the adult human. Although liver does regenerate, it often heals with scar (cirrhosis) as well. With these exceptions, all other mature human tissues heal with scar.

145 Which of the following factors have been demonstrated to promote wound healing in normal individuals?
a. Vitamin A supplementation
b. Vitamin C supplementation
c. Vitamin E application to the wound
d. Zinc supplementation
e. None of the above
Answer: e

Several important systemic factors or conditions influence wound healing. Interestingly, there are no known systemic conditions that lead to enhanced or more rapid wound healing. Overall nutrition as well as adequate vitamins play an important role in wound healing. Vitamin A is involved in the stimulation of fibroplasia, collagen cross-linking, and epithelialization. Although there is no conclusive evidence in humans, vitamin A may be useful clinically for steroid-dependent patients who have problematic wounds or who are undergoing extensive surgical procedures. Vitamin C is a necessary cofactor in hydroxylization of lysine and proline in collagen synthesis and cross-linkage. The utility of vitamin C supplementation in patients who otherwise take in a normal diet has not been established. Vitamin E is applied to wounds and incisions empirically by many patients. The evidence to support this practice is entirely anecdotal. In fact, large doses of vitamin E have been found to inhibit wound healing. Zinc and copper are also important cofactors for many enzyme systems that are important to wound healing. Deficiency states are seen with parenteral nutrition but are rare and readily recognized and treated with supplements. Overall, vitamin and mineral deficiency states are extremely rare in the absence of parenteral nutrition or other extreme dietary restrictions. There is no evidence to support the concept that supranormal provision of these factors enhance wound healing in normal patients.

146 Which of the following statement(s) is/are true concerning excessive scarring processes?
a. Keloids occur randomly regardless of gender or race
b. Hypertrophic scars and keloid are histologically different
c. Keloids tend to develop early and hypertrophic scars late after the surgical injury
d. Simple reexcision and closure of a hypertrophic scar can be useful in certain situations such as a wound closed by secondary intention
Answer: d

True keloids are uncommon and occur predominantly in dark skinned people with a genetic predisposition for keloid formation. In most cases, the gene appears to be transmitted as an autosomal dominant pattern. The primary difference between a keloid and a hypertrophic scar is that a keloid extends beyond the boundary of the original tissue injury. It behaves as a tumor and extends into or invades the normal surrounding tissue creating a scar that is larger than the original wound. Histologically, keloids and hypertrophic scars are similar. Both contain an overabundance of collagen. Although the absolute number of fibroblasts is not increased, the production of collagen continually out paces the activity of collagenase, resulting in a scar of ever increasing dimensions. Hypertrophic scars respect the boundaries of the original injury and do not extend into normal unwounded tissue. There is less of a genetic predisposition, but hypertrophic scars also occur more frequently in Orientals and the Black population. They are often seen on the upper torso and across flexor surfaces. Some improvement in a keloid can be obtained with excision followed by intra-lesional steroid injection. However, the resulting scar is unpredictable and potentially worse. Reexcision and closure should, however, be considered for hypertrophic scars, if the condition of closure can be improved. This is especially pertinent for wounds that originally healed by secondary intention or that are complicated by infection. Keloids typically develop several months after the injury and rarely, if ever, subside. Hypertrophic scars usually develop within the first month after wounding and often subside gradually.

147 Which of the following statement(s) is/are true concerning the vascular response to injury?
a. Vasoconstriction is an early event in the response to injury
b. Vasodilatation is a detrimental response to injury with normal body processes working to avoid this process
c. Vascular permeability is maintained to prevent further cellular injury
d. Histamine, prostaglandin E2 (PGE2) and prostacyclin (PGI2) are important mediators of local vasoconstriction
Answer: a

After wounding, there is transient vasoconstriction mediated by catecholamines, thromboxane, and prostaglandin F2 (PGF2a). This period of vasoconstriction lasts for only five to ten minutes. Once a clot has been formed and active bleeding has stopped, vasodilatation occurs in an around the wound. Vasodilatation increases local blood flow to the wounded area, supplying the cells and substrate necessary for further wound repair. The vascular endothelial cells also deform, increasing vascular permeability. The vasodilatation and increased endothelial permeability is mediated by histamine, PGE2, and prostacyclin as well as growth factor VEGF (vascular endothelial cell growth factor). These vasodilatory substances are released by injured endothelial cells and mast cells and enhance the egress of cells and substrate into the wound and tissue.

148 Which of the following statement(s) concerning laboratory studies used in monitoring a patient with intravenous heparinization is/are correct?
a. The platelet count should be followed because of the risk of heparin-associated thrombocytopenia
b. The prothrombin time should be observed if prolonged treatment is necessary
c. The activated partial thromboplastin time (aPTT) should be maintained at approximately 1.5 times normal
d. The serum creatinine should be measured daily to allow adjustments in dose based on renal function
Answer: a, c

In monitoring the effect of heparin, an activated partial thromboplastin time (aPTT) of 1.5 control or a thrombin clotting time (TCT) of 2 times control reflects adequate anticoagulation. The prothrombin time remains normal. Heparin-associated thrombocytopenia from an immune mechanism is a potential complication of the use of this anticoagulant. Therefore any patient undergoing heparin therapy should have a platelet count determined every other day after the fourth day of therapy or earlier if he or she is known to have been exposed to heparin in the past. Heparin is not excreted through the kidneys or the liver but is cleared through the reticuloendothelial system. Therefore the dose of heparin need not be adjusted in cases of liver or renal dysfunction.

149 Which of the following statement(s) is/are true concerning heparin-associated thrombocytopenia?
a. Heparin-associated thrombocytopenia occurs only in the face of over anticoagulation with heparin
b. Severe thrombocytopenia (platelet count less than 100,000) is seen in less than 10% of patients treated with heparin
c. Heparin-associated thrombocytopenia is due to the aggregation of platelets and may result in thrombosis or embolic episodes
d. Heparin-associated thrombocytopenia may be seen within hours of initiation of heparin therapy
Answer: b, c

Heparin-associated thrombocytopenia occurs in 0.6% to 30% of patients who receive heparin, although severe thrombocytopenia (platelet counts less than 100,000) is seen in fewer than 10% of patients treated with heparin. It is caused by a plasma factor, most likely a heparin-dependent platelet antibody, that causes aggregation of platelets when exposed to heparin. Activation of platelets in this setting results in thrombocytopenia, thrombosis and embolic episodes, which can lead to death. Both bovine and porcine heparin have been associated with this syndrome, which usually begins 5 to 15 days after initiating heparin therapy. Even trivial exposure with heparin such as coating on pulmonary artery catheters or low rate infusion into arterial catheters may cause this syndrome.

150 Antithrombin III deficiency is a commonly observed hypercoaguable state. Which of the following statement(s) is/are true concerning this condition?
a. A patient with this deficiency usually presents with thrombosis while on heparin or exhibits an inability to become adequately anticoagulated with heparin
b. This deficiency may be either congenital or acquired
c. Thrombotic episodes are related to predisposing events such as operations, childbirth, and infections
d. Treatment involves acutely the administration of fresh frozen plasma followed by long-term treatment with Coumadin
Answer: a, b, c, d

Antithrombin III deficiency accounts for about 2% of venous thrombotic event. This deficiency has been described in patients with pulmonary embolism, mesenteric venous thrombosis, lower extremity venous thrombosis, arterial thrombosis, and dialysis fistula failure. Antithrombin III is a serine protease inhibitor of thrombin and factors Xa, IXa and XIa. Because one of the main actions of heparin is to potentiate the anticoagulant effects of antithrombin III, a patient with this deficiency usually presents with thrombosis while on heparin or exhibits the inability to become adequately anticoagulated with heparin. This deficiency may be either congenital (1n2000–5000 births) or acquired. Acquired defects occur with inadequate production, as in liver disease, malignancy, nephrotic syndrome, disseminated intervascular coagulation, malnutrition, or increased protein catabolism. Thrombotic episodes are related to predisposing events such as operations, childbirth, and infections. Once the diagnosis of antithrombin III deficiency is established, fresh frozen plasma should be administered followed by long-term treatment with Coumadin.

151 Mini-dose heparin has been shown to be useful in the prophylaxis of postoperative venous thrombosis. Mechanism(s) by which low-dose heparin is/are thought to protect against venous thrombosis include:
a. Enhancement of antithrombin III activity
b. A decrease in thrombin availability
c. Inhibition of platelet aggregation and subsequent platelet release action
d. A mild prolongation of activated partial thromboplastin time
Answer: a, b, c

Low-dose heparin is thought to protect against venous thrombosis through three different mechanisms. First, antithrombin III activity with its inhibition of activated Factor X is enhanced by only trace amounts of heparin; second, there is a decrease in thrombin availability that prevents its activation and thus its fibrin-stabilizing effect; and third, small doses of heparin may inhibit the second wave of platelet aggregation and subsequent platelet release reaction. The standard doses of heparin administered (5000 units bid) does not affect aPTT.

152 Tests of coagulation are used to monitor anticoagulation treatment and detect intrinsic abnormalities in coagulation. Which of the following statement(s) is/are true concerning coagulation tests?
a. Prothrombin time (PT) measures both the intrinsic and extrinsic clotting pathways and fibrinogen
b. Activated partial thromboplastin time (aPTT) can be used to monitor both oral anticoagulation with Warfarin and intravenous anticoagulation with heparin
c. Thrombin clotting time (TCT) is a measurement of the time it takes for exogenously administered thrombin to turn plasma fibrinogen into fibrin clot
d. Whole blood activated clotting time (ACT) is a measurement of the ability of whole blood to clot and is used to monitor heparin levels intraoperatively during cardiovascular and peripheral vascular operations
Answer: a, c, d

Coagulation tests include prothrombin time (PT), which measures the intrinsic and extrinsic pathways of fibrinogen production and is the most common method for measuring a level of oral anticoagulant therapy. The activated partial thromboplastin time (aPTT) identifies the abnormalities of the contact and intrinsic phases of coagulation. Values of aPTT have variably been shown to correlate with heparin dosages and serum heparin levels and are therefore most commonly used in monitoring heparin therapy. It is of no value in long-term management of patients on oral Warfarin therapy. Thrombin clotting time (TCT) is the measure of the time it takes for exogenously administered thrombin to turn plasma fibrinogen into fibrin clot. It is extremely sensitive to levels of heparin and is an excellent measure of measuring the level of heparin-induced anticoagulation. The beauty of the TCT is that it is not specific for any disease condition; thus it may be used to differentiate factor deficiencies from the presence of heparin, or to separate lupus anticoagulant from abnormalities in fibrinogen levels. The whole blood activated clotting time (ACT) is a measurement of the ability of whole blood to clot, and as such, is an available technique for monitoring heparin levels intraoperatively. The ACT responds in a linear fashion to increasing heparin dosage and correlates well with the observed clinical anticoagulation. Adequate anticoagulation for extracorporeal circulation is defined as an ACT of 480 seconds or more while for peripheral vascular applications, values of 250 seconds or greater are considered appropriate.

153 Thrombolytic therapy has become a useful adjunct in the management of peripheral arterial occlusion. In this setting, direct intraarterial administration rather than intravenous has been advocated to decrease the risk of systemic bleeding. Which of the following true statement(s) concerning the use of thrombolytic agents for arterial occlusion is/are true?
a. A standard technique involves infusing high-dose urokinase, 4000 units per minute for 1–2 hours, directly into the clot by a catheter embedded in the thrombus
b. If progress is made, further fibrinolytic therapy is given at 1000 to 2000 units per minute until clot lysis has occurred
c. The usual infusion time by the above-stated technique is usually in excess of 24 hours
d. Successful clot lysis occurs more frequently in arterial graft occlusions than native arterial occlusions
e. The use of intraoperative thrombolytic therapy may be indicated for situations where complete clot evacuation cannot be accomplished surgically
Answer: a, b, e

The most popular method for intraarterial thrombolytic therapy for arterial occlusion involves passing a guidewire through the thrombus with arteriographic guidance and then infusing high-dose urokinase, 4000 units per minute for 1–2 hours, directly into the clot. If progress is made, further fibrinolytic therapy is given at 1000 to 2000 units per minute for a 6–12 hour period or until clot lysis has occurred. Using this technique, mean infusion time in a recent study was found to be 18 hours and the incidence of bleeding complications was significantly lessened. Selective intraarterial infusion of urokinase was associated with complete clot resolution in 77% of native arterial occlusions versus only 41% with arterial graft occlusion. After thrombolytic therapy has reopened an occluded vessel or graft, radiologic or surgical correction of the lesion responsible for the thrombosis in the first place must be addressed for any hope of long-term success. The use of intraoperative thrombolytic therapy is advocated in those situations where complete clot resolution cannot be accomplished (such as following balloon embolectomy for acute arterial occlusion) or when distal vasculative is occluded and precludes appropriate inflow patency.

154 Which of the following statement(s) is/are true concerning hemophilia A?
a. Hemophilia A is inherited as a sex-linked recessive deficiency of factor VIII
b. A positive family history for bleeding disorders present in all patients
c. Laboratory tests reveal a prolongation of aPTT, prothrombin time (PT), thrombin clotting time and platelet aggregation
d. Spontaneous bleeding is unusual with factor VIII levels greater than 10% of normal
Answer: a, d

Hemophilia A is inherited as a sex-linked recessive deficiency of factor VIII although 0% of cases are secondary to spontaneous mutation. The incidence of this abnormality is approximately 1/10,000 births. Laboratory screening tests usually reveal a prolongation of an aPTT but normal prothrombin time (PT), thrombin clotting time (TCT) and platelet aggregation testing. The minimum level of VIII required for hemostasis is 30% for minor bleeding, whereas spontaneous bleeding is unusual with factor levels greater than 5 to 10% of normal. In severe genetic deficiency states however, factor levels as low as 1% have been noted and patients are at risk for spontaneous bleeding.

155 Fibrinolytic therapy is based on activation of plasminogen, the inactive proteolytic enzyme of plasma that binds to fibrin during the formation of thrombosis. Activation of plasminogen to plasmin results in selective thrombolysis at the fibrin clot surface. Which of the following statement(s) is/are true concerning agents used in thrombolytic therapy?
a. Streptokinase is a bacterial protein which is antigenic in humans, resulting in allergic reactions in up to l5% of cases
b. Tissue plasminogen activator acts directly on plasmin without an intermediate drug–plasmin complex
c. The half-life of urokinase, streptokinase, and TPA all exceed 30 minutes
d. Streptokinase is significantly cheaper than urokinase or TPA
Answer: a, b, d

Streptokinase is a bacterial protein produced by group C b-hemolytic streptococci. It is therefore antigenic in humans and can be associated with allergic reaction in between 2 and 18% of cases. In addition an unusual serum sickness has been reported with streptokinase. Neither urokinase or TPA which is now manufactured with recombinant DNA technology are either associated with allergic side effects or antigenicity. Streptokinase acts through a streptokinase-plasmin complex, whereas urokinase and TPA act directly on plasmin without intermediate drug plasmin complex. The level of the lytic state is greatest with streptokinase, intermediate with urokinase, and least with TPA with the half-lives ranging all less than 1/2 hour in duration. Although the relative efficacy of the three agents has been compared in a number of studies, there appears to be no significant benefit of one agent over the other. Streptokinase however, is markedly less expensive than either urokinase or TPA.

156 Von Willebrand’s disease is a common, congenital bleeding disorder. Which of the following statement(s) is/are true concerning Von Willebrand’s disease?
a. As in hemophilia, it is much more common in men
b. A history of spontaneous bleeding is common
c. Screening laboratory tests will include a prolonged aPTT with a normal prothrombin time
d. Pre-treatment for elective surgery require administration of cryoprecipitate to achieve levels of 23–50% of normal
Answer: c, d

Von Willebrand’s factor is an adhesive protein that mediates platelet adhesion to collagen. In addition, it protects and prevents the rapid removal of factor VIII from blood. The classical deficiency state, Von Willebrand’s disease, is caused by reduction of factor VIII activity (although not as great as Hemophilia A) and the Von Willebrand factor. Clinical manifestations include epistaxis, gingival bleeding, menorrhagia, rare joint or muscle bleeding, and subcutaneous bleeding. Spontaneous bleeding is not as common as in classic Hemophilia A. The syndrome is transmitted as both autosomal dominant (heterozygous) and autosomal recessive disease (homozygous) traits. Therefore there is no sex predilection. Screening laboratory tests include a prolonged aPTT with a normal prothrombin time. In addition, because of the importance of this factor in platelet adhesion, patients display a prolonged bleeding time and have decreased level of factor VIII activity, decreased immunoreactive levels of Von Willebrand’s antigen, and abnormal platelet aggregation responses to ristocetin. The most reliable source of Von Willebrand’s factor is cryoprecipitate.

157 External pneumatic compression has been advocated for the prevention of deep venous thrombosis during operative procedures. Which of the following statement(s) concerning the use of external pneumatic compression devices is/are true?
a. Intermittent pneumatic compression is as effective as low-dose heparin in prevention of venous thrombosis
b. These devices function by compressing the lower extremities therefore augmenting venous return
c. Pneumatic compression devices may also exhibit their antithrombotic effect through stimulating local and systemic fibrinolysis
d. The length of time that intermittent pneumatic compression should be used includes through the operation and for at least several days in the postoperative period
Answer: b, c, d

In many well-controlled studies of venous prophylaxis, intermittent pneumatic compression has been found to be as effective as low-dose heparin therapy. In addition to augmentation of venous return with these devices, local and systemic fibrinolysis appears to be stimulated. Fibrinolytic activities are usually reduced for a 7–10 day period after an operation. Studies have demonstrated that the pneumatic-compression devices may exhibit their antithrombotic effect through prevention of this fibrinolytic shutdown even when applied to the upper extremity. The length of time that intermittent pneumatic compression should be used has not been adequately determined but most data suggest that devices should be used through the operation and for at least five days in the face or prolonged immobilization.

158 The standard management oral anticoagulant therapy for chronic treatment of venous thromboembolism is with the drug warfarin. Which of the following statement(s) is/are true concerning the administration of warfarin?
a. An important complication of warfarin therapy is skin necrosis in patients with protein C deficiency
b. Warfarin interferes with vitamin K dependent clotting factors II, VII, IX, X
c. For effective anticoagulation the prothrombin time (PT) should be kept at 2 control
d. It is recommended that warfarin be continued for at least one year after initial episode of deep venous thrombosis
Answer: a, b

Warfarin interferes with the vitamin K dependent clotting factors II, VII, IX and X, protein C, and protein S. An important complication of warfarin is skin necrosis with patients both with and without protein C deficiency. This syndrome usually involves full thickness skin slough over fatty areas such as the breasts and buttocks. Warfarin therapy should be monitored using the one stage prothrombin time (PT). The PT should be kept at 1.3 to 1.4 control for effective anticoagulation. At higher levels, there is a five-fold increase in the frequency of bleeding complications. Two major complications of Warfarin therapy include recurrent thrombosis and bleeding. It is recommended that Warfarin be continued four months after an initial episode of deep venous thrombosis. Between ten weeks and four to six months after deep vein thrombosis, there is a recurrent thrombosis rate of 8.3 episodes per 1000 patient months. Between four months and three years, recurrences fall to four episodes per 1000 patient-months. At four months, the risks of bleeding complications matches and exceeds the benefit from anticoagulant therapy and thus is the basis for discontinuing warfarin administration at this time.

159 Which of the following statement(s) is/are true concerning the management of a patient with hemophilia A undergoing an elective surgical operation?
a. Concentrates of factor VIII should be given several days prior to elective surgery
b. The half-life of factor VIII concentrates is less than 24 hours
c. A dose of 40–50 IU/kg of factor VIII concentrate should be given prior to the planned surgical procedure
d. Factor VIII concentration administration should be given for the first 24 hours after surgery but may then be stopped if no abnormal bleeding has been observed
e. A new recombinant preparation of factor VIII offers the advantage of being virus-free
Answer: b, c, e

Although the half-life of factor VIII is 2.9 days in normal individuals, the half-life of factor VIII concentrates is 9 to l8 hours. Levels of 80% to 100% of normal should be obtained for surgical bleeding or life-threatening hemorrhage. A dose of 40 to 50 IU/kg of factor VIII should be given with half of this dose then administered every twelve hours. After surgery, transfusion of factor VIII concentrates should be continued for at least ten days. Unfortunately, past use of concentrates of factor VIII obtained from donors has led to a high incidence of HIV infection in the hemophilia population. A new recombinant preparation of factor VIII offers the advantage of being virus-free.

160 Transfusions of blood products can be associated with a number of complications including immediate and delayed hemolytic reactions; nonhemolytic reactions; infectious disease transmission; and complications of massive transfusions. Which of the following statements are true concerning complications of blood transfusions?
a. Immediate hemolytic transfusion reactions are caused by major ABO blood group incompatibility
b. Nonhemolytic transfusion reactions are usually due to RH incompatibility and are therefore more common in women of childbearing age
c. The most common complication of massive blood transfusion is dilutional thrombocytopenia
d. Routine impaired calcium supplementation is necessary during most massive transfusion episodes
Answer: a, c

Immediate hemolytic reactions are usually caused by blood group ABO incompatibility although they may be caused by antigens of other blood group systems on the transfused red blood cells. The clinical manifestations revolve around the antigen on the red blood cell stroma and the antibody in the patient’s serum, and include production of bradykinin, compliment activation, release of vasoactive agents from platelets, and initiation of systemic clotting. Chills and fevers, chest pain and lumbar pain, tachycardia and hypotension in the conscious patient, and often diffuse bleeding in the anesthetized, unconscious patient constitute this syndrome. Although reaction occurs immediately, death related to the syndrome is uncommon, unless associated with a transfusion of more than 100 ml of blood. Death usually occurs from acute renal failure or hemorrhage due to DIC. Nonhemolytic reactions occur with the frequency of 1 to 2% of all transfusions and consist primarily of chills and fevers during the transfusion or in the first 2 to 3 hours after the transfusion is complete. Mechanism of these reactions includes the presence of antibodies to white blood cell antigens in the transfused blood, especially in the multitransfused or multiparous patient. Massive transfusion complications relate to the rate and volume of blood transfused. The most common complication is dilutional thrombocytopenia. Factor deficiency of the labile factors V and VIII rarely is of sufficient magnitude to result in problems with hemostasis. For hypocalcemia to occur with massive transfusion, citrated blood must be administered, one unit every five minutes. Routine empiric calcium supplementation is unnecessary during most massive transfusion episodes. Conversely, hypothermia is clearly a problem, especially when associated with massive transfusion during complex intraoperative procedures such as thoracoabdominal aneurysm resection.

161 A 67-year-old male with advanced cholangiocarcinoma develops gram-negative sepsis. Excessive bleeding is noted around vascular catheters and from needle puncture sites. The diagnosis of disseminated intervascular coagulation (DIC) is considered. Which of the following laboratory test(s) is/are indicative of DIC?
a. Decreased platelet count
b. Decreased fibrinogen level
c. Normal prothrombin time
d. Elevated fibrin split products
Answer: a, b, d

Disseminated intravascular coagulation (DIC) is the primary form of acute thrombosis. Causes of this syndrome include abruptio placenta, gram-positive and gram-negative sepsis, endotoxemia, malignant tumors, pelvic operations, certain snake bites, hematologic malignancies, and hepatic failure. Blood coagulation is activated by the release of tissue factor into the circulation, which activates factor VII of the extrinsic pathway to VIIa, leading to massive thrombin production and fibrin generation. This in turn activates the fibrinolytic system, leading to bleeding in the later stages of the syndrome due to consumption of coagulation factors, depletion of fibrinogen, and unchecked plasma activities. Laboratory values in DIC usually include a decline in the platelet count and fibrinogen level, along with an elevation of fibrin split products.

162 Which of the following substances, not normally present in the circulation, trigger the initiating events in the hemostatic process?
a. Thrombin
b. Platelet factor 3
c. Tissue factor
d. Collagen
Answer: c, d

The initiating agents for hemostasis involve two substances that are not normally present in the circulation—collagen and tissue factor. Tissue factor is released from injured cells, beginning the activation of the extrinsic pathway of coagulation, while disruption of the protective endothelial barrier of blood vessels exposes the underlying collagen to the activation of platelets. In the bloodstream, tissue factor complexes with factor VII which then activates factor X to factor Xa. At the same time, activated platelets change from their discoid shape with their procoagulant phospholipid (termed platelet factor 3) buried on the inner side of the surface membrane to a spreading shape to allow for the externalization of platelet factor 3 activity. Activated factor X, activated factor V, ionized calcium and factor II (prothrombin) then assemble on the platelet phospholipid surface to form the so-called prothrombinase complex which catalyzes the formation of thrombin.

163 Bleeding complications are frequently associated with fibrinolytic therapy. Which of the following statement(s) concerning complications of fibrinolytic therapy is/are true?
a. Careful monitoring of prothrombin time and aPTT time are necessary to avoid bleeding complications
b. A level of serum fibrinogen less than 100 mg/dl is associated with an increased risk of bleeding
c. Recent (less than 10 days) major surgery is a contraindication to systemic but not regional fibrinolytic therapy
d. A patient with a cerebrovascular event occurring less than two months ago can be treated with fibrinolytic therapy if head CT scan is normal
Answer: b

Fibrinolytic therapy induces a hemostatic defect through a combination of factors. Hypofibrinogenemia and fibrin degradation products inhibit fibrin polymerization and, in combination with a decrease in the clotting factors V and VIII, prolong the ability of blood to clot. However, coagulation tests in general do not correlate well with bleeding complications. A level of fibrinogen less than 100 mg/dl is associated with an increased risk of bleeding. Absolute contraindications to thrombolytic therapy include active internal bleeding, recent (less than 2 months) cerebral vascular accident, and documented left heart thrombosis. Recent (less than 10 days) major surgery, obstetric delivery, organ biopsy, or major trauma is considered a major relative contraindication to either regional or systemic thrombolytic therapy.

164 Which of the following statement(s) is/are true concerning the results of a National Institute of Health Consensus Conference on venous thrombosis and low-dose heparin prophylaxis?
a. The odds of developing deep venous thrombosis with low-dose heparin prophylaxis decreases by 67%
b. The risk of pulmonary embolism is decreased by almost 50%
c. There is no increase in mortality from other causes found in patients treated with low-dose heparin
d. There was no difference in the incidence of bleeding complications
Answer: a, b, c

In a metaanalysis of 70 randomized trials in 16,000 patients comparing low-dose heparin prophylaxis with standard therapy, the odds of developing deep venous thrombosis with low-dose heparin prophylaxis decreased 67%, whereas for pulmonary embolism (both fatal and non-fatal), the odds decreased by 47%. For fatal pulmonary embolism, the odds reduction was even greater (64%). No increase in mortality from other causes was found in those patients treated with low-dose heparin. Bleeding complications were more frequent in the heparin-treated patients, with no difference between 5000 units twice daily and 5000 units three times daily. Similarly, the effectiveness of prophylaxis was not influenced by either two or three times daily dosage.

165 Laboratory monitoring of coagulation and anticoagulation includes testing of platelet function. Which of the following statements is/are true concerning tests of platelet function?
a. A platelet count of 50,000/µL or more usually ensures hemostasis
b. Bleeding time assays assessibility of platelets to perform hemostatic plugs and is determined from a sample of blood drawn in an EDTA coated test tube
c. Aspirin therapy can be associated with a bleeding time in the range of 8–15 minutes
d. Tests of platelet aggregation should be part of the standard preoperative evaluation of patients using aspirin
Answer: a, c

Tests of platelet function include peripheral platelet counts, bleeding times, and platelet aggregation. Usually, a platelet count of 50,000/mL or more ensures adequate hemostasis, whereas counts less than 10,000/mL are dangerous and may lead to spontaneous bleeding. Bleeding time performed by observing the clotting of blood induced with a small needle stick, assesses the ability of platelets to perform hemostatic plugs and are usually shorter than eight minutes. A bleeding time between 8 and 15 minutes most often reflects a low plasma level of Von Willebrand’s Factor or the use of antiplatelet drugs. A bleeding time greater than 15 minutes is clearly prolonged and indicates severe platelet functional impairment. Platelet aggregation studies involve the use of a number of different agonists. Although a relatively straightforward technique, platelet aggregation is not available in most laboratories, probably because of the observer-dependent nature of the test.

166 As thrombin generation proceeds, the body has natural anticoagulant systems opposing further thrombus formation. Natural anticoagulants include:
a. Tissue plasminogen activator (TPA)
b. Antithrombin III
c. Activated protein C
d. Heparin cofactor II
Answer: b, c, d

Just as thrombin generation is the key to coagulation, antithrombin III is the most central anticoagulant proteins. This glycoprotein binds to thrombin, preventing its removal of fibrinoprotein A and B from fibrinogen, prevents the activation of factor V and VIII and the activation and aggregation of platelets. The second line of defense is the activated protein C, which inactivates factors Va and VIIIa. This inactivation reduces the ability of the prothrombinase complex to accelerate the rate of thrombin formation. A third natural anticoagulant is heparin cofactor II. Its concentration in plasma is estimated to be some four-fold lower than antithrombin III, and its action is primarily implicated in the regulation of thrombin formation in extravascular tissues. Tissue plasminogen activator (TPA) is a natural catalyst for the activation of plasminogen to plasmin, the main fibrinolytic enzyme in the body. Therefore, TPA is part of the fibrinolytic system rather than a natural anticoagulant.

167 Infectious disease transmission during blood transfusions is of clinical significance to surgeons and of major importance to patients contemplating surgery potentially associated with the need for blood administration. Which of the following statement(s) is/are true concerning the transmission of infectious disease during blood transfusions?
a. Post-transfusion hepatitis is usually due to hepatitis B
b. Hepatitis and HIV transmission is greatest with the administration of pooled plasma products such as serum albumin
c. The most important cause of post-transfusion disease in immunosuppressed patients is CMV infection
d. The risk of HIV transmission in blood transfusions is significantly less than the risk of hepatitis transmission
Answer: c, d

The most common infectious diseases transmitted during blood transfusions include viral hepatitis, CMV, and HIV infection. Post-transfusion hepatitis in 90% of cases consists of non-A, non-B hepatitis known as hepatitis C. All blood products except for immune serum globulin and albumin can carry and transmit this form of hepatitis. Because heat treatment eliminates the risk of viral transmission, products from pooled plasma that are heat treated such as albumin are not at risk for HIV or hepatitis transmission. CMV transmission exists in three forms—primary, reinfection, and reactivation. Primary exposure results in an IgM response to the virus. Reactivation is most commonly related to pregnancy, transplantation, and immunosuppression, and is the most important cause of post-transfusion disease accompanying immunosuppression of patients. Although the risk of the public concern for transmission of HIV disease associated with blood transfusions has significantly outweighed other infectious disease transmission, the risks of HIV transmission is markedly less than that of hepatitis.

168 There are a number of hypercoaguable states which can be associated with arterial or venous thrombosis and embolic phenomenon. These include:
a. Heparin-associated thrombocytopenia
b. Antithrombin III deficiency
c. Von Willebrand disease
d. Vitamin C deficiency
Answer: a, b

A number of hypercoaguable states are present. These include heparin-associated thrombocytopenia in which a heparin-dependent platelet antibody causes aggregation of platelets when the patient is exposed to heparin. Activation of platelets in this setting results in thrombocytopenia, thrombosis, and embolic episodes. Antithrombin III deficiency accounts for about 2% of venous thrombotic events and has been described in pulmonary embolism, mesenteric venous thrombosis, lower extremity venous thrombosis, arterial thrombosis, and dialysis fistula failure. Von Willebrand’s disease is a hereditary complex coagulation factor deficiency which is manifested by a reduction of factor VIII activity, and the Von Willebrand factor which is an adhesive protein that mediates platelet adhesion to collagen. Severe vitamin C deficiency results in a disorder in soft tissue increasing vascular permeability and fragility resulting in the potential for bleeding disorders.

169 Cytokines with clearly defined actions in acute inflammation and early tissue injury include which of the following?
a. Cysteine-X-Cysteine (C-X-C) chemokines
b. Tumor Necrosis Factor (TNFa)
c. Transforming Growth Factor-b (TGF-b)
d. Interleukin-6 (IL-6)
e. Platelet Derived Growth Factor (PDGF)
Answer: a, b, c, d, e

Polypeptide mediators, such as TNFa and IL-1, are considered “early response” cytokines and are actively involved in the initiation of the cascade of events which precipitate acute inflammation. In addition to being important triggers for the induction of other cytokines important inflammatory network, TNFa and IL-1 appear to be key mediators in promoting the adherence of inflammatory cells to the endothelium. IL-1 is a complex, multifunctional molecule that shares many overlapping biological properties with TNFa. In addition, both IL-1 and TNFa potentiate the effects of one another. The most important function of IL-6 appears to be the regulation of the hepatic acute phase response. Following injury, a number of physiologic changes develop within several hours. IL-6 is one of the primary stimuli for the production of acute phase proteins from the liver. Endotoxin, IL-1, TNFa and PDGF are capable of causing significant induction of IL-6 synthesis.
Over the last decade, at least 12 different C-X-C chemokines have been identified. These include IL-8, one of the most potent mediators of chemotaxis known. TNFa and IL-1 are key molecules for the induction of IL-8, which in turn is important for the induction of neutrophil recruitment and activation.
Similar properties are apparent for other members of this chemokine family.
Platelet activation and degranulation occur during coagulation following injury, leading to the deposition of a number of cytokines into the provisional matrix. These cytokines include transforming growth factor-a, (TGFa), transforming growth factor b (TGF-b), platelet-derived growth factor (PDGF), and neutrophil activating peptide-2 (NAP-2). These cytokines are either important growth factors or chemotaxis for leukocytes, endothelial cells, fibroblasts, and keratinocytes which are key components in the process of tissue repair. Thus, coagulation and platelet activation provide the initial foundation for subsequent cellular recruitment.

170 Which of the following statements regarding transforming growth factor b (TGF-b) are true?
a. TGF-b expression is autoregulated
b. TGF-b enhances collagen synthesis
c. TGF-b inhibits extracellular matrix production
d. TGF-b may inhibit or promote cellular proliferation
Answer: a, b, d

TGF-b appears to be one of the key cytokines in control of tissue repair. TGF-b is strongly chemotactic for neutrophils, T cells, monocytes, and fibroblasts. TGF-b activates inflammatory cells to elaborate fibroblast growth factor, TNFa, IL-1 and increase their synthesis of extracellular matrix proteins. TGF-b also induces both the infiltrating cells and resident cells to produce more TGF-b. This auto-induction amplifies its biological effects at the site of injury and may play an important role in the development of chronic fibrosis in a variety of pathologic states. TGF-b enhances collagen synthesis as well. Lastly, TGF-b may function as a mitogen or growth inhibitor for a wide variety of cell types, including selected cell types of mesenchymal origin. Whether TGF-b stimulates or inhibits proliferation depends on the presence of other growth factors, the concentration of TGF-b, and the cell density. Thus, at low doses, TGF-b stimulates the proliferation of densely plated human marrow fibroblasts, but is inhibitory at high concentrations.

171 Leukocyte activation and adhesion to vascular endothelial cells is a critical step in the inflammatory process. This process is regulated by which of the following molecules?
a. The selectins
b. The b5 integrins
c. The immunoglobulin supergene family
d. Nitric oxide
e. IL-8
Answer: a, c, d, e

The temporal events that initiate and propagate neutrophil recruitment and inflammation include endothelial cell activation and expression of endothelial-derived neutrophil adhesion molecules, neutrophil-endothelial cell adherence, and neutrophil transendothelial migration via established neutrophil chemotactic gradients. There are three major families of adhesion molecules which are expressed on the surface of leukocytes and endothelial cells and are important for leukocyte-endothelial cell interactions. These include the immunoglobin supergene family (ICAM-1, VCAM-1, and PECAM-1), the selectins (E-selectin, P-selectin and L-selectin), and the integrins. The leukocyte b2 integrin adhesion molecule family consists of three members with heterodimeric glycoproteins displayed as a variable alpha and a constant beta chain. Nitric oxide regulates the adhesion process both by direct influence on leukocyte binding as well as by regulation of regional blood flow. IL-8 is one of the most potent mediators of chemotaxis in the C-X-C chemokine family. It serves an important role in neutrophil recruitment and activation, and the continued propagation of the inflammatory response.

172 A 65-year old patient has colon carcinoma metastatic to the liver and lungs. He has had a weight loss of 10 kg. Cytokine-dependent tumor cachexia is attributable to which of the following?
a. Increased glucose uptake and increased glycogen breakdown occur in this circumstance.
b. Suppressed activity of lipoprotein lipase results from TNFa
c. TNFa stimulates lipolysis
d. The differentiation process of pre-adipocytes is impaired
e. Partial reversal of differentiated adipocytes to pre-adipocyte morphology and gene expression occurs
Answer: a, b, c, d, e

Tumor cachexia appears to be mediated by TNFa. Lipopolysaccharide (LPS), as well as other cytokines, activate a variety of inflammatory cells, most importantly macrophages, to produce TNFa. Both the chronic administration of TNFa to rats and implantation of tumors secreting TNFa in mice induce a syndrome of cachexia. In vitro, higher TNFa concentrations alter glucose metabolism in cultured myotubules by increasing glucose uptake and glycogen breakdown. It has also been demonstrated that purified TNFa suppresses lipoprotein lipase activity and stimulates lipolysis in cultured adipocytes. Further, TNFa not only inhibits the differentiation process of preadipocytes, but partially reverses differentiated adipocytes to a preadipocyte morphology and pattern of gene expression. All of these metabolic effects at least partially explain the chronic syndromes of anorexia, weight loss, and cachexia that are associated with both chronic infection and malignancy.

173 Which of the following statements regarding fibroblasts and their function in wound healing are true?
a. IL-1 has both inhibitory and promotional effects on fibroblast growth
b. TNFa stimulates fibroblast collagen synthesis
c. IL-1 and TNFa have opposite effects on the healing of bone
d. In human clinical trials, EGF (epithelial growth factor) has been demonstrated to accelerate epidermal regeneration in cutaneous wounds
Answer: a, d

IL-1 appears to be important in the process of normal wound repair. IL-1 has been shown to stimulate skin fibroblast and keratinocyte growth, as well as fibroblast collagen synthesis and keratinocyte chemotaxis. IL-1 also promotes increased transcription of the matrix degradative enzymes collagenase and stromelysin. These are important and potent tissue degrading proteinases. Other studies have demonstrated that IL-1 inhibits fibroblast growth and matrix synthesis, and stimulates collagenase production. These actions are at least partly due to the ability of IL-1 to upregulate prostaglandin E2 production which results in the down regulation of matrix synthesis. IL-1 has both promoting and inhibiting effects on fibroblast collagen synthesis, therefore, the overall activity in this area is somewhat unclear in comparison to other well-defined fibroblast growth-promoting cytokines. TNFa inhibits fibroblast collagen synthesis, however it also has potent mitogenic effects. The mitogenic response correlates well with an increased stimulation of tyrosine phosphorylation. Both IL-1 and TNFa have similar effects upon bone. Both stimulate cartilage resorption, the release of proteoglycans from cartilage by limited proteolytic degradation, and both inhibit proteoglycan synthesis. Recent studies have also demonstrated that TNFa inhibits fracture healing in experimental animals. This is due to the inhibition of cartilage formation and new bone synthesis, and the inhibition of mesenchymal cell differentiation into chondroblasts. The family of epithelial growth factor (EGF)-like molecules induce mitogenesis and play a role in wound healing. In human clinical trials, EGF has been demonstrated to accelerate epidermal regeneration in cutaneous wounds. In vitro data show that recombinant EGF enhances keratinocyte migration. EGF is also a potent chemoattractant for granulation tissue fibroblasts.

174 Neutrophil chemotaxis is a fundamental aspect of inflammatory injury in conditions such as the Adult Respiratory Distress Syndrome (ARDS). Neutrophil chemotaxis is directly attributable to which of the following molecules?
a. C5a
b. TNFa
c. LPS
d. IL-1
e. ENA-78 (Epithelial Neutrophil Activating Protein)
Answer: a, e

There is a large collection of peptide, polypeptide and lipid mediators which have chemotactic properties. Although TNF a, IL-1 and LPS were initially reported to have direct neutrophil chemotactic activity, recent studies have demonstrated that these molecules are not directly chemotactic for neutrophils. This finding suggests that cytokine networks may be operative in vivo and depend on the initial expression of early response cytokines. This initial interaction is followed by the generation of more distal inflammatory mediators that directly influence neutrophil chemotaxis and activation. There is a particularly important group of novel chemotactic cytokines which share significant homology with the presence of four conserved cysteine amino acid residues. These cytokines in their monomeric forms are all less than 10 kD, are characteristically basic heparin-binding proteins, have specific neutrophil chemotactic activity and display four highly conserved cysteine amino acid residues, with the first two cysteines separated by one non-conserved amino acid residue. Because of their chemotactic properties and the presence of C-X-C cysteine motif, these have been designated the C-X-C chemokine family. Twelve different chemokines have been identified in the last decade. These include IL-8, epithelial neutrophil activating protein (ENA-78), and others. Among the other endogenous chemoattractants are several complement-derived peptides. Perhaps, the most potent of these is the short-lived C5a peptide.

175 Which of the following statements regarding angiogenesis are true?
a. Angiogenesis is a seminal biologic event with clinical relevance limited to its effect upon tumor growth
b. C-X-C chemokines regulate angiogenesis
c. PF-4 has angiogenic properties
d. Sites of atherosclerosis demonstrate chronic angiogenic activity
Answer: b, d

An important component of tissue repair and wound healing is the process of angiogenesis. This normal, physiologic process is a local, transient event which is regulated strictly. A biological imbalance in the production of angiogenic and angiostatic factors contributes to the pathogenesis of several angiogenesis-dependent disorders. These include both malignant and nonmalignant disorders such as rheumatoid arthritis, scleroderma, psoriasis, atherosclerosis, and idiopathic pulmonary fibrosis. Persistent neovascularization in these benign disorders is a prerequisite for the perpetuation of fibroproliferation. IL-8 and potentially other C-X-C chemokines are involved with the angiogenesis process. IL-8 is a potent angiogenic factor. In contrast, another member of the C-X-C chemokine family, PF-4 has angiostatic properties. This suggests that the C-X-C chemokines may function as either angiostatic or angiogenic factors, and the biologic balance that is maintained between these factors may govern overall angiogenic potential in a variety of physiological and pathophysiological states.

176 Which of the following statements regarding IL-1 are correct?
a. While IL-1 and TNFa share many biologic effects, IL-1 appears to be more potent
b. IL-1 expression is in part autoregulated
c. IL-1 inhibits prostaglandin production
d. The ability of IL-1 to upregulate endothelial cell-neutrophil adhesion molecules is relatively limited
Answer: b

IL-1 and TNFa share many biologic properties. In addition, each potentiates the effects of the other one when given concurrently. Overall, IL-1 alone probably has weaker effects than TNFa with respect to the induction of shock; its role is likely to be important with respect to its marked potentiating abilities as it relates to TNFa. IL-1 expression is regulated by a host of factors including IL-2, granulocyte macrophage colony stimulating factor (GM-CSF), transforming growth factor b (TGF-b), TNFa, all of the interferons, and IL-1 itself. Other endogenous stimuli for IL-1 production include antigen-antibody complex, the Fc region of IgG, and C5a; other nonspecific exogenous stimuli include silica particles and UV irradiation.
One of the key proinflammatory features of IL-1-induced inflammation is the stimulation of arachadonic acid metabolism. IL-1 stimulates the release of pituitary stress hormones and increases the synthesis of collagenases, resulting in the destruction of cartilage, bone and other collagen-rich structures. IL-1 stimulates prostaglandin production.
One of the most important properties of IL-1 involves its interaction with the vascular endothelium. This includes the adherence of neutrophils, basophils, eosinophils, monocytes, and lymphocytes to the vascular endothelium via interaction between adhesion molecules on leukocytes and adhesion-receptor complex on the endothelial cells. By inducing the expression of ICAM-1, E-selectin, and VCAM-1 on endothelial cells, IL-1 provides a key step in the extravasation of leukocytes to sites of local inflammation and injury.

177 Which of the following statements regarding TNFa are true?
a. TNFa has a marked procoagulant effect
b. Passive immunization of patients with neutralizing antibodies to TNFa improves survival from multi-organ system failure
c. TNFa upregulates E-selectin expression
d. The most potent known stimulus for TNFa production and release is IL-1
Answer: a, c

TNFa has a marked procoagulant effect on endothelial cells, precipitating intravascular thrombosis. TNFa causes endothelial cells to release procoagulant activity (tissue factor), platelet activating factor, and von Willebrand factor, all of which favor thrombosis. TNFa also down regulates the expression of thrombomodulin, which has the potential to block the assembly of protein C and protein S complexes, further decreasing the anticoagulant properties of the endothelial cell surfaces.
Administration of recombinant TNFa to experimental animals produces a clinical syndrome similar to that seen in septic shock and multi-organ system failure in humans. Passive immunization of animals with neutralizing antibodies against TNFa, prior to the infusion of TNFa or endotoxin, has been shown to prevent the development of this syndrome. No such evidence exists in human patients.
TNFa upregulates a variety of leukocytic adhesion molecules including ICAM-1, PECAM-1, VCAM-1, E-selectin and P-selectin. A variety of exogenous and endogenous factors (including IL-1) are capable of inducing cells to produce TNFa, however the most potent stimulus for TNFa production and release is endotoxin.

178 Which of the following belong to the family of C-X-C chemokines?
a. IL-8
b. IL-10
c. Growth Related Oncogene-a
d. Leukotreine B4
e. b Thromboglobulin
Answer: a, c, e

A particularly important group of novel chemotactic cytokines has been elucidated over the last decade. Twelve are known and are listed below.
C-X-C Chemokines
Connective Tissue Activating Protein III
b-Thromboglobulin
Growth Related Oncogene-a
Growth Related Oncogene-b
Related Oncogene-g
Interleukin-8
Epithelial Neutrophil Activating Protein
Granulocyte Chemotactic Protein-2
Platelet Factor-4
g-Interferon-inducible Protein
Monokine-induced by g-Interferon
Each has unique biologic functions. There appear to be important in vivo cytokine networks involving these molecules which regulate chemotaxis, and other fundamental aspects of inflammation.

179 Which of the following statements regarding the complement system are true?
a. Complement activation yields products which are directly cytotoxic as well as products which act indirectly via activated leukocytes
b. Complement products referred to as anaphylatoxins include C1, C3a, C4a, and C5a
c. The principal role of C5a is in bacterial opsonization
d. The alternative and classical pathways converge proximal to generating the membrane attack complex (C5b-9)
Answer: a, d

The complement system is composed of two different but linked sequences, the classic and alternative pathways. The pathways involve serum proteins that act to amplify the inflammatory-immune response as well as to directly mediate tissue injury. Complement activation by either pathway has been associated with a cascade of events, some of which are mediated directly at a physiologic level by complement products and some of which occur indirectly via activated leukocytes. The direct physiologic effects mediated by C3a and C5a, and to a lesser extent C4a, include increased vascular permeability and contraction of smooth muscle. These are key elements of anaphylaxis. C1 is not an anaphylatoxin as it is the initial complement component which binds to antigen-antibody complex to initiate classical pathway activation. C5a acts principally to alter the behavioral characteristics of leukocytes. Effects include enhanced adherence, enhanced chemotactic activity, release of proteinases, and production of toxic metabolites of oxygen. C3, on the other hand, plays a key role in bacterial opsonization, resulting in enhanced phagocytosis of invading microorganisms. The alternative and classical complement pathways converge at the C5 level proximal to generating the membrane attack complex (C5b-9) (Figure 6-3).

180 Which of the following statements regarding neutrophils are true?
a. The neutrophil undergoes final maturation after release into the circulation
b. Patients with chronic granulomatous disease have a defective neutrophil H-oxidase system
c. Neutrophil killing of bacteria is achieved by oxidants, proteinases and cationic proteins
d. The normal human neutrophil circulates in the blood for 7–10 days
Answer: b, c

The neutrophil is a migratory phagocytic cell that defends the host against bacteria and eliminates necrotic tissue. The neutrophil matures in the bone marrow and is released into the circulation as a fully differentiated cell. It is loaded with granules containing a variety of proteinases, hydrolases, antimicrobial agents and cationic proteins. The cell phagocytoses material and the granules fuse with the phagocytic vacuoles to degrade the foreign material. When the cells are challenged with a large amount of material, the granule contents may be released into the extracellular space where damage to surrounding tissue occurs. The neutrophil normally circulates in the human bloodstream for 7 to 10 hours. Thereafter, neutrophils are thought to exist for 1 to 2 days in the tissues before being cleared from the system. Granule constituents are formed during differentiation and replenishment of spent granules does not occur once the cells are in the circulation. Hence, the neutrophil is a fully differentiated end-cell poised to respond rapidly to stimuli, but it is rapidly spent in the process. Neutrophils have a NADPH-oxidase enzyme system on the plasma membrane which can be activated to produce toxic oxygen species including the superoxide anion (02–). Patients with chronic granulomatous disease (CGD) have a defective NADPH-oxidase system in their neutrophils, and are thus unable to generate 02–. Although neutrophils from patients with CGD are able to phagocytose bacteria, they are unable to kill the intracellular microbes and chronic, unresolved infections result.

181 Which of the following statements regarding the alternative complement pathway are true?
a. C1, C4 and C2 are involved
b. NH3 apparently activates complement via this pathway
c. Factors B and D are involved
d. Endotoxin activates complement via the alternative pathway
Answer: b, c, d

The alternative pathway differs from the classic pathway in that the first steps involving C1, C4 and C2 are bypassed. (See Figure 6-3 previously reproduced.) This pathway can be directly activated by agents other than antigen–antibody complex (e.g., complex polysaccharides like endotoxin and zymosan). Other serum protein factors (e.g., factors B and D) are involved in the activation sequence. Ammonia can attack the thiol-ester, producing amidated C3 and activate the alternative pathway. This leads to membrane attack complex formation (C5b-9) and activation of a number of phagocytic cell functions including toxic oxidant production. This phenomenon may have relevance to several in vivo disease states. In animal models of renal failure, elevated levels of renal vein NH3 have been correlated with impaired renal function and the presence of complement components at the sites of renal injury.

182 Platelet activating factor is:
a. Generated by the action of phospholipase A2 on membrane phospholipids
b. Antiinflammatory in most of its actions
c. Synthesized by endothelial and other cells
d. Exerts a variety of biologic effects which are platelet-independent
Answer: a, c, d

Like the eicosanoids, platelet-activating factor (PAF) is not stored in cells but is rapidly produced during inflammation. PAF exerts a variety of biologic effects that are platelet-independent. The synthesis of PAF is initiated by the activation of phospholipase A2. Activation of phospholipase A2 releases arachidonic acid in addition to lyso-PAF. Hence, PAF synthesis and eicosanoid production are coordinately regulated. PAF is synthesized on activation of a variety of inflammatory cells including platelets, neutrophils, basophils, mast cells, mononuclear phagocytes, eosinophils and vascular endothelium. PAF is a stimulatory agonist for many inflammatory cells, as well as for smooth muscle cells, vascular endothelium and others. PAF enhances the ability of neutrophils to respond to challenge with N-formylpeptides and LTB4. There is considerable overlap and redundancy in the effects produced by PAF and eicosanoids.

183 Platelets have a wide array of functions in inflammation. Which of the following are among these?
a. Synthesis and release of vasoactive eicosanoids
b. Release of chemotactic factors
c. Adherence to and coating of bacterial and tumor cells
d. Increase of vascular permeability
e. Phagocytosis of bacteria
Answer: a, b, c, d

Platelets are anucleated cells derived from megakaryocytes in the bone marrow. Their central role in hemostasis is well known. Platelets possess a wide array of functions in inflammation, including the following:
Synthesis and release of vasoactive eicosanoids
Release of chemotactic factors
Interaction with other inflammatory cells
Interaction with endothelial cells
Adherence to and coating of bacterial and tumor cells
Platelets are not capable of phagocytosis.
Few of the factors released or the functions carried by platelets during inflammation are unique to this cell type. Other inflammatory cells often have the same or similar capabilities. Indeed, some platelet functions may reflect vestigial functions inherited from a primitive precursor inflammatory cell. Platelets serve primarily as an amplifier or modulator of the inflammatory response.

184 Eicosanoids mediate inflammation in a variety of ways. Of the following statements, which are true with regard to this?
a. Eicosanoids are stored in cytoplasmic granules for release after receptor mediated signaling
b. Eicosanoids include prostaglandins, thromboxanes, leukotrienes and lipoxins
c. Eicosanoids generally have a plasma half-life measured in hours
d. Physiologic responses to eicosanoids include vasodilatation, vasoconstriction, increased vascular permeability and both chemotaxis and chemoattractant inhibition
Answer: b, d

The eicosanoids are derived from arachidonic acid (eicosatetraenoic acid) and consist of prostaglandins, thromboxanes, leukotrienes and lipoxins. The eicosanoids are not stored in cells but are rapidly synthesized by cells in response to a variety of stimuli. They have potent effects on vascular and bronchial smooth muscle including vasodilatation, vasoconstriction, bronchodilation and bronchoconstriction. In addition, they directly regulate vascular permeability. LTB4 is a potent, neutrophil chemoattractant whereas lipoxin A4 inhibits other chemoattractants. It appears that eicosanoids are important regulators of the endogenous inflammatory response. The rapid destruction of eicosanoids in the circulation limits their role primarily to that of mediators of local inflammatory changes. The local effects can be substantial. In general, the eicosanoids are rapidly metabolized or are so chemically unstable that they primarily exert their effects near the site of synthesis. Arachidonic acid does not exist in cells but is esterified to membrane phospholipids. Thus, the first step in the production of eicosanoids is phospholipase action, which liberates arachidonic acid. (Figure 6-6)

185 Which of the following statements are true?
a. Eosinophils are the major, if not sole, source of histamine in the blood
b. Basophils are effector cells in allergic reactions by virtue of IgE receptors
c. Mast cells are the major source of tissue histamine except in the stomach and central nervous system
d. Mononuclear phagocytes release a variety of proinflammatory cytokines and growth factors
Answer: b, c, d

Eosinophils constitute 1% to 3% of the leukocyte population of the bloodstream. They also reside in tissues and they exhibit phagocytic capabilities. They are less effective as bactericidal cells than neutrophils, but play a major role in defense against parasites. Eosinophils are primary effectors in allergic reactions by virtue of IgE receptors (which are not found on neutrophils).
Basophils are fully differentiated cells released into the circulation from bone marrow. Basophils are the major, if not sole, source of histamine in the blood. Histamine is a vasoactive amine and the major mediator of the IgE-mediated immediate hypersensitivity response. Histamine release from basophils is induced by complement products as well as by IgE receptors.
Mast cells are formed from bone marrow precursors that differentiate and proliferate in connective tissue. Mast cell granules contain histamine and proteoglycans. They represent the major source of histamine in most tissues except the stomach and central nervous system.
The monocyte–macrophage system consists of phagocytic cells scattered throughout the body. During acute inflammation, monocytes respond to chemoattractants released and are recruited to the site of inflammation. Mononuclear phagocytes respond to inflammatory stimuli by releasing M-CSF, GM-CSF, IL-1, and TNF, in addition to a variety of growth factors. These factors increase the production of mononuclear phagocytes and several of these factors enhance the ability of effector cells to respond to chemotactic stimuli released at the site of injury. Thus, the mononuclear phagocytes are important in initiating and augmenting the cycle of events that result in recruitment and activation of inflammatory cells at sites of inflammation.

186 Cellular injury from oxidants may be manifest by which of the following?
a. Cell membrane lipid peroxidation
b. DNA strand breaks
c. Cytoskeletal disassembly
d. ATP depletion
Answer: a, b, c, d

Free oxygen radicals are chemical species that are intermediates in the normal process of cellular respiration. Oxidants that are free radicals have been implicated as initiators of reactions which lead to a variety of cellular injuries. Oxidants are derived from several sources, notably phagocytes. Among the effects of oxygen free radicals are membrane lipid peroxidation, DNA strand breaks, cytoskeletal disassembly and inhibition of glucose metabolism leading to decreased cellular ATP concentrations. (Figure 6-16)

187 Which of the following acute-phase protein levels are increased in human plasma following acute inflammation?
a. C-reactive protein
b. Serum amyloid
c. a -Proteinase inhibitor
d. Fibrinogen
e. Albumin
Answer: a, b, c, d

The acute-phase response is a series of homeostatic responses of the organism to tissue injury in infection and inflammation. After an inflammatory stimulus occurs, a number of events occur within hours. These reflect altered set-points for various physiologic parameters including thermoregulation (fever), nitrogen balance (negative), and levels of various plasma proteins (increased or decreased). The erythrocyte sedimentation rate, which increases with inflammatory states, is an example of this phenomenon. The increased sedimentation rate is due to increased levels of fibrinogen and some of the other acute-phase reactants in plasma. Some proteins show a large increase (about 1000-fold), some a 4-to 5-fold increase, and others about a 50% increase over resting nonstressed levels.
Note that albumin is an acute-phase reactant. Levels of albumin drop after an inflammatory stimulus, usually 30% to 50% of the level before injury. The reason for the decrease in production is poorly understood.

188 Which of the following statements regarding endothelial cells in acute inflammation are true?
a. Endothelial cells are characterized by phenotypic homogeneity
b. Specific patterns of receptor expression regulate leukocyte adherence
c. Endothelial cell nitric oxide generation regulates regional blood flow and leukocyte adhesion
d. Endothelial cells may be capable of phagocytosis
Answer: b, c, d

Endothelial cells are increasingly recognized to be phenotypically heterogeneous. Specific receptor molecules are expressed at various sites where they help to direct lymphocytes and other leukocytes to their appropriate target organ. In the high endothelial venues, these receptor molecules are known as vascular addressing. Endothelial cells play a major role in regulating vascular tone. This is the result of angiotensin-converting enzyme on the cell surface as well as the production of both endothelia (a potent vasoconstrictor) and nitric oxide (a potent vasodilator). Both play important physiologic roles in determining the distribution of blood flow. In addition, recent evidence suggests that NO may have direct effects upon the expression of a variety of leukocyte adhesion molecules. Under unusual circumstances, endothelial cells can exhibit macrophage-like properties in that they can act as antigen-presenting cells and also phagocytose particles. They may also be a significant source of oxidants in inflammatory reactions after ischemic injury. Endothelial cells are not passive participants in inflammatory processes; rather, they possess the ability to direct and focus many aspects of an inflammatory event.

189 The first line of host defense is the barrier presented to the external environment. Which of the following statement(s) is/are true concerning host barriers?
a. Sebaceous glands secrete chemical compounds that maintain a relatively high pH, providing effective bacterial stasis
b. Within the respiratory tract, ciliary function serves to extrude microorganisms trapped within the mucus secretion layer
c. The low pH within the stomach markedly decreases bacterial content of the upper gastrointestinal tract
d. Gut peristalsis serves to prevent microbial adherence and invasion
Answer: b, c, d

The skin, mucus membranes, and epithelial layers of various organs of the body constitute effective physical barriers against microbial invasion. In certain portions of the body, these barriers have developed ancillary adaptations to increase the effectiveness of the barrier functions. Skin structures such as sebaceous glands secrete chemical compounds that serve to maintain a relatively low pH, providing effective bacterial stasis. Mucus secretion by specialized glands within the bronchi and gut provide a mucus layer that represents a physical and chemical barrier to microbial invasion. Within the respiratory tract, ciliary function serves to extrude microorganisms trapped within this mucus layer. In the alimentary track, the very low pH within the stomach and gut peristalsis both serve to prevent microbial adherence and invasion.


190 Which of the following statement(s) is/are true concerning the antibody response to an invading antigen?
a. All antibodies are composed of one type of heavy and one type of light protein chain
b. The carboxyl terminus of the heavy chain is the antigen binding site
c. Antibody of the immunoglobulin G class is the initial antibody produced in response to an antigenic stimulus
d. Immunoglobulins A, D, and E play an active role in the circulating humoral response
Answer: a

Humoral defenses consist of antibody (immunoglobulin; Ig) and complement. All Ig classes (IgM, IgG, IgA, IgE, IgD) and IgG subclasses are composed of one type (M, G, A, E, D) of heavy and one type (K and g ) of light protein chains that consist of several domains both structurally and functionally. Each Ig molecule contains one or more units that consist of two heavy and two light chains linked by disulfide bonds. The amino terminus of both heavy and light chains contain several hypervariable regions that fold in three dimensions to produce the antigen-binding site. The carboxyl terminus of the heavy chains contain regions that activate complement and bind Fc receptors, by which direct adherence to polymorphonuclear leukocytes and macrophages take place after antigen binding occurs.
Initially, antibody of the IgM class is produced in response to an antigenic stimulus. A second exposure to the same antigen, or a cross-reactive antigen, leads to the so-called second set response, in which antibody of the IgG class with two binding sites is produced more rapidly and in larger quantity compared to the initial IgM primary response. Immunoglobulin of the IgA class is secreted by gut-associated lymphoid tissue and is combined with secretory components of protein to form a dimer termed secretory IgA. This antibody acts at a variety of epithelial sites to prevent microbial adherence and invasion. IgD and IgE exist in smaller amounts in the circulation and do not appear to play a major role as host defense components.


191 Increasing evidence has implicated gram-negative bacterial lipopolysaccharide (LPS endotoxin) as the portion of the gram-negative bacterial cell membrane responsible for many, if not all the toxic effects that occur during gram-negative bacterial sepsis. The following statement(s) is/are true concerning LPS and the host response.
a. The LPS molecule can in itself cause physiologic responses similar to that seen during gram-negative bacterial sepsis
b. LPS triggers host macrophages to release a variety of cytokines including TNF-a, IL-1a, and IL-1b, IL-6, and IFN-a
c. Excessive cytokine production is not associated with detrimental consequences
d. TNFa and IL-1b appear to be the primary mediators within the host, exerting deleterious effects on the host when excessive amounts reach the systemic circulation
Answer: a, b, d

The LPS molecule exerts diverse effects on the mammalian host. Immunologic responses to LPS include nonspecific polyclonal B-cell proliferation, macrophage activation and cytokine secretion, tolerance to subsequent LPS or bacterial challenge, and production of antibody directed against various portions of the LPS molecule after repeated challenge. Physiologic responses similar to those seen during gram-negative bacterial sepsis occur during LPS administration alone and include hypotension, hypoxemia, acidosis, bacterial translocation across the gut, complement and coagulation cascade activation, white blood cell and platelet margination, and death. Indirect effects result from LPS-triggering of host macrophages. Activated macrophages secrete a wide array of cytokines that include TNF-a, IL-1a, and IL-1b, IL-6 and interferon-a (IFNa). Excessive secretion of cytokines produce substantial systemic effects in the mammalian host. TNFa and IL-1b appear to be the primary mediators within the local host milieu, exerting deleterious effects on the host only after large amounts are secreted and reach the systemic circulation.


192 Which of the following statement(s) concerning the gut microflora is/are correct?
a. Gut microflora evolves constantly throughout development
b. The gut microflora can contribute to the physical and chemical barriers at the mucus membrane level
c. Most of the microorganisms found in the oropharynx eventually pass into the intestine
d. In the colon, anaerobic organisms outnumber aerobic organisms in a ratio in excess of 100:1
Answer: b, d

The composition of the gut microflora is established in neonates after ingestion of microbes that are acquired during contamination from the birth canal and during initial feeding, and remain relatively constant thereafter. Although this flora acts to promote development of the immune system, the specific interactions that produce this effect have not been fully elucidated. The microflora also contributes to physical and chemical barriers at the mucus membrane level, in that many autochthonous microbes possess adhesion proteins by which they can bind to certain areas of the mucosal cell or to specific types of bacteria, occupying potential binding sites for pathogenic organisms and producing a substantial physical mucobacterial layer. The oropharynx contains a number of aerobic and anaerobic microorganisms, however, these microbial inhabitants do not usually pass into the intestine, because the stomach itself represents a significant barrier to invading microorganisms by virtue of its low pH, which kills most microbes. The upper small intestine contains few organisms, mainly gram-positive aerobes and lactobacilli. Conversely, the lower small intestine contains a large number of aerobes and anaerobic forms, especially in patients in whom the ileocecal valve allows free backwash of cecal contents into the terminal ileum. Within the colon, a wide diversity and a large number of facultative and strict anaerobic isolates are present. Only a small number of aerobes are present, these microbes being outnumbered 100–300 to 1 by anaerobes.


193 The use of antibiotics can be based on either the clinical course of a patient without the benefit of well-defined microbiologic data (empiric therapy), or targeted at specific identified pathogens once sensitivity reports are available (directed therapy). The following statement(s) is/are true concerning these therapies.
a. The issue of toxic side effects of antibiotics is only important in dealing with emperic therapy
b. Single agent therapy is generally inferior to specific multi-drug therapy (aminoglycoside plus an antianaerobic agent) for the treatment of secondary bacterial peritonitis due to appendicitis, diverticulitis, penetrating gastrointestinal injury, or anastomotic leak
c. With the empiric use of antibiotics, a diligent search for the septic source should be undertaken and continued until identified
d. In clinical situations in which polymicrobial infection is identified, specifically-directed treatment for the predominant organism is satisfactory
Answer: c

The use of empiric therapy without the benefit of well-defined microbiologic data is appropriate when there is sufficient clinical evidence to support the diagnosis such that it would be imprudent to withhold antimicrobial therapy. In this setting, however, a diligent search for the septic focus source should be undertaken and continued (cultures, radiographic procedures, etc.), and initial limits should be placed in the course of empiric therapy with continued reevaluation based on the clinical course of the patient. The choice of antibiotic agents should be based on the clinical situation and known activity patterns within the given institution. Single broad-spectrum agents, although suffering slightly from a lack of individual pathogen specificity, are useful in this setting in that they provide a broad coverage against several groups of pathogens and may avoid some of the toxic effects with specific combined modality regimens. Similarly, for directed therapy, single-agent therapy has been demonstrated to be equivalent to combined therapy and should be chosen in an attempt to select agents with appropriate sensitivities which retain suitable clinical efficacy but exhibit minimal toxicity. After review of cultural reports, many patients have demonstrated polymicrobial infection. Because experimental clinical evidence supports the concept of aerobic-anaerobic synergy, therapy should be directed against all potential components of the infection if the body site is such that these microorganisms may be present.


194 The following statement(s) is/are true concerning newer detection methods of systemic infection.
a. Enzyme-linked immunosorbent assay (ELISA) is a rapid immunologic assay used for both antigen and antibody detection
b. Southern, northern, and western immunoblot techniques are used to detect DNA, RNA, or proteins, respectively
c. Polymerase chain reaction (PCR) is a sensitive assay used to detect small amounts of microbial DNA, thus detecting infection at its early stages
d. Infectious agents currently detected using advanced molecular techniques include cytomegalovirus (CMV) and human immunodeficiency virus (HIV)
Answer: a, b, c, d

Although the classic detection of infection based on clinical signs of infection and bacterial culture remain the most common clinical tools, increasing reliance has been placed on assays that do not employ cultural data. Specifically, the antibody and cytokine host responses are being intensely examined and extremely sensitive amplified assays that rely on antigen, antibody or microbial DNA detection are employed in the clinical setting. Enzyme-linked immunosorbent assay (ELISA) is a rapid, antigen-based, immunologic assay that can be used for both antigen and antibody detection, for determination of antibody titer, as well as for screening for monoclonal antibody production. Transblot techniques are being used increasingly in the clinical setting. These include southern, northern, and western immunotransblot techniques used to detect DNA, RNA, or proteins, respectively. The polymerase chain reaction (PCR) is being used in some centers as a sensitive assay to detect small amounts of microbial DNA. This technique involves extraction of the DNA from the test sample with in vitro amplification through repeated nucleic acid denaturing and polymerization so that the gene copy number increases exponentially. This marked amplification of the gene copy number results in extremely sensitive tests which can detect infection at its early stages.
Clinically, these detection methods are being used to detect a wide variety of infectious agents including CMV and HIV. Furthermore, preliminary investigations into possible detection of fungal pathogens are underway.


195 Cytokines are low-molecular-weight polypeptides exerting a wide variety of biologic effects at both local and systemic levels. Which of the following statement(s) is/are true concerning the production and actions of cytokines?
a. Cytokines are produced solely by macrophages
b. Cytokines act only on other cells within the same local environment
c. Cytokines may have both protective and deleterious effects on the host
d. Each specific cytokine is produced by a single cell type
Answer: c

Macrophages, endothelial cells, lymphocytes, and other cells secrete a large number of different compounds, termed cytokines, that are most probably evolved for the purpose of local intercellular and intracellular signaling. Cytokines frequently are secreted after initial lymphocyte or macrophage activation, and may act on the secreting cell itself (autocrine activation) or on other cells within the same local environment (paracrine activation) to cause increased secretion of the same cytokine or other cytokines, respectively. Some cytokines are produced by several cell types, and most produce a wide array of effects. The duality of the effects of the cytokine component of host defenses, exerting both salutatory and deleterious effects on the host, has become increasingly evident.


196 The following statement(s) is/are true concerning cellular defense mechanisms.
a. Macrophages function solely as antigen processing cells in the initial reaction to exposure to an antigen
b. Macrophages may become activated and secrete cytokines
c. Macrophages serve as phagocytic cells in the tissues but not within the bloodstream
d. Polymorphonucleocytes (PMNS) are normally present in only small numbers within the tissue and enter an area of infection through diapedesis
Answer: b, d

A wide variety of cell types serve to provide host defense at several levels. Macrophages act as the initial antigen processing cell that serves to present antigen to help T cells, thus initiating the immune response. Macrophages, however, are pluripotent cells that, in the process of engulfing and processing antigen, may become activated. Activated macrophages secrete a variety of cytokines. Macrophages also act as phagocytic cells in the tissues and within the bloodstream, and because of their resident nature in many tissues, also represent the first line of host defenses in many areas of the body, even before activation. PMNS are present within the bloodstream, but only in small numbers within the tissue, and enter an area of infection through diapedesis after chemotactic stimuli are excluded by macrophages, bacterial breakdown products, and complement activation.


197 A diabetic develops a severe perineal infection with skin necrosis, subcutaneous crepitance, and drainage of a thin, watery, grayish and foul-smelling fluid. Management should consist of:
a. Gram stain of the fluid, which will likely demonstrate multiple bacteria including predominantly gram-positive rods
b. A CT scan is indicated in a stable patient to define the extent of the disease
c. Broad spectrum antibiotics followed with prompt extensive debridement is indicated
d. A safe guideline is to resect infected necrotic tissue so that a several centimeter margin of grossly normal, healthy tissue can be achieved
e. A colostomy is of little benefit in this situation
Answer: a, b, c, d

The presence of severe perineal infection (referred to as Fournier gangrene when this process involves the perineum and scrotum in males) is associated with a continued high mortality despite aggressive and appropriate therapy. The clinical description provided would suggest an underlying soft tissue necrosis. In a stable patient radiologic studies including a CT scan to define the extent of the disease and the presence of pelvic infection is indicated. Gram stain will likely show evidence of polymicrobial organisms but the presence of Clostridia marked by gram-positive rods would suggest involvement with this organism. Prompt, aggressive and extensive debridement to remove all devitalized and affected tissue and the addition of broad spectrum antibiotics, fluid resuscitation, hemodynamic monitoring, and nutritional support would appear to afford the patient the best chance of survival. The clearest guidelines to determine the limits of resection involve removal of clearly infected, necrotic tissue so that margins several centimeters into grossly normal, healthy tissue are achieved. Because the entire perineal region and buttocks are frequently involved in these patients, performance of a fecal stream diversion by means of a colostomy often provides improved wound care and patient management, although it is not invariably a positive outcome.


198 The use of prophylactic antibiotics has become commonplace. Which of the following statement(s) is/are true concerning the prophylactic use of antibiotics?
a. The appropriate use of prophylactic antibiotics must include the initiation of the agent prior to the surgical procedure
b. Continuing the antibiotic into the postoperative period has led to improved results in antibiotic prophylaxis
c. The prophylactic administration of broad-spectrum agents (third-generation cephalosporins) has been shown to be particularly advantageous
d. The topical use of antimicrobial agents is of no advantage in the prophylactic setting
Answer: a

Intravenous administration of an antibiotic is clearly indicated for patients undergoing clean contaminated operations. These antibiotics should be administered prior to surgery to obtain adequate tissue levels at the time of potential contamination. However, there has been no added benefit demonstrated for the postoperative use of antibiotics with regard to prophylaxis. The choice of antibiotic is a complex issue which remains unresolved largely because both superficial and deep wound infections can occur as a result of either or both skin (superficial wound) flora (e.g., Staphylococcus aureus) and body site (deep wound) infection. For this reason, the administration of agents which possess activity directed against these expected agents is reasonable. Although administration of a first-generation cephalosporin is acceptable, second-generation cephalosporins or extended-spectrum penicillins with gram-positive and gram-negative activity and biliary tract excretion may be more suitable for patients undergoing gastrointestinal or biliary tract procedures. Similarly, the use of agents with additional anaerobic activity for patients undergoing gastrointestinal procedures involving the small bowel or colon should be considered. The administration of broad-spectrum agents such as third-generation cephalosporins for prophylaxis does not seem to provide additional benefit in comparison to the above-mentioned type antibiotics and may foster the development of resistant organisms within a given institution or superinfection within a given patient. There is evidence that in some cases the topical use of antimicrobial agents is equivalent to the administration of intravenous antimicrobial agent antibiotics.


199 If a necrotizing soft tissue infection is considered, therapy mandates:
a. Empiric administration of antibiotics active against gram-positive, gram-negative, and anaerobic bacteria
b. Due to usually resistant species, penicillin is not indicated
c. Immediate operative intervention and aggressive resection of all involved tissues is mandatory
d. The use of hyperbaric oxygen has been demonstrated to be clearly advantageous
Answer: a, c

Identification of a necrotizing, soft tissue infection mandates immediate operative intervention with aggressive resection of all involved tissues and empiric administration of antibiotics active against gram-positive, gram-negative, and anaerobic bacteria. In most cases, this involves the use of several antimicrobial antibiotics in combination. Because of concern in all cases for the presence of Clostridia infection, high doses of aqueous penicillin G are administered. Gram-positive organisms are treated with vancomycin or a semisynthetic penicillin and gram-negative organisms are treated with an aminoglycoside or a monobactam. Anaerobic coverage is typically achieved by use of metronidazole of clindamycin. The use of hyperbaric oxygen therapy is controversial and unfortunately due to the rarity of the disease, prospective randomized data is not available so that the literature remains without controlled trials demonstrating any additional benefits derived from hyperbaric oxygen therapy.


200 Wounds are classified according to the likelihood of bacterial contamination. Which of the following statement(s) is/are true concerning wound classifications?
a. A clean-contaminated wound would be that associated with an elective colon resection with adequate mechanical and antibiotic bowel preparation
b. A contaminated wound would include a resection of obstructed bowel with gross spillage of intestinal contents
c. In a clean wound, no viscus is entered
d. Antibiotic prophylaxis should be administered for all clean-contaminated and contaminated wounds and selectively in patients involving a clean wound
Answer: a, b, c, d

Wounds are classified under three classes according to the likelihood of bacterial contamination: 1) clean (no viscus is entered; e.g., herniorrhaphy); 2) clean-contaminated (minimal contamination; e.g., elective colon resection with adequate mechanical and antibiotic bowel preparation, and 3) contaminated (heavily contaminated surgery; e.g., resection of unprepared, obstructed bowel with gross spillage of intestinal contents or stool, drainage of abscesses, debridement of traumatic neglected wounds). Antibiotic prophylaxis generally should be administered for class 2 and 3 types of wounds, but patients undergoing clean surgery do not always require antimicrobial antibiotic prophylaxis. An exception to this tenet involves cases in which a prosthetic material may be used (artificial joint, heart valve, tissue patch).


201 The following statement(s) is/are true concerning HIV infection.
a. Initial screening with ELISA is highly sensitive but can be associated with a false positive rate of 25%
b. Treatment with azidothymidine (AZD) appears to prolong survival when administered early in the disease
c. Predisposition to infection in HIV infection is primarily due to reduction in the number of helper T cells
d. Common infections in patients with AIDS are Pneumocystis, carinii pneumonia, CMV pneumonitis, Cryptococcus meningitis, and disseminated infection due to atypical mycobacteria
Answer: b, c, d

Acquired immunodeficiency syndrome (AIDS) is a syndrome caused by the human retrovirus (HIV-1) that infects T lymphocytes and causes severe immunosuppression. Individuals who become infected with HIV are prone to a variety of infections and different types of malignancy. A spectrum exists in which patients regress from asymptomatic infection, to development of AIDS-related complex (ARC) of diseases to AIDS itself. Common infections occurring in patients with AIDS are Pneumocystis carinii pneumonia; CMV pneumonitis; gastritis, hepatitis and meningitis due to Cryptococcus neoformans; and pneumonia and disseminated infection due to atypical mycobacteria. Predisposition to these infections is due, in part, to the lymphotrophic nature of HIV, which markedly reduces the number of helper T cells as well as the absolute number of T cells.
HIV detection typically consists of initial ELISA screening, but this test has about a 1–3% false-positive rate, thus mandating all positive tests be confirmed by the western immunoblot analysis.
Treatment of ARC and AIDS consists of aggressive antiinfective therapy once a specific infection occurs and the use of AZT. AZT has been shown to prolong survival when administered early in the course of disease and is considered routine therapy.


202 The following statement(s) is/are true concerning initial microbiologic diagnostic techniques.
a. Appropriate expeditious transport of specimens to the microbiology laboratory is essential for obtaining accurate clinical information
b. The use of potassium hydroxide in preparing a specimen slide for light microscopy will be useful in identification of anaerobic bacteria
c. Antibiotic sensitivity is determined by exposing the specific microorganism to varying amounts of antibiotic with the concentration of the antibiotic inhibiting growth referred to as the MIC (minimal inhibitory concentration)
d. Serum levels of antimicrobial agents should achieve in excess of a 4-to 8-fold increase over the MIC to be considered clinically efficacious.
Answer: a, c, d

Because most surgical infections are polymicrobial, specimens should be cultured for aerobic and anaerobic bacteria, as well as fungi. Although aerobic and aerotolerant microorganisms often do not require special transport media, a delay in specimen processing may markedly reduce the yield, and anaerobic transport media have been demonstrated to markedly increase the cultural yield of this type of organism. The initial piece of information gained concerning potential infection may come from simple staining of a specimen. Gram stain, which will identify the staining characteristics of the organisms, as well as their number should be performed on all specimens. Potassium hydroxide is useful in that it will lyse bacteria and other cellular elements within a preparation and allow observation of yeast or mycelial elements.
Initial culture results may solely indicate that microorganisms are growing and full characterization may take two to three days. Once a specific microorganism is identified, a sample is inoculated during the log phase into broth containing varying amounts of an antibiotic. After an 18-to 24-hour period, the tube or well that exhibits no visible growth is then noted, and the reciprocal of this dilution is termed the minimal inhibitory concentration (MIC). This value may be compared to either measured or known achievable serum levels for a particular antibiotic. In general, antimicrobial agents that achieve in excess of a 4-to 8-fold increase over MIC during the peak serum level have been demonstrated to be clinically efficacious.


203 The complement system consists of a series of serum proteins that exist in a quiescent or very low-level state of activation in the uninfected host. Which of the following statement(s) is/are true concerning complement activation?
a. The alternate (properdin) pathway of complement activation can occur directly through contact with fungal or bacterial cell wall compounds
b. Complement component fragments may serve to decrease vascular permeability
c. Excessive complement activation can produce deleterious effects
d. Fragments of certain complement components serve as chemoattractants to additional cellular components of the host defense mechanism
Answer: a, c, d

Complement activation can occur through either classic or alternate (properdin) pathways, both of which eventuate in deposition of terminal complement pathway components on the antigenic cell surface. The classic pathway of complement activation usually begins with immunoglobulin G-binding which has also bound the antigen. The alternate pathway activation occurs in response to activation of direct binding of the antigen or directly through contact with fungal and bacterial cell wall compounds such as zymosan and gram-negative bacterial lipopolysaccharide (LPS endotoxin). Several complement components represent important host defenses acting to recruit or augment cellular host defenses or to directly inactivate invading microbes through lytic activity. The production of complement component fractions C3a and C5a during activation of this cascade serve primarily to markedly increase vascular permeability, and C5a functions as a PMN and macrophage chemoattractant. This process leads to the recruitment of additional humoral and cellular defenses to the specific area of infection. Excessive complement activation can produce deleterious effects in some instances. Complement activation causes enhanced PMN adhesion, margination, and release of lysosomal enzymes that can directly damage certain target tissues, such as the lung.


204 A 55-year-old renal transplant patient has been hospitalized in a Surgical Intensive Care Unit, receiving a prolonged course of antibiotics following an attack of acute cholecystitis. The following statement(s) is/are true concerning his management.
a. Due to the potential risk of Candida infection, prophylaxis with oral nystatin should be instituted early in the patient’s course
b. A Candida urinary tract infection should be treated with systemic amphotericin B
c. Changes of Candida retinitis are of little significance
d. The presence of a virulent Candida bacteremia should suggest a dosage reduction in immunosuppressive agents until the infection can be adequately controlled
Answer: a, d

Infections due to fungal pathogens have become increasingly common during the past decade, frequently occurring in patients undergoing prolonged hospitalization in the Surgical Intensive Care Unit and in immunocompromised individuals. Prophylaxis with oral antifungal agents (nystatin) is warranted, especially during periods of maximal immunosuppression in transplant patients, in patients with uncontrolled diabetes, or during some cases of prolonged antibacterial microbial therapy. In general, local, apparently noninvasive Candida infections involving the integument and mucus membranes are treated with oral decontamination and topical antifungal therapy using topical agents such as nystatin. Candida urinary tract infections can be treated with either an oral antifungal agent or with topical amphotericin B as a continuous bladder irrigation. Several studies have demonstrated that those patients with three positive sites of Candida infection, or with peritoneal or blood cultures positive for Candida exhibit higher survival rates when amphotericin B therapy is instituted earlier in the course of infection. The presence of retinal changes compatible with Candida retinitis or Candida present within the peritoneal cavity are considered indications for a limited course of amphotericin B therapy (300% to 500 mg). Patients receiving exogenous immunosuppressive agents should undergo a marked dose reduction, and some agents should be discontinued until evidence of infection is absolutely controlled or is eradicated.


205 The initiation of a humoral immune response involves a complex interaction of the antigen, cells and intercellular messengers. Which of the following statement(s) concerning the initiation of the humoral immune response is/are correct?
a. Helper T lymphocytes stimulate B lymphocytes through secretion of cytokines such as interleukin 4 and 6
b. A number of cells can aid in presenting the antigen to the helper T cell including B lymphocytes and macrophages
c. All antigens require coordinated efforts of the various cellular components of the immune system
d. An antigen must be a living microorganism
Answer: a, b

Stimulation of the immune system occurs after a variety of antigen-presenting cells (B lymphocytes, macrophages, dendritic cells, and Langerhans cells) act to engulf, process, and present antigen to T lymphocytes of helper lineage. These T lymphocytes, in turn, act to stimulate B lymphocytes to become mature plasmacytes (through secretion of cytokines such as interleukin 4 and 6) dedicated to the production of antibody directed against the specific antigen. An antigen may be defined as any substance that stimulates the host immune response; that is, that the host immune system recognizes is foreign. Thus, an antigen may be an invading microorganism, an inert particle, or any type of chemical compound that triggers the host immune system. Although some antigens are able to directly stimulate B lymphocytes in and of themselves to produce antibody (many polysaccharides), most antigens require coordinated efforts of the various cellular components of the immune system.


206 The following statement(s) is/are true concerning viral infections.
a. The most common post-transplantation viral infections are caused by herpes viruses and include CMV and herpes simplex virus
b. Viral infections occur at equal frequency anytime during the post-transplantation period
c. CMV infection in the post-transplant patient is most likely a pulmonary process
d. Herpes simplex virus (HSV) infection primarily presents with a mononucleosis-type syndrome with fever, lethargy, and cough
Answer: a, c,

Solid organ transplant patients are prone to develop viral infection by virtue of exogenous immunosuppression. The most common post-transplantation viral infections are those caused by herpes viruses (CMV, herpes simplex virus [HSV], Epstein-Barr virus [EBV], and Varicella-Zoster virus [VZV]). All are most common during periods of maximal host immunosuppression that occur immediately post-transplantation and during periods of allograft rejection. CMV is a common cause of fever after solid organ transplantation, and evidence of CMV infection occurs in approximately 30% of patients. The most common presentation for CMV infection is that of a febrile, leukopenic patient with a cough, diffuse interstitial infiltrates on chest x-ray, and hypoxia.
HSV infection causes primarily oral pharyngeal ulcerations in most cases, although sporadic cases of disseminated disease have been reported. EBV causes an occasional case of mononucleosis-type syndrome but has also been clearly indicated in the pathogenesis of post-transplantation lymphomas. VZV infection can present as disseminated and occasionally life-threatening infections in the nonimmune transplant patient or as painful herpes zoster in patients who have previously developed chicken pox.


207 The following statement(s) is/are true concerning necrotizing fascitis.
a. Mortality rates as high as 40% can be expected
b. The infection involves only the superficial fascia, sparing the deep muscular fascia
c. An impaired immune system is a common factor predisposing to this condition
d. The infection is usually polymicrobial
e. Necrotizing fascitis is most likely to develop in the face of impaired fascial blood supply
Answer: a, c, d, e

Necrotizing fascitis is an uncommon infection of the deep and superficial fascia that is associated with mortality as high as 40% in many series. Although many underlying disease processes predispose patients to necrotizing fascitis, three common factors are almost invariably present: 1) impairment of the immune system; 2) compromise of fascial blood supply, and 3) the presence of microorganisms that are able to proliferate within this environment. Infections of this type are usually polymicrobial in nature, with gram-positive organisms such as staphylococci and streptococci, gram-negative enteric bacteria, and gram-negative anaerobic being frequently identified. These polymicrobial cultural results are assuredly indicative of the occurrence of a synergistic process, perhaps in large part accounting for the severity of these infections. Some microorganisms possess virulence factors that, in conjunction with an underlying host predisposition, allow this disease process to occur without dependence on other bacteria. Examples of such bacteria include Clostridium, Pseudomonas, and Aeromonas. In these patients, the process is often fulminant and is frequently associated with cellulitis, myositis, fascitis, and bacteremia with attendant high mortality.


208 New treatment modalities designed to modulate host defense mechanisms that have been demonstrated conclusively to be of benefit include:
a. Gut decontamination
b. Anti-LPS antibody
c. Anti-TNF antibody
d. Thymopentin
e. None of the above
Answer: e

Selective gut decontamination involves the use of orally administered antibiotics that achieve a high intraluminal level directed against gram-negative aerobes and yeast, leaving the host anaerobic intestinal microflora relatively undisrupted. Although a reduction and alteration of the microorganisms responsible for infectious episodes have been demonstrated in certain groups of patients, a clear-cut impact on host mortality has not been shown. Because LPS may be responsible for toxicity both directly and through host mediator systems, the availability of agents to bind against this portion of the gram-negative bacteria to reduce mortality has been intensively examined. Unfortunately, large multicenter randomized trials provide no evidence of benefit for this treatment. Similarly, since many of the systemic manifestations of gram-negative bacteremia are mediated by cytokines, the effect of an anti-TNF antibody preparation is currently in clinical trial. No proven benefits have yet been identified. Finally, the use of immunostimulants to enhance the state of activation of host defenses has been proposed. Thymopentin is a peptide that contains active thymopoetin, a thymic molecule that acts to stimulate T-lymphocyte activity. Preliminary trials indicate that this agent ameliorates host septic response after major operations and trauma but conclusive evidence that concurrent reduction of infection-related mortality occurs is not available.


209 Antibacterial agents can be classified with regard to their structure, mechanism of action, and activity pattern against various types of bacterial pathogens. Which of the following statement(s) is/are true concerning antimicrobial classes?
a. Penicillins and cephalosporins share the compound structure of a b-lactam ring which binds to bacterial division plate proteins
b. Tetracyclines and macrolides such as erythromycin inhibit bacterial ribosomal activity and therefore protein synthesis
c. Aminoglycosides act in a similar fashion to tetracyclines and therefore are both bacteriostatic
d. Sulfonamides and trimethoprim act synergistically to inhibit purine synthesis
Answer: a, b, d

Penicillins, cephalosporins, and monobactams possess a b-lactam ring of some type and act to bind bacterial division plate proteins, thus inhibiting cell wall peptidoglycan synthesis and either causing or inducing autolytic bacteriolysis. Because gram-positive and gram-negative bacteria possess different types of division plate proteins, many of these agents exhibit differential activity between these two types of microorganisms. Tetracyclines, chloramphenicol, and macrolides inhibit bacterial ribosomal activity, and thus overall protein synthesis by a variety of different mechanisms. Aminoglycosides act to inhibit protein synthesis and also presumably act on a different target site, a supposition based on the fact that aminoglycosides are bacteriolytic and the other agents are bacteriostatic. Vancomycin inhibits assembly of peptidoglycan polymers, whereas quinolones bind to DNA helicase proteins and inhibit bacterial DNA synthesis. Sulfonamides and trimethoprim act in different mechanisms to inhibit protein synthesis, therefore two agents in combination act synergistically.


210 The treatment of the following patient should include:
a. Initial empiric therapy directed against both aerobes and anaerobes
b. The addition of anti-fungal therapy in an elderly patient
c. A minimum of two weeks of antibiotic therapy is indicated
d. The addition of appropriate antibiotic therapy has made surgical therapy unnecessary in such cases
e. Either a single agent or combination therapy is appropriate if the agents selected possess activity against both aerobic and anaerobic bacteria
Answer: a, e

The primary treatment for a perforated viscus is surgical, however antimicrobial therapy is an extremely important adjunct. Empiric antibiotic therapy for secondary bacterial peritonitis and intraabdominal abscess should be directed against both aerobes and anaerobes. Administration of an agent directed against only one component of the infection or the other is inferior to combined therapy. Several studies indicate that the results of using several agents in combination is equivalent to the use of a single agent therapy as long as the agents selected possess activity against both components of the infection. The addition of antientercoccal or antifungal agents as initial therapy has not been substantiated. The most beneficial duration of antibiotic therapy must be based on the setting for the specific patient. Minimal peritoneal contamination with adequate surgical treatment may be treated with a three-to five-day course of antibiotics, whereas longer periods are indicated for immunosuppressed patients and with patients with extensive contamination.


211 The following statement(s) is/are true concerning host defense mechanisms to intraabdominal infection.
a. Bacterial clearance can occur via translymphatic absorption
b. Phagocytic activity and bacterial killing can occur via resident phagocytic cells and an influx of PMNs
c. A fibrinogen-rich inflammatory exudate is released into the peritoneal cavity, trapping large numbers of bacteria and other particulate matter
d. Perforations of a bowel may be walled off but are seldom sealed by the omentum and other mobile viscera
Answer: a, b, c

The introduction of microorganisms into the normally sterile peritoneal environment invoke several potent specialized host antimicrobial defense mechanisms. Bacterial clearance, also termed translymphatic absorption, occurs through specialized structures found only on the peritoneal mesothelium on the underside of the diaphragm that act as conduits for both fluid and particulate matter. Lymphatic channels eventually form which drain into the venous circulation via the thoracic duct. Bacteria not cleared via translymphatic absorption are rapidly engulfed by resident and recruited phagocytic cells including resident macrophages on the peritoneal surface and omentum and attracted PMNs. The final primitive host defense mechanism is sequestration by which a fibrinogen-rich exudate containing plasma oposonins appears during peritoneal infection and fibrin polymerization occurs. Fibrin has the capacity to trap large numbers of bacteria and other particulate matter. Acting in conjunction with omentum and other mobile viscera, perforations are sealed and the contaminated enteric contents walled off, preventing continued soilage of the peritoneal cavity.


212 A 67-year-old male presents with an intraabdominal abscess secondary to perforated sigmoid diverticulitis. The following statement(s) is/are true concerning his intraabdominal abscess.
a. Culture will likely reveal a solitary organism
b. Both aerobic and anaerobic islets are encountered in 50% of specimens
c. The most common aerobic islet will be likely E. coli and other gram-negative enteric bacilli
d. The most common anaerobic islet will be a Bacteroides species
Answer: b, c, d

Typically an intraabdominal infection results in perforation of a hollow viscus and the ensuing contamination of a normally sterile peritoneal cavity. The normal bacterial flora found in that particular location of the alimentary tract thus determines the initial inoculum. In parallel with the overall quantity of microorganisms, (both aerobes but predominantly anaerobes) perforations of the lower small bowel and colon produce a high frequency of infections that contain anaerobic microorganisms. Certain predictable patterns of bacterial islets are found, but on average four to five islets occur in patients with established intraabdominal infection, more than half of which are anaerobes. Both aerobes and anaerobes are encountered in 80% to 90% of specimens. Commonly encountered aerobes isolated are E. coli and other gram-negative enteric bacilli such as Enterobacter, Klebsiella. Among the anaerobes, Bacteroides species (especially B. fragilis, Clostridium), and anaerobic cocci are most consistently isolated.


213 The following statement(s) is/are true concerning gram-negative bacterial sepsis.
a. Mortality due to this condition has almost been eliminated due to therapeutic intervention with antibiotics, aggressive hemodynamic monitoring and fluid resuscitation
b. Recent series have noted a decrease in the incidence of this condition
c. Predisposing factors include old age, malnutrition, and immunosuppression
d. Pseudomonas bacteremia is the most common cause of gram-negative sepsis
e. Polymicrobial sepsis is generally considered a more serious problem than sepsis due to a single organism
Answer: c, e

Gram-negative bacterial sepsis is a serious disease process that produces substantial morbidity and mortality in both normal and immunocompromised patients (10% to 20% and 30% lethality, respectively), despite therapeutic intervention with antimicrobial agents, aggressive hemodynamic monitoring, fluid resuscitation, and metabolic support. During the past several decades, nosocomial infections due to gram-negative pathogens have increased in frequency with resultant increase in the incidence of gram-negative bacteremia to between 3 and 13 cases per 1000 hospital admissions. Factors that predispose to these infections include: 1) underlying host disease processes such as malignancy, diabetes; 2) old age and disability; 3) malnutrition; 4) previous or concurrent antimicrobial antibiotic therapy; 5) major operations; 6) respiratory or urinary manipulation or intubation; and 7) immunosuppression.
Although many different organisms cause this form of sepsis, E. coli predominates in overall frequency. Also common are isolates of Klebsiella, Enterobacter and Serratia; Pseudomonas bacteremia is less common. Some studies, however, have suggested that Pseudomonas sepsis is associated with the highest lethality. In several series, 10% to 20% of patients have had polymicrobial series, and most investigators agree that polymicrobial sepsis is more lethal than infection with a single organism.

214 Which of the following statement(s) is/are true concerning the various types of shock?
a. Traumatic shock is more commonly associated with subsequent organ injury and multiorgan failure syndrome than hemorrhagic shock
b. Cardiogenic shock can be of either an intrinsic or compressive nature
c. Hypodynamic septic shock is associated with a decreased mortality risk when compared with hyperdynamic septic shock
d. Hypoadrenal shock usually responds quickly to resuscitation
e. Neurogenic shock occurs with the absence of sympathetic activity
Answer: a, b, d, e

Classification schemes of shock based on cause have been developed for the seemingly dissimilar processes leading to circulatory collapse and the shock state. Hypovolemic shock, the most common, is the result of intravascular volume depletion through loss of red blood cell mass or plasma volume. Microvascular hypotension results from a combination of low intravascular blood volume, diminished cardiac output, and compensatory sympathetic peripheral vasoconstriction. Shock associated with trauma (traumatic shock) arises from the consequences of hypovolemia due to hemorrhage in conjunction with direct soft tissue injury and bone fracture. Hypovolemia caused by blood loss and fluid extravasation into injured tissues is compounded by activation of maladaptive inflammatory cascades initiated by the tissue injury. In contrast to pure hemorragic shock, subsequent organ injury and multiorgan failure syndrome (MOFS) occurs much more frequently following traumatic shock due to the over-expression of these immuno-inflammatory cascades. Cardiogenic shock is the result of failure of the heart as an effective pump, resulting in inadequate cardiac output, tissue perfusion and oxygen delivery. Intrinsic causes include myocardial infarction, cardiomyopathy, valvular heart disease, or rhythm disturbances. Compressive cardiogenic shock is a discrete entity that results when extrinsic compression of the heart limits diastolic filling and thus systolic ejection and cardiac output. Septic shock refers to hypotension and circulatory insufficiency developing as a consequence of infection and the systemic response to that infection. In its hyperdynamic form, septic shock is marked by diminished peripheral vascular resistance and generalized vasodilatation causing relative hypovolemia. In contrast, hypodynamic septic shock occurs in situations of inadequate resuscitation or preterminal cardiovascular decompensation, and is associated with vasoconstriction and a greatly increased mortality risk. Sympathetic denervation through spinal cord injury, spinal anesthesia, or severe head injury produces generalized arterial vasodilatation and venodilation. Shock occurs when the normal blood volume fails to fill the available intravascular space and severe relative hypovolemia exists. Despite hypotension, there is a noteworthy absence of sympathetic activity, as occurs in hypovolemia or cardiogenic shock. Profound shock can occur in surgical patients following stress due to the loss of the homeostatic corticosteroid response. Hemodynamic instability may develop after an operative procedure or coincident with an unrelated illness. The profound circulatory collapse is often refractory to vigorous resuscitation with fluids and pressor agents. The response to exogenous corticosteroids is usually dramatic and potentially life-saving.


215 Which of the following statement(s) is/are true concerning metabolic derangements in sepsis and the systemic inflammatory response syndrome which may follow progressive shock?
a. Alterations in glucose metabolism lead to the development of efficient substrate utilization
b. A progressive rise in serum triglyceride levels result from less efficient clearance and increased hepatic lipogenesis
c. A net negative nitrogen balance occurs due to the oxidative metabolism of proteins to meet energy needs
d. The serum aromatic amino acids fall rapidly as they are actively used in oxidative metabolism
Answer: b, c

A broad spectrum of metabolic abnormalities become apparent in sepsis and the systemic inflammatory response syndrome following shock. Disruption of the normal cycles of carbohydrate, lipid, protein, and oxygen metabolism occur as hypermetabolism develops. Through the Cori cycle, lactate from the periphery is shuttled back to the liver, where it is used in the production of glucose. Because pyruvate is converted to alanine in the periphery, flux of alanine also contributes to hepatic gluconeogenesis. The glycolytic oxidation of glucose to pyruvate and its subsequent glugoneogenic regeneration from lactate is an inefficient cycling of substrate. There is no net energy production, but heat is released in significant quantities. Alterations in lipid metabolism cause a progressive rise in the serum triglyceride level as a result of less efficient clearance of exogenous triglycerides coupled with increased hepatic lipogenesis. Profound alterations in protein and amino acid metabolism develop with characteristic changes in amino acid levels, nitrogen balance, and skeletal muscle mass. Initially levels of the branch chain amino acids are reduced, whereas those of the aromatic amino acids are elevated. There is an increase in the oxidative metabolism of protein to meet energy needs and a tremendous mobilization of nitrogen with net negative nitrogen balance. The branch chain amino acids are preferentially utilized in the TCA cycle to maintain an activity that otherwise would be lost from the diminished entry of carbohydrate-and fatty acid-generated acetyl coenzyme A. This results in reduced serum level of leucine, isoleucine and valine.


216 Which of the following statement(s) is/are true concerning the microvascular and cellular response to shock?
a. Osmodically induced mobilization of intracellular fluid is the initial response to restore intravascular volume
b. With larger volume hemorrhagic shock deterioration of normal cellular transmembrane potential occurs resulting in an increase in extracellular sodium and water
c. The accumulation of anaerobic metabolites override normal homeostatic vasomotor tone and contribute to the maladaptive vasodilatation
d. Abnormal intracellular calcium homeostasis may contribute to the cellular dysfunction of shock
Answer: c, d

Moderate hypovolemia results in a relatively rapid spontaneous restitution of intravascular volume through expansion of the plasma space. This plasma expansion by erythrocyte free fluid occurs within one hour as a result of alterations in pressure and osmolarity and produces an associated hemodilution. Sympathetic discharge, associated arteriolar constriction, and induced metabolic changes in osmolarity initiate the compensatory events at the microcirculatory level. The initial pressure-related phase of restitution of blood volume in shock is overlapped by a second phase involving osmotically induced mobilization of intracellular fluid. Osmotic mechanisms contributing to the restitution of blood volume after moderate hemorrhage are not adequate in hemorrhage of greater magnitude. In larger hemorrhages (over 25% blood volume), there is also deterioration of the normal cellular transmembrane potential, an increase in intracellular sodium and water, and a concomitant decrease in extracellular fluid volume. Tissue hypoxia results, anaerobic metabolites accumulate, and the cell cannot maintain the normal cell membrane potential. Accumulation of hydrogen ion, lactate, and other products of anaerobic metabolism override homeostatic vasomotor tone and contribute to a maladaptive vasodilatation, further augmenting hypotension and hypoperfusion. The uptake of fluid by the “failing” cell is a major source of food sequestration following shock. Loss of membrane function is proportional to both the extent and duration of shock or degrees of sepsis. The etiology of membrane failure is unclear but appears multifactorial. Loss of intracellular ATP energy stores during hypoperfusion or direct toxicity during sepsis may inhibit the membrane sodium-potassium pump. Cellular dysfunction also appears to be related to abnormal intracellular calcium homeostasis.


217 Which of the following statement(s) is/are true concerning the pulmonary response to shock?
a. The acute pulmonary vascular response to shock differs markedly from that of systemic vasculature
b. The pulmonary edema of ARDS occurs in the face of elevated left heart pressures
c. The initial physiologic changes of ARDS involve the capillary endothelial cells and the type I pneumocyte
d. Mechanisms proposed in the pathogenesis of ARDS include injury from mediators of inflammation elsewhere and from activated cellular elements
e. A decrease in lung compliance may result from the loss of type I pneumocytes
Answer: c, d, e

Contributing pathophysiologic processes to the pulmonary manifestations of shock include the pulmonary component of the cardiovascular response, disruption of the normal lung mechanics, and acute lung injury or ARDS due to sepsis. Pulmonary function may be further compromised by pathology intrinsic to the lung itself, including pulmonary contusion, aspiration, airway obstruction, pneumonia, pneumothorax, hemothorax, and atelectasis. The acute pulmonary vascular response to shock largely parallels that of the systemic vasculature. The increase in pulmonary vascular resistance, which may proportionally exceed that of the systemic circulation, transiently accompanies the systemic adrenergic response. ARDS is a syndrome of progressive lung injury that may arise as a direct consequence of shock or other disease processes. The characteristic findings of ARDS are the presence of pulmonary edema, hypoxemia, and significantly decreased lung compliance. The pulmonary edema is noncardiac in origin and occurs in the face of normal left heart pressures. The hypoxemia results from the development of intrapulmonary shunting and perfusion of under and nonventilated alveoli. The decrease in lung compliance results from the loss of surfactant and lung volume in combination with the presence of interstitial fluid and alveolar edema. Progressive histologic changes of ARDS become apparent in pulmonary capillaries, interstitium, and alveoli. Initially, interstitial edema develops with swelling of the capillary endothelial cells and the type I pneumocytes. The type I pneumocytes subsequently slough, and alveolar edema ensues. Functional surfactant is lost with a significant increase in alveoli opening pressure and decrease in alveolar surface tension. Mechanisms proposed in the pathogenesis of ARDS include injury from mediators of inflammation elaborated elsewhere, and from activated cellular blood elements.


218 Which of the following statement(s) is/are true concerning the diagnosis and management of hypovolemic shock?
a. A fall in hematocrit or hemoglobin always accompanies hemorrhagic shock
b. The treatment of shock is generic regardless of the etiology
c. Pharmacologic intervention to increase myocardial contractility in hypovolemic shock is an important part the early management
d. Complications are less frequent after treatment of hemorrhagic shock than septic or traumatic shock
Answer: d

Hypovolemic shock is readily diagnosed when there is an obvious source of volume loss and overt signs of hemodynamic instability and increased adrenergic output are present. After acute hemorrhage, hemoglobin and hematocrit values do not change until compensatory fluid shifts have occurred or exogenous fluid is administered. These values decrease once transcapillary refill, osmotic-induced shifts, or non-RBC volume resuscitation expands the blood volume. It is imperative that the distinction be made between hypovolemic and cardiogenic forms of shock, because appropriate therapy differs dramatically. Restoration of perfusion in hypovolemic shock requires reexpansion of circulating blood volume in conjunction with necessary interventions to control ongoing volume loss. Continued hemodynamic instability after fluid resuscitation implies that shock has not been reversed or that there is ongoing blood or volume loss. In severe, prolonged hypovolemia, ventricular contractile function may itself become depressed and require inotropic support to maintain ventricular performance, but in general, pharmacologic interventions directed toward increased contractility in situations of inadequate preload are ineffective, further complicate metabolic derangements, and are not indicated until adequate volume replacement has been completed. Complications are less frequent after treatment of hemorrhagic shock than in situations of septic or traumatic shock. In the later circumstances, the massive activation of inflammatory mediator response systems and consequences of their disseminated, indiscriminate cellular injury can be quite profound.


219 Which of the following statement(s) is/are true concerning the neuroendocrine responses to shock?
a. Sympathetic nerve endings release epinephrine which is responsible for greater than 80% of systemic vascular resistance
b. Endogenous epinephrine is the primary contributor to systemic vascular resistance
c. Increased pancreatic secretion of glucagon contributes to glucose intolerance associated with injury and sepsis
d. The renin-angiotensin axis further augments the sympathetic-mediated vasoconstriction
Answer: c, d

The neuroendocrine response to shock attempts to achieve restoration of effective blood volume, mobilization of metabolic substrates, and maintenance of central profusion. Both peripheral and central afferent stimuli to the central nervous system are involved in inducing this response. Hypotension, associated with a decrease in impulses from the aortic and carotid baroreceptors, disinhibits the vasomotor center. This disinhibition results in increased adrenergic output and decreased vagal activity. Sympathetic nerve endings release norepinephrine, inducing peripheral and splanchnic vasoconstriction which is responsible for greater than 80% of systemic vascular resistance and is a major contributor to maintenance of central organ perfusion and venous return. Plasma levels of both epinephrine and norepinephrine are elevated with injury, and the degree of the catecholamine elevation corresponds to the magnitude of injury. In shock the effects of endogenous epinephrine are largely metabolic. In addition to initiating autonomic nervous activity, the hypothalamus secretes releasing hormones, which induce the stress hormone release of the pituitary. As part of this response, adrenocorticotropic hormone (ACTH) secretion by the anterior pituitary is increased stimulating cortisol secretion by the adrenal cortex. In conjunction with elevated plasma levels of cortisol and epinephrine, increased pancreatic secretion of glucagon accelerates hepatic gluconeogenesis and further aggravates the glucose intolerance that follows injury and sepsis. The secretion of renin is increased in responses to adrenergic discharge and decreased perfusion of the juxtaglomerular apparatus in the kidney. Renin allows formation of angiotensin I in the liver, which is then converted to angiotensin II in the lungs. Angiotensin II is an extremely effective vasoconstrictor that further augments sympathetic-mediated vasoconstriction.

220 A 22-year-old man sustains a single stab wound to the left chest and presents to the emergency room with hypotension. Which of the following statement(s) is/are true concerning his diagnosis and management?
a. The patient likely is suffering from hypovolemic shock and should respond quickly to fluid resuscitation
b. Beck’s triad will likely be an obvious indication of compressive cardiogenic shock due to pericardial tamponade
c. Echocardiography is the most sensitive noninvasive approach for diagnosis of pericardial tamponade
d. The placement of bilateral chest tubes will likely resolve the problem
Answer: c

Shock from cardiac compression occurs when external pressure on the heart impairs ventricular filling. Because ventricular filling is a function of venous return and myocardial compliance, any process that places pressure on the heart can cause compressive cardiogenic shock. Included among these are pericardial tamponade, tension pneumothorax, mediastinal hematoma, and positive pressure from mechanical ventilation. Any patient with hypotension after a wound in proximity of the heart should be considered to have compressive cardiogenic shock until proven otherwise. The classical clinical findings of pericardial tamponade include Beck’s triad of hypotension, neck vein distention and muffled heart sounds. Pulses paradoxus may be noted (this involves a decrease rather than the normal increase of systolic blood pressure with inspiration; values 10mmHg are significant). These findings, however, may be obscured in a noisy emergency room environment by positive pressure ventilation or by associated injuries. Placement of a CVP catheter confirms the elevation of right-sided filling pressure. If a pulmonary artery catheter has been placed, findings consistent with tamponade or other forms of cardiac compression are a trend toward equalization of chamber pressures as hypotension progresses. In the patient at risk, echocardiography is an extremely sensitive and noninvasive approach to demonstrate pericardial fluid and the need for operation. Pericardial tamponade must be relieved urgently and cardiac injuries require emergent sternotomy. Chest tube placement would not be appropriate as the sole treatment in this patient.


221 A 32-year-old man suffers a spinal cord injury with a resultant paraplegia in a motorcycle accident. He presents to the emergency room with hypotension. Which of the following statement(s) is/are true concerning his diagnosis and management?
a. The low blood pressure can be assumed to be due to neurogenic shock
b. The sole cause of hypotension is the loss of sympathetic input to the venous system
c. Despite significant hypotension, secondary organ injury will be uncommon
d. There is no role for pharmacologic intervention to maintain blood pressure
Answer: c

Neurogenic shock results from interruption of sympathetic vasomotor input and develops after spinal cord injury, spinal anesthesia, and severe head injury. Under normal conditions, baseline sympathetic activity establishes a degree of arteriolar and venous constriction. Ablation of this tone results in decreased systemic vascular resistance and a dramatic increase in venous capacity, causing hypotension due to relative hypovolemia. Arteriolar dilatation not only lowers the systemic vascular resistance but also allows previously unopened vascular beds to be perfused, greatly expanding venous capacity. Removal of sympathetic inputs to innervated portions of the venous system allows further venodilatation. Restoration of an effective, albeit expanded, intravascular volume may require extremely large volumes of resuscitation fluid to restore normal cardiac filling pressures. This will restore cardiac output and reverse hypotension. However, pharmacologic intervention with vasoactive drugs may be necessary and is preferable to excessive volume resuscitation. Post-shock sequelae are infrequent. Although there is significant hypotension with neurogenic shock, there is usually little if any hypoperfusion. Thus, activation of inflammatory cascade and subsequent organ injury rarely occur.
A major pitfall in the management of neurogenic shock arises when there is coexistent hemorrhage or ongoing volume loss that is not appreciated. This is not an unusual situation because cervical spine trauma causing paraplegia or severe head injury is frequently associated with multiple injuries. Thus, in trauma the initial response to neurogenic shock is large volume resuscitation regardless of the presumed etiology. If hemodynamic instability persists after initial trauma resuscitation, one must assume that the cause is not neurogenic and search for occult blood loss or cardiogenic causes of shock.


222 Which of the following statement(s) is/are true concerning septic shock?
a. The clinical picture of gram negative septic shock is specifically different than shock associated with other infectious agents
b. The circulatory derangements of septic shock precede the development of metabolic abnormalities
c. Splanchnic vascular resistance falls in similar fashion to overall systemic vascular resistance
d. Despite normal mechanisms of intrinsic expansion of the circulating blood volume, exogenous volume resuscitation is necessary
Answer: d

The clinical findings in sepsis and septic shock represent the host response to infection. Gram-positive and gram-negative bacteria, viruses, fungi, rickettsiae, and protozoa have all been reported to produce a clinical picture of septic shock, but the overall response is independent of the specific type of invading organism. Septic shock develops as a consequence of the combination metabolic and circulatory derangements accompanying the systemic infection. It appears that the circulatory deficits are preceded by the metabolic abnormalities induced by infection. In fact, the circulatory changes in hyperdynamic sepsis appear to be an adaptive response to the underlying metabolic dysfunction. Cardiac output is high and systemic vascular resistance low in hyperdynamic septic shock. However, splanchnic vasoconstriction is pronounced even in the absence of systemic hypotension and even though systemic vascular resistance is reduced. Expansion of circulating blood volume can occur through either transcapillary refill or fluid resuscitation. Due to the ongoing inflammatory mediator-induced increases in capillary permeability and continued loss of intravascular volume, exogenous volume resuscitation must be provided to restore venous return and ventricular filling.


223 Which of the following statement(s) is/are true concerning tumor necrosis factor (TNF)?
a. TNF is a product of activated macrophages secreted in response to contact with endotoxin or lipopolysaccharide, antibody complexes, or inflammatory stimuli
b. The liver and gut appear to be a major source of TNF following hypoperfusion
c. Circulating levels of TNF correlate well with severity of tissue injury in shock
d. Recently completed clinical trials of anti-TNF antibody in septic patients shows a marked improvement in survival
Answer: a, b

Tumor necrosis factor (TNF), a protein product of activated macrophages, is secreted in response to contact with endotoxin or lipopolysaccharide, antibody complexes, or other inflammatory stimuli. Elevation of serum levels of TNF have been reported shortly after experimental trauma and shock, however, documentation of elevated circulating levels of TNF in human shock is less clear. Furthermore, circulating levels of TNF cannot be correlated with severity of tissue injury or shock. This variability is thought to be due to rapid clearance and uptake by membrane receptors and by soluble membrane receptors that are released from multiple cells following stress and injury. Following hypoperfusion the liver and gut appear to be the major source of TNF that is rapidly cleared but responsible for inducing hepatocyte changes following shock. The release of breakdown products and escape of bacterial and endotoxin through the damaged mucosal barrier of the gut following shock allows or induces activation of tissue-fixed macrophage (Kupffer cell) of the liver which then produces secondary inflammatory mediators contributing to the post-resuscitation clinical response and inflammatory mediator activation seen in the systemic inflammatory response syndrome.
TNF is central to inflammatory response, particularly in sepsis and following endotoxemia or bacteremia. TNF also induces secondary inflammatory responses through direct interaction with specific membrane receptors, TNF-r. Treatment with anti-TNF antibody in the experimental setting protects animals from the deleterious effects of lethal bacteremia and endotoxemia. However, recently completed clinical trials in septic patients utilizing infusion of monoclonal antibodies to the TNF molecule have shown no overall survival benefit.


224 Which of the following statement(s) is/are correct concerning the immunoinflammatory response to shock?
a. The anaphylactoxins, C3a and C5a, are products of activation of only the classical pathway of the compliment cascade
b. Eicosanoids, such as prostaglandins are stored in platelets and endothelial cells and released in response to inflammatory stimuli
c. Thromboxane and PGI2 have similar effects
d. Platelet-activating factor can be released by both circulating and fixed tissue cells
Answer: d

Inflammatory mediators have recently been recognized as playing a significant role in the clinical manifestations and progression of shock and the development of subsequent complications. These mediator systems function primarily as parcrine and autocrine agents in the local environment and are not usually detectable systemically. The over-expression and systemic dissemination of these mediators produces the toxic autodestructive processes underlying multiorgan failure syndrome with attendant high mortality. The compliment cascade is activated in shock and tissue injury through both the classical and alternative pathways. Activation of either pathway results in generation of the anaphylatoxin, C3a and C5a, soluble products with potent systemic hemodynamic effects. The eicosanoids, which include the prostaglandins and leukotrienes are formed acutely from arachidonic acid released from the membrane phospholipid by phospholipase A2. Eicosanoids are not stored in any measurable level and are generated as needed from readily available arachidonic acid in response to various inflammatory phenomena. Platelets, white cells, and endothelial cells are a rich source of these compounds. Thromboxane (TXA2) is the major arachidonic acid metabolite elaborated by platelets. TXA2 induces intense vasoconstriction, platelet aggregation and degranulation, neutrophil margination in the microcirculation and bronchial constriction. PGI2, the major arachidonic acid metabolite formed by endothelial cells, serves a check against actions of TXA2. PGI2 is a vasodilator and a potent inhibitor of platelet aggregation. Platelet aggravating factor is a potent phospholipid mediator released by neutrophils, platelets, macrophages and endothelial cells in response to ischemia, tissue injury and sepsis. Its effects include decreased cardiac function, increased pulmonary vascular resistance, bronchoconstriction, peripheral vasodilatation, and increased vascular permeability.


225 Which of the following physical findings are associated with the various classes of hemorrhagic shock?
a. Mild shock (< 20% blood volume): Pallor, cool extremities, diminished capillary refill and diaphoresis b. Moderate shock (20%–40% blood volume): All of the above plus tachycardia and hypotension c. Severe shock (> 40% blood volume): Systemic hypotension, changes in mental status, tachycardia, oliguria
d. All of the above
Answer: a, c
PHYSICAL FINDINGS IN HEMORRHAGIC SHOCK*

Moderate
Mild (<20% (20%-40% Severe(>40%
Blood Volume) Blood Volume) Blood Volume)
Pallor Pallor Pallor
Cool extremities Cool extremities Cool extremities
Diminished capillary Diminished capillary Diminished capillary
refill refill refill
Diaphoresis Diaphoresis Diaphoresis
Collapsed Collapsed Collapsed
subcutaneous subcutaneous subcutaneous
veins veins veins
Tachycardia Tachycardia
Oliguria Oliguria
Postural Hypotension
hypotension Mental status
changes

* Alcohol or drug intoxication may alter physical findings.


226 A 68-year-old male who underwent a repair of an abdominal aortic aneurysm 5 days ago, develops tachycardia, tachypnea, hypotension with cool, pale, mottled cyanotic extremities. He is agitated and complains of shortness of breath. Which of the following statement(s) is/ are correct concerning his diagnosis and management?
a. Myocardial ischemia secondary to preexisting coronary artery disease is most likely the underlying cause of this problem
b. Invasive hemodynamic monitoring with a Swan-Gantz catheter will demonstrate a low cardiac output, a high systemic vascular resistance, and elevated cardiac filling pressures
c. The use of morphine sulphate and nitrates should be part of the initial management
d. The primary pharmacologic treatment involves the use of moderate doses of inotropic agents
e. Afterload reduction with nitroprusside is absolutely contraindicated
Answer: a, b, d

Intrinsic cardiogenic shock results from failure of the heart as an effective pump. Coronary artery disease is the most common cause of myocardial insufficiency, but contractile dysfunction may also rise as a consequence of cardiomyopathy, myocarditis, or metabolic abnormalities. Invasive hemodynamic monitoring often establishes a specific nature of shock and allows appropriate treatment to be delivered in an effective and expedient manner. Hemodynamic findings consistent with cardiogenic shock include a low cardiac output and high systemic vascular resistance, with elevated cardiac filling pressures. The initial measures in the management of cardiogenic shock include the administration of supplemental oxygen, mechanical ventilation (as needed), and appropriate treatment of dysrhythmias. Hypotension usually precludes the use of morphine sulfate and nitrates, drugs typically used in simple congestive heart failure to alleviate cardiac pain and ameliorate pulmonary vascular congestion. The use of beta-adrenergic agonists such as dopamine and dobutamine, in moderate doses, offers positive inotropic support without excessive alpha-adrenergic activity. Increasing the inotropic state of the heart shifts the entire Starling curve upward, resulting in increased cardiac output for each level of cardiac filling. Afterload reduction may prompt increases in cardiac output through decreases in resistance to flow. The use of nitroprusside or other dilators requires relative blood pressure stability and close hemodynamic monitoring. Infusion of afterload-reducing agents can be administered in conjunction with inotropic support.


227 Which of the following statement(s) is/are true concerning ischemia reperfusion injury?
a. During ischemia, ATP degradation results in increased plasma and intracellular levels of hypoxanthine and xanthene
b. Oxygen free radicals such as the superoxide radical are involved in the expression of the proinflammatory phenotype of endothelial cells, macrophages and neutrophils
c. The intracellular adhesion molecule-1 (ICAN-1) contributes to injury and disruption of the endothelial lining, with extensive capillary leak and resultant interstitial edema
d. Animal models have demonstrated that passive immunization with antibodies to neutrophil adhesive complex lessen the ischemic/reperfusion microvascular injury
Answer: a, b, c, d

During the ischemia and hypoperfusion phase, degradation of ATP stores essential to maintain cell integrity and significant loss of diffusible intracellular adenine neuclotides occurs. As ATP further degrades there is an elevation in plasma and intracellular levels of hypoxanthine and xanthene which upon restoration of perfusion and reoxygenation are catalyzed by xanthine oxidase resulting in the formation of superoxide radicals. These radicals plus others such as hydrogen peroxide and hydroxyl radical are generated and lead to endothelial and parenchymal cell injury through membrane lipid peroxidation and activation of critical enzymes. These radicals have also been shown to be involved in the expression of proinflammatory phenotype endothelial cells and on macraphages and neutrophils. The proinflammatory phenotype of the endothelium includes procoagulant activity and the expression of adhesion molecules on the membrane surface, including the intercellular adhesion molecule-1 (ICAN-1) and the selectins. The subsequent adhesion of activated neutrophils to the endothelial leads to an explosive oxidative burst producing additional radicals and extensive release of proteolytic enzymes leading to injury and disruption of the endothelial lining, extensive capillary leak, and massive interstitial edema. Passive immunization of animals with monoclonal antibodies to either the neutrophil adhesive complex or the endothelial selectins dramatically lessens ischemia/reperfusion microvascular injury.


228 Which of the following statement(s) is/are true concerning the physiology of the microvascular system?
a. Filtration of capillary fluid into the interstitial and the subsequent reabsorption is influenced by Starling’s law of ultrafiltration
b. The most important variable controlling blood to a capillary bed is the length of the vessel
c. Most of the resistance to systemic blood flow occurs at the arteriolar level
d. Adrenergic vasoconstriction can arrest blood flow to an entire capillary bed
Answer: a, c, d

Exchange of material between the vascular space and the cell of various tissues via the interstitial space is essential for organ viability and occurs at the capillary level. The filtration of capillary fluid into the interstitium and its subsequent reabsorption into the post capillary venule is governed by microvascular permeability in conjunction with the balance between hydrostatic and oncotic pressures. The relation of these forces to one another (and their net effects) are illustrated by what is termed Starling’s law of ultrafiltration. In normal circumstances, a net filtration from capillary to interstitium is effected by a relatively higher capillary hydrostatic pressure, whereas net reabsorption from the interstitium back into the post capillary venule occurs as hydrostatic pressure falls and oncotic forces predominate. Although the mechanisms controlling blood flow to the capillary bed are complicated and vary among the different tissues, certain concepts are useful. Poiseuille’s law describes the relation between flow of fluid through a tube and the tube length and radius, the fluid viscosity, and the pressure gradient between ends of the tube. The radius of the tube (or vessel) is the single most important variable, because flow is proportional to the radius to the fourth power. Vasoconstrictive and vasodilatory influences directly impact local blood flow, as well as flow to other tissues through secondary effects on the systemic pressure. This secondary effect of peripheral vasoconstriction maintains the pressure gradient for central perfusion of the heart and brain. Systemic blood flow meets most of its resistance at the arteriolar level. While the individual capillary radius is significantly smaller, the vast number of capillaries offers less total resistance. The vascular smooth muscle in arterioles has both a-and b- adrenergic receptors. Alpha stimulation affects vasoconstriction where beta stimulation affects vasodilatation. The efferent sympathetic fibers innervating the precapillary resistance vessels and the venous capacitance vessels release norepinephrine on stimulation, which induces smooth muscle contraction and narrowing of the caliber of the vessels. These contractions are potent enough that blood flow to entire capillary beds can be arrested by adrenergic vasoconstriction.


229 Which of the following statement(s) is/are true concerning the effects of MOFS?
a. Pulmonary dysfunction tends to arise early and may resolve within 7 to 10 days
b. Unless the precipitating insult has prompted oliguric acute tubular necrosis, renal function tends to be maintained early in the course of MOFS
c. Although hepatic dysfunction is common with MOFS, the GI tract plays little role in this process
d. Intercurrent nosocomial infection, most commonly pulmonary, is a common complication providing a “second hit” to the patient
Answer: a, b, d

Pulmonary dysfunction typically arises early in the development of systemic inflammation and may represent mild relatively localized acute lung injury or it may be a prelude to fulminant ARDS. The lung injury, and associated dysfunction, may resolve over the initial 7 to 10 days or persist, depending on the ongoing pathologic process. Many times a “second hit” such as a nosocomial infection, which is most commonly pulmonary, is a complication which can frequently worsen the pulmonary condition. Renal function tends to be maintained early in the course unless the precipitating insult has been prompted by a sudden oliguric acute tubular necrosis. With persistent activation and inflammatory mediators, glomerular filtration falls and the development of oliguric or polyuric renal failure marks the gradual transition into MOFS. Gastrointestinal abnormalities include ileus, stress ulceration, diarrhea, and mucosal atrophy. Breakdown of the mucosal barrier allows translocation of bacteria and endotoxin. Hepatic dysfunction is marked by progressive rise in serum bilirubin levels after a latent period of several days.


230 Which of the following statement(s) is/are true concerning hypoadrenal shock?
a. In the United States, idiopathic adrenal atrophy (Addison’s disease) is the most common cause
b. Laboratory abnormalities include hyponatremia, hypochloremia, and hyperkalemia
c. Fever may be seen with hypoadrenal shock
d. ACTH stimulation test is the diagnostic test of choice to confirm hypoadrenal shock
Answer: b, c, d

Shock of a dramatic nature, poorly responsive to resuscitation, may develop as a consequence of adrenal insufficiency. In this country, adrenal insufficiency most commonly arises as a consequence of the chronic therapeutic administration of high doses exogenous corticosteroids causing adrenal suppression. Other causes include idiopathic adrenal atrophy (Addison’s disease), tuberculosis, metastatic disease, bilateral hemorrhage, and amyloidosis. The stress of illness, operation, or trauma typically requires that the adrenal glands secrete cortisol in excess of that required in the nonstressed state (approximately 3–4 fold). Insufficiency not otherwise apparent may manifest itself only after major physical stress. Findings associated with adrenal insufficiency include weakness, fatigue, anorexia, abdominal pain, nausea, vomiting, and weight loss. Surgical patients with significant adrenal insufficiency need not present with the above findings. More typical is the development of refractory shock, frequently with hyperthermia, in the course of injury or illness. Hypotension may be dramatic despite massive volume resuscitation and pressor support. Laboratory findings suggesting hypoadrenalism include hyponatremia, hypochloremia and hyperkalemia. The diagnosis of adrenal insufficiency may be confirmed or excluded by means of an ACTH stimulation test. A significant major cortisol response should be elicited by ACTH administration.


231 Which of the following statement(s) is/are correct concerning the cardiovascular response to shock?
a. Changes in cardiac contractile function shift the Frank Starling curve up and down
b. Venoconstriction from skeletal muscle is a significant contributor to the restoration of blood volume with shock
c. Arterial vasoconstriction affects all vascular beds equally
d. The total circulating blood volume is equally split between the arterial and venous system
Answer: a

Central in the general cardiovascular response to shock is the action of the heart itself. The principle determinants of cardiac function in the normal heart are the volume of blood available for the heart to pump (preload), the systolic contractile capability, and the diastolic filling of the ventricles. In hypovolemia, the two dynamic variables of cardiac function, ventricular filling and myocardial contractility remain paramount and determine the stroke volume. The product of stroke volume and heart rate in turn determines the cardiac output. Increases in ventricular end-diastolic volume, reflecting venous return, cause ventricular distention. Ventricular distention in turn produces increased volume output with each stroke, the Frank Starling mechanism. Contractile function may vary independent of volume status. Changes in the contractile function shift the Starling curve up and down, producing increases or decreases in stroke volume for any given end-diastolic volume. A fundamental requirement for cardiovascular function is adequate cardiac filling, and cardiac output cannot exceed venous return. The venous system contains nearly two-thirds of the total circulating blood volume, including 20% to 30% within the splanchnic venous system. Most of this volume resides in small veins, which comprise the bulk of venous capacitance. The venous system, especially that of the splanchnic circulation, becomes important in the physiologic compensation to hypoperfusion because it serves as a dynamic reservoir for the autoinfusion of blood volume involving both active and passive mechanisms. The splanchnic circulation makes major contributions to the maintenance of venous return, therefore, it is likely that sympathetic venoconstriction is responsible for a portion of the blood mobilized from the splanchnic venous circulation. Sympathetic mediated venoconstriction in skin and skeletal muscle is probably not as significant as a source of blood volume. Selective vasoconstriction occurs in response to alpha adrenergic receptor stimulation with increased sympathetic activity in shock. Sympathetic stimulation does not cause significant vasoconstriction of either cerebral or coronary vessels, with normal blood flow maintained in these circulations. Blood flow to the skin is sacrificed early, followed by that to the kidneys and splanchnic viscera.


232 Which of the following statement(s) is/are true concerning pharmacologic agents used in the treatment of shock?
a. The primary difference between dopamine and dobutamine is the absence of significant a adrenergic activity
b. The renal and mesenteric vasoconstrictive effects of norepinephrine complicate and sometimes restrict its use
c. The apparent paradoxical use of vasodilators, such as nitroprusside, in shock is indicated as a means to augment cardiac function
d. Isoproterenol with its potent b-adrenergic effect, is a particularly useful agent in the treatment of all forms of shock
Answer: a, b, c

Therapeutic adjustments of intravascular volume (preload) and systemic vascular resistance (afterload) form the basis of the treatment strategies for all forms of shock. Optimal volume resuscitation should precede measures to augment to contractile function of the heart. Inotropic agents are used in shock when there is inadequate cardiac output despite adequate circulating blood volume. Dopamine and dobutamine are often times first line agents in the pharmacologic treatment of shock. Dopamine, at low doses, stimulates dopaminergic receptors producing renal arteriolar vasodilatation with associated increases in renal blood flow, urine output, and sodium excretion. At moderate doses, stimulation of cardiac b-receptors produces increases in contractility and cardiac output with little effect on heart rate or blood pressure. At higher doses, peripheral vasoconstriction from increasing a activity becomes more pronounced, prompting significant increases in vascular resistance and blood pressure. Dobutamine’s predominant effect is an increasing cardiac contractility with lesser increases in heart rate. Some reduction of peripheral vascular resistance may also occur. When compared to dopamine, dobutamine produces less peripheral vasoconstriction and less chronotropic response. Norepinephrine exerts both a and b-adrenergic effects, with a effects being evident at lower infusion rates and a effects more prominent at high doses. The major use of norepinephrine in current practice is in the patient with hypotension that persists despite appropriate volume resuscitation and the use of inotropic agents. Renal and mesenteric vasoconstrictive effects of norephinepherine complicate its use, especially when support is needed for significant periods of time. Isoproterenol is a potent b-adrenergic agent. With isoproterenol, myocardial oxygen demand is increased and diastolic coronary feeling is limited by tachycardia or diminished diastolic pressure. Indications for the use of isoproterenol are fairly limited, because agents with fewer adverse effects have become available.
Vasodilators are used to augment cardiac function through optimization of ventricular filling pressures (preload) and systemic vascular resistance (afterload) both of which reduce demands on the myocardium. Decreases in afterload prompt increases in cardiac output and venodilatation contributes to decreases in pulmonary venous pressure and central venous pressure. Hypotension, however, may develop therefore patients must have careful constant monitoring of arterial pressure and repeated hemodynamic measurements with a pulmonary artery catheter.


233 Which of the following statement(s) is/are true concerning the treatment of MOFS?
a. Prevention and therapy of MOFS requires control of the infectious or inflammatory source
b. Restoration of normal clinical parameters such as blood pressure, pulse rate, and urine output ensures optimal resuscitation in most patients
c. Branch chain amino acids play and important role in the nutritional support of the patient
d. Because of the nature of gut injury, total parenteral nutrition is preferred for most patients with MOFS
Answer: a, c

The therapy of MOFS is directed towards interrupting the involving pathophysiologic process and providing an optimal physiologic environment for healing and recovery. Fundamental concerns are control of the source of infection, inflammation or instability; restoration of microcirculatory blood flow and oxygen transfer, and the institution of optimal supportive care. Both the prevention and therapy of MOFS, therefore, requires source control and restoration of adequate profusion. Resuscitation efforts are directed toward restoration of adequate microcirculatory blood flow in all organ systems. Restoration of normal clinical parameters such as blood pressure, pulse rate, urine output, and acid-base balance does not ensure optimal resuscitation. The physiologic endpoint that most closely corresponds with adequate microcirculatory flow is the level of cardiac output and the oxygen delivery at which oxygen consumption and lactate production remain independent of flow.
The importance of metabolic support in the patient with MOFS cannot be overemphasized. The malnutrition of MOFS is markedly different than that of starvation and the nutritional requirements also differ. If optimal quantities of appropriately formulated amino acid solutions are given, protein synthetic rates can approach catabolic rates and the goal of nitrogen balance can be achieved. Formulas rich in branch chain amino acids appear to be more efficient in promoting nitrogen retention and minimizing urea production. Whenever feasible, enteral feeding is preferred over TPN because evidence suggests that bacterial translocation from the gut can be limited through the use of enteral feeds. Enteral absorption and processing of nutrients appears superior to TPN and lessens overall complications.


234 Which of the following statement(s) is/are true concerning the multiorgan failure syndrome (MOFS)?
a. Changes in the splanchnic and pulmonary microcirculation are critical to the development of MOFS
b. Tissue fixed microphages, including the liver Kupffer cell, have little role in the development of MOFS
c. MOFS represents systemic consequences of loss of homeostatic control of local inflammation and microcirculatory hypoperfusion
d. The nature of MOFS is highly dependent upon the etiology of the underlying problem
Answer: a, c

The nature of multiorgan failure syndrome (MOFS) is that of a diffuse cellular injury, developing systemically as a consequence of losing homeostatic control of local inflammation and microcirculatory hypoperfusion. Endothelial injury, platelet aggregation and activation of macrophages and neutrophils occur, and the clotting, fibrinolytic, kinin, and complement cascades are activated, along with the release of potent inflammatory cytokines. The effects of shock, resuscitation, and reperfusion, and the subsequent development of MOFS appear to be critically dependent on changes in the splanchnic and pulmonary microcirculations. These vascular beds appear to be major sites of activation of subsequent inflammatory mediator production that underlies the diffuse systemic inflammatory response. Extensive activation of the liver Kupffer cell and release of inflammatory mediators coupled with the ongoing release of activated neutrophils and by-products of activated gut macrophages is responsible for the injury to the pulmonary microcirculation and secondary induction of alveolar macrophage and additional inflammatory mediator systems. Excessive and persistent macrophage activation plays an essential role in MOFS and is hypothesized to represent the penultimate step in a series of continuous immuno-inflammatory stimulatory events, including local hypoxia, exposure to bacteria and toxins, and mediator release from localized areas of inflammation. When infection is the underlying or major contributing process, the diffuse inflammatory response develops independently of the specific type of microorganism. In noninfectious cases, the response also appears independent of the specific underlying cause.


235 Invasive hemodynamic monitoring using a Swan-Gantz catheter is essential in the optimal management of patients in shock or those suffering post-shock sequelae. Which of the following physiologic characteristics are associated with the various forms of shock?
a. Hypovolemic shock: Decreased pulmonary capillary wedge pressure (PCWP), decreased cardiac output, increased systemic vascular resistance (SVR)
b. Cardiogenic shock: Increased PCWP, decreased cardiac output, decreased SVR
c. Septic shock (hypodynamic): Decreased cardiac output, increased SVR
d. Neurogenic shock: Decreased PCWP, increased cardiac output, decreased SVR
Answer: a, c

236 Which of the following statement(s) is/are true concerning the relationship between cardiac function and effective blood volume?

a. A pulmonary capillary wedge pressure of 5–10 rules out fluid overload as a cause of pulmonary edema
b. A shift to the right in the Frank-Starling curve is associated with compromised cardiac function
c. Dilutional anemia may contribute to tachycardia even though blood volume and filling pressures are normal
d. The sole purpose of a pulmonary artery catheter is to measure pulmonary artery pressure and cardiac output
Answer: b, c

Although physical findings are often adequate to establish a diagnosis and institute management of cardiac failure, direct measurement of filling pressures of the right heart (central venous pressure) or the left heart (pulmonary artery pressure) may be required. Placement of a pulmonary artery catheter allows us to measure cardiac output by thermodilution and, more importantly, to sample mixed venous blood for saturation measurements which tell us the ratio between systemic oxygen delivery and oxygen consumption. From all of these measurements we can determine if cardiac output is normal for the level of filling pressure of the left ventricle, or if contractility is decreased. In the latter case, cardiac output will be lower than predicted for a given level of filling pressure. In the Frank-Starling curve, if the patient is to the right of the normal range, then cardiac function is compromised either because of valvular disease, extrinsic pressure such as pericardial tamponade, or more commonly, a decrease in contractility. If cardiac function and anatomy are normal, then blood volume, filling pressure and cardiac function are related to the Starling curve. The intake and output of fluid and salt is autoregulated to maintain the filling pressure of the left ventricle around 10 mm Hg. Extracellular fluid expansion is usually associated with normal blood volume. Gross expansion of extracelluar space results in deleterious effects if tissue edema can and often do exist with perfectly normal blood volume. In other words, a pulmonary capillary wedge pressure of 5–10 does not rule out fluid overload as a cause of pulmonary or GI dysfunction. In critically ill patients, the fear of hypotension and effect of perfusion usually results in infusion of intravenous salt and water in quantities which exceed losses. Consequently, most patients in the Intensive Care Unit have anemia, dilutional hypoproteinemia, and a compensatory increase in cardiac output. In response to anemia, these patients are tachycardic, even though blood volume is normal, filling pressures are normal, and total body extracellular fluid is excessive.


237 Which of the following statement(s) is/are true concerning methods of nutritional support?

a. Optimal results for enteral feedings are achieved with approximately half of calories supplied as carbohydrate and half as fat
b. Diarrhea is the most common complication of enteral feeds and is due to the high osmolarity of the carbohydrate components
c. The hyperosmolar nature of parenteral fat solutions requires central venous administration
d. Approximately 25–50% of calories should be provided as fat emulsion in patients receiving total parenteral nutrition
Answer: a, b, d

Most formulas for enteral feeding range from 1.0 to 2 cal/ml and include 3 to 7% protein. Most of the calories are supplied as glucose or sucrose, so that the solutions have a very high osmolarity. Cramps or diarrhea can result when these high osmolar solutions are placed into the stomach or intestine. Diarrhea is the major complication with most tube feeding formulas. Diarrhea can be minimized by the use of starch or fat as an energy source in tube feedings. This can be supplied as part of the commercial preparation or added in the form of medium chain triglycerides or other oils. The best results are usually achieved by supplying approximately half the calories as carbohydrate and half as fat. In patients receiving total parenteral nutrition, energy source is provided as carbohydrate, fat, and amino acid solutions. Parenteral feeding with carbohydrate is limited by the sclerotic effect of hyperosmolar solutions on veins. Fat is a more efficient energy source and can be given through peripheral veins in concentrations of either 10 or 20%. Most intensivists favor supplementing standard total parenteral nutrition solution with intravenous fat to provide at least 100 grams of fat emulsion each week to preclude fatty acid deficiency. Giving up to 25 to 50% of calories each day as fat emulsion may optimize the delivery of this caloric delivery.


238 Which of the following statement(s) is/are true concerning the autoregulation necessary to maintain oxygen consumption and oxygen delivery?

a. A change in oxygen consumption is followed by a proportionate change in oxygen delivery
b. A change in oxygen delivery is followed by a change in oxygen consumption
c. Increases in oxygen delivery are due solely to an increase in cardiac output
d. The normal ratio of oxygen delivery to consumption is 2:1
Answer: a

The relationships between oxygen consumption and oxygen delivery represent one of the most interesting regulation systems in homeostasis. First of all, if one of the three components of oxygen delivery is abnormal, endogenous mechanisms regulate the other two until normal oxygen delivery has been restored. The various combinations of compensatory mechanisms supply adequate oxygen for systemic metabolism through a wide range of variations in oxygen delivery. When there is a change in oxygen consumption, there is a proportionate change in oxygen delivery, which occurs almost immediately, mediated completely by a change in cardiac output. Conversely, a primary change in oxygen delivery is not followed by any change in oxygen consumption. The normal ratio of oxygen delivery to consumption is approximately 5:1.


239 Which of the following statement(s) is/are true concerning O2 venous saturation monitoring?

a. The normal saturation of mixed venous blood is 50%
b. Mixed venous blood obtained for saturation monitoring can be obtained from any peripheral vein
c. If arterial blood is fully saturated, the saturation of mixed venous blood is 80%
d. In less than fully saturated blood, the difference between arterial and venous saturation corresponds to oxygen extraction
Answer: c, d

The relationship between oxygen delivery and oxygen consumption is reflected in the amount of oxygen in venous blood. Under normal circumstances, oxygen delivery is 1000 cc/min and oxygen consumption is 200 cc/min. The amount of oxygen extracted is 20% of that delivered, and 80% of oxygen is still present in venous blood returning to the heart. Usually arterial blood is fully saturated, and under normal circumstances, the saturation of mixed venous blood (SVO2) will be 80%. This measurement must be made in mixed venous blood since the relative extraction of organs served by the superior and inferior vena cava and coronary sinus are quite different. As long as arterial blood is fully saturated, this observation holds true regardless of the absolute level of oxygen consumption or oxygen delivery. If the arterial blood is less than fully saturated, the difference between arterial and venous saturation corresponds to the oxygen extraction, hence the oxygen delivery/oxygen consumption ratio.


240 Which of the following statement(s) is/are true concerning oxygen kinetics in a critically ill, febrile patient?

a. Oxygen consumption will likely exceed three times normal
b. The high cardiac output and pulse rate are designed to increase oxygen delivery
c. The hyperdynamic response may actually increase oxygen delivery to exceed the increase in oxygen consumption
d. The patient can maintain adequate compensation as long as the oxygen delivery/oxygen consumption rate is greater than 2:1
Answer: b, d

In critically ill patients oxygen consumption may be elevated or depressed, but slight to moderate elevations in oxygen consumption is the most common abnormality in critically ill patients. Oxygen consumption will be elevated in proportion to the amount of inflammation. A febrile patient with significant signs of septic toxicity will typically have an oxygen delivery at 1.5 to 2 times normal. It is very unusual for a critically ill patient to experience oxygen consumption greater than twice normal. This occurs only in situations of severe muscular exercise such as seizures or tetanus. During hypermetabolism, a change in oxygen consumption is followed promptly by a proportionate change in oxygen delivery. Hence, it is “normal” for a hypermetabolic patient to have a high cardiac output and pulse rate. Rarely the hyperdynamic response exceeds the increase in oxygen consumption, reflected in a ratio higher than 5:1 and venous saturation greater than 80%. Some patients cannot mount an increased oxygen delivery in response to increased oxygen consumption because of the combination of hypoxemia, anemia, and myocardial failure. If this occurs, then the oxygen delivery/oxygen consumption ratio will be less than 5:1. The patient will compensate for this by increased oxygen extraction, however, and the patient will remain stable as long as the ratio is greater than 2:1.


241 Which of the following statement(s) is/are true concerning the treatment of pulmonary interstitial edema?

a. Diuresis and blood transfusion is a valuable step
b. Salt-poor albumin leaks through the capillaries and worsens the condition
c. Mannitol is contraindicated as a diuretic in this clinical situation
d. Isoproterenol is a poor choice as an ionotropic agent
Answer: a

Treatment of pulmonary edema has two important goals, the first is to improve oxygenation if it is impaired, and the second is to minimize fibrosis and bacterial infection, which often accompany pulmonary edema caused by capillary injury. The treatment of interstitial edema is to maintain the hydrostatic pressure as low as compatible with adequate cardiac output and to raise the oncotic pressure selectively in the vascular space. These measures, combined with fluid restriction and diuresis, will decrease the amount of pulmonary edema. Since it is desirable to maintain filling pressures of the left ventricle as low as possible while maintaining a good cardiac output, inotropic drugs to improve left ventricular contractility are helpful. Isoproterenol or dopamine should be used, with serial cardiac output and filling pressure measurements. The first step in decreasing pulmonary edema is to decrease the pulmonary capillary hydrostatic pressure as low as is compatible with an adequate cardiac output. This is done by diuresis and fluid restriction. As the patient falls behind in blood volume, signs of hypovolemia may appear. Blood volume is then replenished with a fluid that stays in the vascular space. Packed red cells are ideal for this application. When the hematocrit is normal, concentrated salt-poor albumin should be used. This hyperoncotic fluid replenishes the blood volume by attracting interstitial fluid from throughout the body into the vascular space and supplements diuresis. This technique is useful even in the septic patient who may have increased capillary permeability and may loose albumin from the vascular space at a rapid rate. Even if albumin “leaks out”, the short term effects of expanding blood volume and decrease in edema will appear.


242 Which of the following statement(s) is/are true concerning the pathophysiology of gas exchange?

a. Hypoventilation in relation to perfusion can result in an oxygen saturation of less than 100%
b. Diffusion block and / mismatch can almost completely be overcome by breathing 100% oxygen
c. Transpulmonary shunting does not occur under normal circumstances
d. The normal arterial oxygen saturation should be 100%
Answer: a, b

Under normal conditions, red blood cells in the pulmonary capillaries become fully saturated and oxygen dissolves in plasma resulting in blood PO2 of 100 and O2 saturation of 100%. This equilibration may be disturbed by hypoventilation in relationship to the perfusion (/ mismatch), diffusion block caused by interstitial fibrosis, or perfusion of nonventilated alveoli. Diffusion block and / mismatch can almost be completely overcome by breathing 100% oxygen, hence hypoxemia during exposure to high alveolar PO2 is caused by total / mismatch, so-called transpulmonary shunting or venous ad mixture. Under normal circumstances, about 5% of the blood entering the left atrium has been shunted away from the pulmonary capillaries, either as the result of bronchial nutritive blood flow or through thebesian veins opening directly into the left side of the heart. This phenomenon, combined with a normal minor / mismatch associated with breathing at rest and positional changes in pulmonary blood flow, result in the fact that normal arterial PO2 is 90–100 mm Hg and the normal O2 saturation is 98%.


243 Which of the following statement(s) is/are true concerning CO2 transfer in the lung?

a. Carbon dioxide excretion is a direct function of alveolar ventilation
b. Normally end tidal CO2 should be identical to PaCO2
c. The gradient between end tidal and arterial CO2 can be an indirect measure of nonperfused alveoli
d. Positive pressure ventilation under normal airway pressures creates a significant end tidal PaCO2 gradient
Answer: a, b, c

The amount of carbon dioxide excretion is directly related to alveolar ventilation. While oxygenation is a function of matching blood flow to alveoli, carbon dioxide excretion is a direct function of ventilation or hyperventilation of alveoli with some blood flow. Normally the end tidal CO2 represents mixed alveolar gas which is at equilibrium with pulmonary capillary blood, hence with arterial blood. Therefore, the end tidal CO2 and the PaCO2 should be identical. End tidal CO2 measurement is a very useful continuous measurement of PaCO2 which can be used as a monitor when the lung is normal, as in ventilator weaning. Furthermore, the gradient between end tidal and arterial CO2, when it is large, acts as an indirect measure of nonperfused alveoli and/or compression volume. In patients who are ventilated with positive pressure ventilation, a significant end tidal PaCO2 gradient occurs only when peak airway pressures are very high (over 30 cm H2O) and the compression volume is a significant component of each exhaled breath.


244 Which of the following statement(s) is/are true concerning the assessment of protein reserve?

a. Conventional serum proteins such as albumin and globulin are early indicators of malnutrition
b. The total lymphocyte count reflects immune status and not nutrition
c. Antigen skin testing reflects patient immunity and not nutrition
d. Measurement of urea excretion in urine can be used as a measurement of protein breakdown
Answer: d

Since protein is the functional and structural chemical of the body, most nutritional assessment techniques are estimates of protein reserves. The actual nitrogen balance can be measured by measuring the amount of nitrogen excreted. This is most conveniently done by measuring the amount of urea excreted in the urine, assuming that urea constitutes 85% of the total nitrogen excretion. Knowing nitrogen excretion, the amount of protein catabolized can be estimated and compared with the amount of protein ingested by the patient. Indirect assessments of protein reserves are based on single measurements of body substances that are dependent on rapid protein synthesis for maintenance of normal levels. Conventional serum proteins such as albumin and globulin are not affected by malnutrition until it is very severe. Proteins such as prealbumen and transferrin, which turn over more rapidly, are better indicators of protein status. Lymphocytes are rapidly destroyed and protein is required for the formation of new cells. Consequently, the absolute lymphocyte count is a useful measure of the status of protein reserves. The lymphocyte count is considered by some the best single “static” measurement characterizing nutritional status. Protein is also required for synthesizing the cells and mediators involved in skin test reactivity. Although skin test reactivity is a manifestation of lymphocyte-mediated immunity, its usefulness in patient assessment is probably that of assessment of the inflammatory response than lymphocyte activity per se. Some chronically and acutely malnourished patients convert from reactive to anergic, and reactivity can be restored by nutritional repletion.


245 Useful steps to optimize systemic oxygen delivery include:

a. Maintaining mean arterial blood pressure between 50 and 90 mm Hg
b. Optimizing PEEP levels by monitoring mixed venous saturation
c. Turning the patient prone
d. Sedation or paralysis
Answer: a, b, c, d

Optimizing systemic oxygen delivery in relationship to oxygen requirement is the primary goal of management. Improving oxygenation of the blood itself by improving alveolar inflation is only one of the steps in optimizing oxygen delivery. Equally or more important are treating anemia and optimizing cardiac output. Cardiac output should be optimized to maintain delivery of four to five times consumption. In general, this means avoiding those factors which decrease cardiac output, rather than actively trying to increase cardiac output. Blood pressure should be maintained high enough to provide coronary perfusion (over 50 mm Hg mean pressure) but not so high as to limit left ventricular function (over 90 mm Hg mean arterial pressure). Alveolar collapse is treated by cleaning the airways, avoiding 100% oxygen, and moving fluid from the lung or chest, and finally by the use of positive end exploratory pressure to hold open those alveoli which have been opened by other measures. The optimal level of PEEP is that level that maintains arterial oxygenation but does not decrease venous return or cardiac output. This optimal level is best determined by monitoring mixed venous saturation. Another step in optimizing lung function is to take advantage of the gravitational effects on pulmonary blood flow by turning the patient prone or to a full lateral position to direct blood flow to areas of optimal alveolar function. This step will often result in an opening in the closed posterior alveoli which have been compressed by the weight of the fluid in the lungs. At the same time that oxygen delivery is optimized, oxygen consumption should be decreased to normal or even below normal if necessary. Treating infection, providing adequate sedation, and establishing muscular paralysis decrease oxygen consumption, and decrease the need for oxygen delivery.


246 Phases of multiorgan failure will include:

a. Generalized increased capillary permeability
b. A hypermetabolic state
c. Organ malfunction
d. All of the above
Answer: d

Clinically the multiple-organ failure patient progresses through well-defined phases. These phases include: Phase 1—a generalized increased capillary permeability resulting in edema, weight gain, and intravenous volume replacement, increased protein concentration in urine and lymph. Although the pulmonary microvasculature has been most thoroughly studied, it is apparent that the lung is simply the most obvious end organ in a generalized permeability defect. Phase 2—A hypermetabolic state, with increased oxygen consumption and a compensatory increase in oxygen delivery characterized by tachycardia and high cardiac output. This condition following systemic ischemic and reperfusion is similar to hypermetabolism following endotoxemia, localized sterile inflammation, and infusion of stress hormones, suggesting a common mechanism. Phase 3—Organ malfunction due to localized edema and cellular injury, particularly in the kidney, liver, brain, and host defense system. Hemorrhagic shock predisposes to bacterial translocation and endotoxin absorption from the intestine. Phase 4—In the absence of systemic sepsis, organs may recover to normalcy or may be irreversibly damaged, leading to a need for chronic support. If the organ failure phases lead to systemic infection or irreversible tissue damage in the lung or brain, the death of the entire organ is likely.


247 Which of the following statement(s) is/are true concerning oxygen consumption (O2)?

a. O2 is normally 100–120 cc2/m2/min
b. Resting O2 is controlled by the level of thyroid and catecholamine hormones
c. Under steady state conditions the amount of oxygen consumed exceeds the amount of oxygen taken up by the pulmonary capillaries
d. O2 is dependent on the status of pulmonary function
Answer: a, b

Oxygen consumed in the process of metabolism is expressed as the volume of oxygen per minute (O2). O2 is normally 100–120 cc2/m2/min, or 200 cc/min for a typical adult. Resting O2 is a function of metabolizing body cell mass, with fine tuning control provided by the level of thyroid and catecholamine hormones. O2 decreases under conditions of hypothermia, paralysis, and hypothyroidism. O2 increases during exercise or muscular activity, hyperthermia, profound hypothalamic injury, hyperthyroidism, catecholamines, and inflammatory mediators, particularly the interleukin cytokines. Under steady state conditions, the amount of oxygen consumed in systemic metabolism is exactly equal to the amount of oxygen taken up by the pulmonary capillaries via the airway. This is true regardless of the status of pulmonary function or dysfunction, so we measure O2 across the lung and assume that it is exactly the amount consumed in the systemic metabolism.


248 Which of the following statement(s) is/are true concerning the outcome in patients with acute renal failure?

a. Mortality for ischemic acute tubular necrosis without other organ failure is approximately 6%
b. Multiple organ failure complicated with acute renal failure is associated with mortality ranging from 50% to 90%
c. Recovery of renal function after six weeks is unlikely
d. There is no difference in survival between oliguric and nonoliguric renal failure
Answer: a, b, c

Survival of patients with acute renal failure is a function of the successful treatment of the primary disease from which the renal failure was derived. The mortality for ischemic acute tubular necrosis without organ failure has been reported at approximately 6%. By contrast, mortality of multiorgan failure complicated by acute renal failure ranges from 50% to 90%. In patients who survive the acute phase of illness, recovery of renal function after acute renal failure is dependent on the type and extent of injuries to the renal parenchyma. If renal function is not returned after six weeks, recovery is unlikely. Nonoliguric renal failure is usually limited in its extent and is almost always reversible.


249 Which of the following statement(s) is/are true concerning oxygen delivery?

a. The amount of oxygen delivered to peripheral tissues is dependent upon the oxygen content in arterial blood and cardiac output
b. Oxygen content is commonly measured in arterial blood
c. The normal arterial-venous difference is 4 cc O2/dL
d. Normal systemic oxygen delivery for a typical adult is approximately 1000 cc/min
Answer: a, c, d

The amount of oxygen that is delivered to peripheral tissues is the product of the oxygen content in arterial blood times the cardiac output. Normally, oxygen content of arterial blood is approximately
20 cc/dL, and the normal cardiac index is 5 L/min. Therefore, the normal systemic oxygen delivery is approximately 1000 cc/min. Although oxygen content is the most important measure of oxygen in the blood, PO2 and oxyhemoglobin saturation is more commonly measured in the Intensive Care Unit, hence it is necessary to convert between these measurements. The normal oxygen content of venous blood is 16 cc/dL. Hence, the normal arterial-venous difference is 4 cc O2/dL.


250 Which of the following statement(s) is/are true concerning carbon dioxide kinetics?

a. The amount of carbon dioxide produced is equivalent to the amount of oxygen consumed
b. Carbon dioxide levels in blood, present mostly as a bicarbonate ion, can quickly change
c. Normally the amount of carbon dioxide excreted through the lung is exactly equal to the amount of carbon dioxide produced in peripheral tissues
d. The amount of carbon dioxide excreted is a function of ventilation of perfused alveoli
Answer: a, c, d

The total amount of carbon dioxide produced by systemic metabolism is roughly equivalent to the amount of oxygen consumed (100–120 cc/m2/min, 200 cc/min in a typical adult). CO2 production is increased or decreased by each of the factors that causes an increase or decrease in oxygen consumption. Most of the carbon dioxide in blood is present as bicarbonate ion which cannot be changed quickly. However, the metabolically produced CO2 is mostly present as dissolved carbon dioxide, added to the blood in the peripheral tissues and excreted in the lung. In a steady state, the amount of carbon dioxide excreted through the lung is exactly equal to the amount of carbon dioxide produced in peripheral tissues. The amount of carbon dioxide excreted is a function of ventilation of perfused alveoli (i.e. the alveolar ventilation/min).


251 Which of the following result in a decrease in functional residual capacity?

a. Shallow breathing
b. Partial airway occlusion
c. Absorption atelectasis
d. Hemothorax
Answer: a, b, c, d

A decrease in functional residual capacity is caused by incomplete alveolar inflation related to 1) shallow breathing; 2) partial or complete airway occlusion, which may be generalized (as in bronchospasm) or localized (as in gastric aspiration); 3) absorption atelectasis, which occurs when oxygen is substituted for nitrogen in the inspired gas; or 4) conditions in which air or fluid is occupying a potential alveolar space in the chest such as pneumothorax, hemothorax, or pulmonary edema.

252 Which of the following statement(s) meet the criteria for organ failure?

a. Bilirubin greater than 5 mg/dl
b. Creatinine greater than 3 mg/dl
c. Alveoloarterial O2 gradient greater than 300 mm Hg
d. Glasgow Coma score less than 10
Answer: a, b, c, d

Multiple organ failure is defined by dysfunction of two or more of the six vital organ systems: cardiovascular, respiratory, nervous system, renal, liver, and host defenses.


253 Which of the following statement(s) is/are true concerning pulmonary edema?

a. Pulmonary edema effectively narrows bronchi and increases pulmonary vascular resistance
b. Ventilation and perfusion are decreased equally
c. Positive pressure ventilation improves gas exchange by decreasing lung edema
d. The condition is frequently caused by decreased plasma protein levels
Answer: a

The causes of pulmonary edema are: 1) increased hydrostatic pressure; 2) increased capillary permeability and 3) decreased plasma oncotic pressure. The latter, however, is rarely a problem unless the concentration of plasma protein is very low. When fluid begins to collect in the lung interstitium, it migrates to the loose areolar portion of the lung microanatomy that surround the small bronchioles and pulmonary arteries. The edema in these areas has the effect of narrowing bronchi and increasing resistance in the pulmonary vasculature. This will decrease both ventilation and perfusion in the edematous area, but ventilation is often affected more than blood flow, resulting in a decreased / ratio, with all of its attendant effects on gas exchange. Ventilator treatment of pulmonary edema which increases airway pressure tends to hold the alveoli open, spreading out the space available for water accumulation and overcomes the effect of small bronchial occlusion. Positive pressure ventilation does not, therefore, affect the amount of edema in the lung, only its manifestations.


254 Which of the following statement(s) is/are true concerning various causes of acute renal failure?

a. Acute tubular necrosis is the most common pathologic finding of acute renal failure
b. Drug-induced renal failure is compounded in situations of hypovolemia
c. Myoglobin-induced renal failure can be prevented using diuretics and alkalization of urine
d. The incidence of radiographic contrast dye-induced renal failure occurs independent of preexisting conditions
Myoglobin is a direct nephrotoxin
Answer: a, b, c

Acute tubular necrosis results from ischemia to the renal parenchyma and is the most common pathologic finding of acute renal failure. In conditions of diminishing renal blood flow, perfusion to the kidneys is first maintained by vasomotor responses which dilate the afferent arteriole and constrict the efferent arteriole. As continued hypotension occurs, the renin-angiotensin system is activated and vasoconstriction of the afferent arteriole occurs which exacerbates corticohypoperfusion. Pigment nephropathy is a common cause of acute renal failure occurring after trauma, burns, operations, or hemodynamic catastrophe. With ischemia or blunt injury to large muscles, myoglobin is released into the circulation. In the kidney, it is filtered from blood and reabsorbed by the tubule. Although myoglobin is not a direct nephrotoxin, in the presence of aciduria, myoglobin is converted to ferrihemate, which is toxic to renal cells. Prevention of myoglobin-induced renal failure may include the use of diuretics and alkalinization of urine. Drug-induced acute renal failure is responsible for approximately 5% of all cases of acute renal failure. Through normal reabsorption and secretion, the kidney is exposed to high concentrations of drugs and solutes, which may be toxic. This problem is compounded by hypovolemia, which causes increased reabsorption of water and solutes and exposes the lumen to even higher concentrations of toxins. The incidence of radiographic contrast dye induced nephropathy is approximately 1 to 10% and may be predicted according to a number of risk factors which include contrast load, age, preexisting renal insufficiency, and diabetes. The incidence in patients with normal renal function is significantly lower at 1% to 2%.


255 The patient requires renal replacement therapy. Which of the following statement(s) is/are true concerning the differences between hemodialysis and continuous arteriovenous hemodialysis (CAVHD)?

a. Anticoagulation is not required for CAVHD
b. Hemodynamic instability will be a particular problem with both techniques
c. Both techniques will decrease serum urea ni+62trogen levels
d. CAVHD will likely result in better removal of excessive volume
Answer: c, d

256 Which of the following statement(s) is/are true concerning continuous arteriovenous hemofiltration (CAVH)?

a. The technique runs continuously
b. It is not associated with the hemodynamic instability
c. Systemic heparin anticoagulation is necessary
d. Fluid balance and correcting electrolyte abnormalities takes several days
Answer: a, b

Continuous arteriovenous hemofiltration (CAVH) is an extracorporial filtration technique that removes extracellular fluid across a synthetic membrane via hydrostatic pressure gradient created between the indwelling arterial and venous catheters. Arteriovenous access is accomplished by percutaneous cannulation of femoral artery and vein with a low incidence of complications. Although full systemic anticoagulation is not necessary for CAVH, heparinization of the extracorporial circuit is required. CAVH is run continuously for as many days as renal replacement is required. Experience with CAVH has demonstrated very little or no hemodynamic instability with treatment of critically ill renal failure patients. The stable nature of this therapy is attributed to a slow and continuous fluid and solute removal and to the fact that the membrane does not induce compliment activation when in contact with blood. Fluid balance and serum electrolyte concentrations can be titrated to any level in a matter of hours by manipulating the composition and rate of replacement solution. Solute clearance with CAVH is limited by the ultrafiltration and replacement fluid exchange rate. In patients with high urea generation rates, solute removal with CAVH may be inadequate and variations of the technique may be used to enhance clearance.


257 A 64-year-old diabetic patient develops acute renal failure following an aortic aneurysm repair. Which of the following statement(s) is/are true concerning his diagnosis and management?

a. Resting energy expenditure will likely be less than would be expected for a patient with normal renal function
b. Maintenance of positive energy balance reduces protein catabolism and makes the management of renal failure easier
c. Expected metabolic abnormalities include hyperkalemia, hypercalcemia, and metabolic alkalosis
d. A nonoliguric renal failure is usually associated with a better outcome
Answer: b, d

In patients with nonoliguric renal failure, treatment may differ little from that required for identical patients with normal renal function. Management of fluids, solutes, and nutrition is usually unaffected by nonoliguric renal failure, although BUN may be elevated. The extent of renal dysfunction is limited and almost always reversible. The use of renal replacement therapies is rarely necessary. Acute renal failure can result in severe derangements in electrolyte and acid-based physiology. Of all electrolyte abnormalities that might occur, hyperkalemia is the most serious. Other electrolyte abnormalities such as hyponatremia, hyperphosphatemia, hypocalcemia, and metabolic acidosis are common and must be monitored carefully.
The metabolic requirements of a patient with acute renal failure are those of a critically ill hospitalized patient. The actual measurements of resting energy expenditure has shown that caloric requirements for multiorgan failure patients with renal failure are often 50% above normal, healthy individuals. Although acute renal failure may require fluid restriction, providing adequate nutrition is an important aspect of their treatment. Positive energy balance may make management of uremia and hyperkalemia less difficult. By providing adequate calories, endogenous protein catabolism with resultant generation of urea and release of potassium can be avoided. Maintenance of positive energy balance with glucose and lipids should reduce protein catabolism, urea generation, and hyperkalemia.


258 Which of the following statement(s) is/are true concerning various energy sources?

a. Carbohydrate is the most efficient source of energy
b. Endogenous fat is the major source of energy during starvation
c. The respiratory quotient of carbohydrate is greater than either fat or protein
d. Ketones can be used as a source of energy during starvation
Answer: b, c, d

The major sources of energy are carbohydrates and fats. Carbohydrates are a major source of energy during normal, non-starving existence. The brain, the red cells, and some other organs are obligate glucose users. The brain and red cells can develop the capacity to use ketones as an energy source, a process known as starvation adaptation. Fat is the most efficient source of energy. Fat produces 9 calories of energy per gram of substrate metabolized while carbohydrate produces only 4 calories. The respiratory quotient represents the number of molecules of carbon dioxide for each molecule of oxygen consumed. For carbohydrates it is 1.0, whereas for fat, this respiratory quotient is 0.7. Endogenous fat is the major source of energy during starvation. The glycogen storage is basically depleted after a day of fasting and fat becomes a major energy source with protein breakdown supplying glucose through the process of gluconeogenesis.


259 Which of the following statement(s) is/are true concerning the treatment of multisystem organ failure?

a. Forced diuresis with negative fluid balance may improve survival and acute respiratory failure
b. The titration of ionotropic drugs based on desired blood pressure optimizes the results.
c. Nutritional support should be withheld for several days until the patient’s condition stabilizes
d. Continuous arteriovenous hemofiltration is preferred to intermittent hemodialysis for most critically ill patients
e. Hepatic failure should be treated specifically with pharmacologic manipulation
Answer: a, d

The important principles in the management of multiple organ failure are to avoid further episodes of local or systemic ischemia and to keep the brain viable by pharmacologic or mechanical support of the failing organs until organ recovery occurs. Respiratory failure is treated by mechanical assistance for lung inflation and ventilation and by decreasing lung edema as much as possible. Airway intubation is usually required. There is now good evidence that forced diuresis and negative fluid balance is associated with improved survival and acute respiratory failure. Cardiac failure is treated with inotropic drugs. Although ionotropic drugs are usually titrated to achieve a desired arterial blood pressure, it is more sensible to titrate ionotropes to achieve a normal oxygen delivery/oxygen consumption ratio. Pulmonary artery pressure and mixed venous saturation monitoring are essential for intelligent management of the patient with severe respiratory or cardiac failure. Adequate nutrition is also important for recovery from organ failure. Renal failure is treated by mechanical substitution of renal function. Although hemodialysis and peritoneal dialysis can serve this purpose, each has a significant drawback in the critically ill, multiple organ failure patient. Continuous arteriovenous hemofiltration (CAVH) and continuous arteriovenous hemodialysis (CAVHD) are the methods of choice for renal replacement therapy. Hepatic failure often occurs as part of the multiple organ failure syndrome but unfortunately there is no specific treatment.


260 Which of the following statement(s) is/are true concerning pulmonary mechanics?

a. The standard compliance or volume pressure curve is measured during lung inflation
b. The decreased compliance in acute respiratory failure occurs because the lung is smaller not stiffer
c. In acute respiratory failure, higher pressures are required to achieve the same level of inflation
d. Areas of normal lungs are more vulnerable to overdistention which may lead to progressive lung dysfunction
Answer: b, c, d

The standard compliance or volume pressure curve is drawn by measuring volume and pressure at stages of lung deflation after total inflation. The decreased compliance in acute respiratory distress syndrome occurs because the lung is smaller, not stiffer. In acute respiratory failure, the cause of decreased compliance is almost always associated with a decrease in functional residual capacity (FRC). The decreased FRC represents lost alveoli which are either collapsed or filled with fluid but still perfused with blood. Because the lung is smaller, the compliance curve has shifted to the right and much higher pressures are required to achieve the same level of inflation. Lung damage can be caused by high airway pressure, so that overdistension is not merely inefficient but actually detrimental. Since the most normal areas of lung have the best compliance, they are most vulnerable to overdistension, contributing to the steady progression of lung dysfunction in patients ventilated at high peak pressure.


261 Which of the following statement(s) is/are true concerning the use of a ventilator in the treatment of respiratory failure?

a. The assist-control mode is appropriate in the paralyzed patient
b. Peak inspiratory pressure should be optimized at a level in excess of 40 cm H2O
c. A patient receiving excessive carbohydrate as a nutritional support may have an elevated minute ventilation and may tire with spontaneous breathing
d. In general, weaning requires an adequate inspiratory force, vital capacity, and a minute ventilation less than 10 L/min
Answer: c, d

Most intensivists favor setting the ventilator on the assist-control mode at a low sensitivity. In this fashion, the patient breathes at a rate that regulates the PaCO2 to normal, but each breath is mechanically assisted, providing maximal inflation. The volume of each breath is set by limiting the maximal pressure or maximal volume of each breath. Whichever method is used, the peak inspiratory pressure should not generally exceed 40 cm H2O. If the patient is comatose or paralyzed, the assist mode cannot be used and the rate is set in addition to the volume.
Adequate weaning indices are: inspiratory force greater than 20 cm H2O, vital capacity twice the tidal volume, adequate gas exchange at assisted ventilation at FiO2 of 0.3 and 5 cm H2O of PEEP, and minute ventilation less than 10 L/min. If the patient is hypermetabolic or is receiving excessive carbohydrate as nutritional support, the minute ventilation will be elevated, even during assisted mechanical ventilation. If this is the case, the patient will tire rapidly on spontaneous breathing.


262 Which of the following statement(s) is/are true concerning the estimation and measurement of energy requirements in the critically ill patient?

a. One can only estimate energy expenditure with actual measurement not technically possible
b. The amount of oxygen absorbed through the lungs is equal to the amount of oxygen consumed by metabolic processes
c. Metabolic rate, normalized to body surface area, may underestimate metabolism in a fat person
d. To convert cc’s of oxygen per minute to calories per day, a conversion factor of 10 kcal of energy per liter of oxygen should be used
Answer: b, c

The actual metabolic rate of any patient can be estimated from the predicted basal rate according to the clinical situation. The amount of energy is most conveniently expressed in calories/day. The metabolic rate is normalized to body surface area; however, the actively metabolizing tissue is the lean body cell mass. Consequently, reporting “per square meter” underestimates metabolism in a fat person and overestimates in a very lean person. Although most of studies on nutrition in critical illness have been based on estimated energy expenditure, actual measurement is much more accurate and has become an important aspect of critical care management. The most commonly used method of measurement is indirect calorimetry. In this method, the amount of oxygen absorbed across the lungs into the pulmonary blood is measured over a given period of time. Assuming the patient is at a metabolic steady state during this time, the amount of oxygen absorbed across the lungs is equal to the amount of oxygen consumed in the metabolic process. The metabolic rate, measured in cubic centimeters of oxygen/minute, can be converted to calories/hour or /day if the oxygenated substrates are known. For practical purposes, a conversion factor of 5 kcal of energy/liter of oxygen consumed is a reasonable approximation.


263 Which of the following statement(s) is/are true concerning the response to a decrease in functional residual capacity percent (FRC)?

a. Supplying supplemental oxygen will always improve the situation
b. Respiratory alkalosis may occur
c. Decreasing compliance is a common occurrence
d. Respiratory rate and depth of breathing generally decrease
Answer: b, c

Pulmonary arterial spasm in response to local hypoxia autoregulates pulmonary blood flow and maintains adequate gas exchange during alveolar collapse—up to a point. However, when the loss in ventilation exceeds the decrease in perfusion, a ventilation-perfusion mismatch occurs, which results in incomplete oxygenation of blood perfusing that area of the lung. The resultant hypoxemia stimulates an increased rate and depth of breathing which may serve to reexpand the person’s inflated area of lung. If it does not, hypoxemia will continue but increased ventilation in other areas of the lung will result in excess CO2 excretion, hypocapnea and respiratory alkalosis. The blood gas picture, hypoxemia with respiratory alklalosis, is the most common abnormality of gas exchange in surgical patients and it is a hallmark of ventilation-perfusion imbalance. Oxygenation of blood in the poorly ventilated area of lung can be improved by increasing concentration of oxygen in the inspired gas. The use of supplemental oxygen, however, treats the symptom rather than the basic cause and may actually make the problem worse by adding to absorption atelectasis, depriving the poorly ventilated area of nitrogen to hold alveoli open. This may result in total alveolar collapse. In this circumstance, blood perfusing the nonventilated area will mix with blood from other areas of the lung, resulting in hypoxemia that does not improve significantly in response of administration of oxygen. Aside from the effects on gas exchange, loss of alveolar space results in changes in the volume-pressure relationships in the lung. A decrease in functional residual capacity always results in a shift in the volume-pressure relationship toward a condition of decreasing compliance.

264 Which of the following statement(s) is/are correct concerning the body fluid compartments?

a. Both the extracellular and intracellular components of total body water can be directly measured
b. The intravascular space accounts for the majority of extracellular fluid
c. All water in the interstitial space is freely exchangeable
d. Transcellular fluid, separated from other compartments by both endothelial and epithelial barriers, constitute about 4% of total body water
Answer: d

Total body water (TBW) is distributed within the intracellular and extracellular compartments. Intracellular fluid cannot be measured directly but is calculated as the difference between TBW and the measured extracellular water. Extracellular fluid can be measured directly. The extracellular fluid compartment can be further simplified into the intravascular and interstitial spaces. Intravascular space, which accounts for 20% of the extracellular fluid, contains the plasma volume which is approximately 8% of TBW or 5% of body weight. The interstitial space extends from the blood vessels to the cells themselves and includes the complex ground substance making up the acellular matrix of tissue. Although the water within the space is thought to be freely exchangeable, this water exists in two phases. The free phase contains water that is generally freely exchangeable and in a constant state of flux. The bound or gel phase is composed of water that is closely associated with glycosaminoglycans, mucopolysaccharides, and other matrix components. This water is much less freely exchangeable. An additional extracellular fluid compartment, the transcellular compartment, consists of water that is poorly exchangeable under normal circumstances. This fluid is separated from other compartments by both endothelial and epithelial barriers and includes cerebrospinal fluid, synovial fluid, water within cartilage and bone, fluids of the eye, and the lubricating fluids of the serous membranes. Together, these fluids constitute about 4% of TBW.


265 Which of the following statements(s) is/are true concerning metabolic alkalosis?

a. Either increased extracellular bicarbonate concentration or inhibited renal excretion of bicarbonate can cause metabolic alkalosis
b. In metabolic alkalosis secondary to prolonged gastric outlet obstruction, the urine pH is usually acidic
c. Hypokalemia can lead to metabolic alkalosis
d. The respiration compensatory mechanisms for metabolic alkalosis are quite ineffective
Answer: b, c, d

Sustained metabolic alkalosis occurs only if extracellular bicarbonate concentration is increased and renal excretion of excess bicarbonate is inhibited. Alone, neither is sufficient to result in metabolic alkalosis. Extracellular bicarbonate concentration is increased by numerous mechanisms. Loss of HCl is the leading cause of metabolic alkalosis in surgical patients. External loss of gastric acid results in net gain in bicarbonate, which causes metabolic alkalosis. Although the kidney can excrete excess bicarbonate, this must be accompanied by excretion of sodium. Renal excretion of sodium is limited in the face of volume depletion, which also occurs with external losses of gastric secretion. As volume depletion progresses, sodium is conserved in exchange for hydrogen. Thus, in metabolic alkalosis secondary to prolonged gastric outlet obstruction, the urine, although initially alkalotic, becomes paradoxically acidotic in prolonged or uncorrected cases. Hypokalemia and cellular exchange of potassium for hydrogen can also lead to metabolic alkalosis. Hypokalemia results in enhanced proximal tubular bicarbonate reabsorption and distal tubular acid secretion. The major compensatory mechanism in metabolic alkalosis is respiratory, since the presence of metabolic alkalosis implies renal dysfunction in either generating or failing to excrete increased amounts of bicarbonate. Hypoventilation is limited by the development of hypoxemia, which stimulates ventilation. Among the four major types of acid-base disorders, this compensatory mechanism is the least effective.

266 Which of the following statement(s) is/are true concerning respiratory acidosis?

a. Respiratory acidosis is associated with chronic pulmonary disease far more commonly than is hypoxemia
b. The initial buffering effect occurs at the cellular level
c. Renal compensation occurs within 24 hours
d. Correction of hypoxemia in patients with chronic lung disease may worsen respiratory acidosis
Answer: b, d

Respiratory acidosis, the decrease in extracellular pH from a primary increase in PCO2, is due to inadequate ventilation. Although pulmonary disease commonly causes hypoxemia, respiratory acidosis is far less common, since defusion of O2 is more readily impaired than diffusion of CO2. Increased PCO2 results in increased H2CO3 which disassociates into H+ and HCO3–. Cellular exchange of Na+ and K+ for H+ allows the reaction to continue in this direction with increased extracellular bicarbonate. This tissue buffering is accomplished within minutes. Persistently elevated PCO2 also stimulates increased renal acid excretion. Full renal compensation occurs over 3 to 5 days. The treatment of chronic compensated respiratory acidosis may be complicated by accompanying hypoxemia. In chronic hypercapnia, the chemical chemoreceptors may be insensitive and the accompanying hypoxemia may supply the main respiratory drive through the stimulation of peripheral chemoreceptors. In such patients, complete correction of hypoxemia may further depress respiration and worsen the respiratory acidosis.


267 Which of the following(s) is/are true concerning the control of the volume of body water?

a. Osmoreceptors and baroreceptors work equally to control fluid balance during normal conditions
b. The cardiac atrium regulates volume only by means of its sympathetic and parasympathetic connections
c. The kidney is the primary effector organ in controlling water balance
d. The conversion of angiotensin I to angiotensin II is dependent on the amount of the enzyme, renin, available
e. Nitric oxide plays a number of important roles in regulation of renal hemodynamics
Answer: c, d, e

Changes in volume are detected both by osmoreceptors, which detect changes in plasma osmolality and baroreceptors, which are sensitive to changes in pressure. The osmoreceptors are responsible for day-to-day fine-tuning of volume whereas the baroreceptors contribute relatively little to the control of fluid balance under normal conditions. Changes in effective circulating volume are sensed by the volume receptors of the intrathoracic capacitance vessels and atria, the pressure receptors of the aortic arch and carotid arteries, the intrarenal baroreceptors, and, to a lesser extent, by the hepatic and cerebrospinal volume receptors. These baroreceptors control volume by means of sympathetic and parasympathetic connections. The atria also appear to serve as endocrine organs capable of directing responses to volume changes with the elaboration of the hormone, atrial natriuretic peptide. The major hormonal mediator of baroreceptor modulation of volume control is the renin-angiotensin system. The end result of this complex system of receptors or messengers is a change in sodium and water balance mediated by the kidneys. It is through changes in sodium and water reabsorption that volume and pressure ultimately normalize. Renin is a proteolytic enzyme that is released in response to changes in arterial pressure, changes in delivery of sodium to the macula densa of the distal convoluted renal tubule, increases in beta adrenergic activity and increases in cellular cAMP. Renin cleaves angiotensin I from circulating angiotensinogen. Angiotensinogen is abundant, so this reaction is enzyme dependent rather than substrate dependent. Angiotensin I is further cleaved to angiotensin II which acts with locally and systemically to increase vascular tone. Angiotensin II affects sodium reabsorption by decreasing renal plasma flow and the glomerular filtration coefficient. Finally, angiotensin II increases sodium reabsorption by direct tubular action as well as by stimulation of aldosterone release from the adrenal cortex.
The importance of nitric oxide and its many biologic functions has recently been recognized. Nitric oxide participates in the regulation of renal hemodynamics and renal handling of water and electrolytes.


268 Which of the following statement(s) is/are true concerning maintenance intravenous fluid therapy?

a. The total daily water requirement for a 70-kg man is about 2500 mL/day
b. Normal maintenance IV therapy requires administration of sodium, potassium, calcium, phosphate, and magnesium
c. Fluid volume calculations for elderly patients generally are decreased compared to their younger counterparts
d. A child requires a lesser amount of maintenance fluid per kilogram than a larger individual
Answer: a, c

Maintenance fluid replacement is aimed at replacing fluids normally lost during the course of a day. Calculation of maintenance fluid replacement does not include replacement of preexisting deficits or ongoing additional losses. Formulas exist for calculating maintenance fluid requirements which adjust for differences in body weight and for changes in TBW content. A smaller (or younger) individual who has a high percentage of TBW in relation to body weight requires a greater amount of maintenance fluid per kilogram than a larger individual. The total daily water requirement for a 70-kg man is about 2500 mL/day. Because hypervolemia is poorly tolerated in older individuals and in patients with cardiac disease, the volume calculated is generally diminished in this age group. Normal maintenance therapy requires administration of sodium and potassium. Replacement of calcium, phosphate or magnesium are generally not necessary in patients requiring short-term therapy. In critically ill patients, however, critical deficits in these electrolytes may occur and must be replaced.


269 Which of the following statement(s) is/are true concerning abnormalities in calcium concentration?

a. Parathyroid hormone affects calcium homeostasis only at the exchange of calcium between bone and extracellular fluid
b. About 45% of total plasma calcium is in the ionized state and is responsible for most physiologic actions
c. Changes in plasma protein levels or pH can alter the proportion of calcium in the ionized state
d. Intravenous normal saline administration is the first step in treatment of hypercalcemia
e. Classic signs of hypocalcemia include hyperactive deep tendon refluxes, Cvostek’s sign and Trousseau’s sign
Answer: b, c, d, e

Calcium is a divalent cation found in abundance in the human body. About 99% of total body calcium is located in bone in the form of hydroxyapatite crystals. Calcium homeostasis depends on the exchange of calcium between bone and extracellular fluid, renal excretion, and intestinal absorption. These three processes are controlled to a great extent by parathyroid hormone. In extracellular fluid, calcium exists in three forms: ionized calcium, non-ionized calcium, and protein-bound calcium. Ionized calcium, which comprises about 45% of total calcium is responsible for most physiologic actions of calcium in the body, and its level is tightly controlled by a regulatory mechanisms. Some nonionized calcium is complexed with non-protein anions, including phosphate and citrate, and does not easily disassociate. These molecular forms make up only about 15% of total calcium present in plasma. About 40% of extracellular nonionized calcium is bound to proteins, with most being bound to albumin. Changes in either plasma protein levels or pH can alter the proportion of calcium in the ionized state. The most common cause of hypercalcemia is primary hyperparathyroidism. Hypercalcemia can also occur secondary to malignant disease, caused either by a metastasis to bone or by autonomous tumor secretion of hormone-like substances that alter calcium homeostasis. Neuromuscular effects may be the earliest manifestations and include muscle fatigue, weakness, personality disorders, psychosis, confusion, and coma. Elevation of total serum calcium concentrations to greater than 14mg/dL requires prompt treatment to prevent any serious and potentially lethal complications. Immediate measures are directed toward maximizing renal excretion of calcium. Vigorous hydration with 0.9% saline solution to prompt diuresis should be the initial step in treatment. The addition of potassium to the resuscitation fluid as well as the use of furosemide can also be used for treatment.
Serum calcium levels below 8 mg/dL may be associated with symptoms and signs that are primary manifestations of neuromuscular abnormalities. These include muscle cramps, perioral tingling, parastesias, laryngeal stridor, tetany, seizures and psychotic behavior. Classic signs of hypocalcemia include hyperactive, deep tendon reflexes, Cvostek’s sign, and Trousseau’s sign. Symptomatic hypocalcemia is best treated with intravenous infusion of calcium in the form of calcium gluconate or calcium chloride.


270 Which of the following statement(s) is/are true concerning total body water?

a. Total body water in men represents a higher percent body weight than in women
b. In infants, water comprises up to 80% of body weight
c. Total body water content decreases with increasing age
d. Total body water is equally distributed within the intra-and extracellular compartments
Answer: a, b, c

The total volume of water within the body is termed total body water. The relationship between total body water (TBW) and body weight is relatively consistent for any given individual and depends on the amount of fat within the body. Because fat contains little water, TBW as a percentage of body weight decreases with increasing body fat. The estimated TBW in men is 60% of body weight, whereas in women, who typically have more adipose tissue, the average TBW is 50% of body weight. The percentage of body weight accounted for by water also varies with age. In infants, water comprises about 80% of body weight. Throughout adult life, a gradual decrease occurs in TBW content because of the amount of fat within the body usually increases with age. In obese patients, estimates of TBW should be decreased by 10% to 20% whereas in lean patients, estimates should be increased by about 10%.
TBW is distributed within the intra and extracellular compartments. Intracellular fluid makes up about 2/3 of the TBW, or 40% of body weight.


271 Which of the following statement(s) is/are true concerning the clinical presentation and treatment of severe metabolic alkalosis?

a. In most cases clinical signs are obvious
b. Correction of potassium and volume depletion corrects most cases of metabolic alkalosis
c. Acetazolamide can enhance renal excretion of bicarbonate
d. Acid replacement should be provided at a molar equivalent basis for excess serum bicarbonate
Answer: b, c

Clinical signs of metabolic alkalosis may not be prominent, since the condition usually develops relatively slowly. Correction of the underlying cause is the mainstay of treatment in this disorder. In general, correction of potassium and volume depletion corrects the metabolic alkalosis. In patients without intravascular volume deficits, renal excretion of bicarbonate can be enhanced by administration of the carbonic acid anhydrase inhibitor acetazolamide. If renal excretion of bicarbonate cannot be increased because of underlying renal insufficiency or if the metabolic alkalosis is severe, acid may be administered to directly titrate the excess extracellular bicarbonate. Acids that can be used include ammonium chloride, arginine hydrochloride, or dilute hydrochloric acid. Partial correction of alkalosis is the initial goal. A general guide is that 2.2 mEq/kg decreases serum bicarbonate by about 5 mEq/L.


272 Which of the following statement(s) is/are true concerning abnormalities in serum sodium?

a. The most common cause of hyponatremia is a deficit in total body sodium
b. Hyponatremia can occur in situations of excessive solute
c. Most surgical patients with hyponatremia are best treated by free water restriction
d. Central nervous system effects are the predominant symptom of hypernatremia
e. Hypernatremia should be rapidly corrected with free water administration
Answer: b, c, d

The most common cause of hyponatremia is an excess of free water rather than a deficit of total body sodium. Hyponatremia is frequently seen in the postoperative or post-trauma patients because increased ADH secretion acts on the collecting tubules of the kidney to increase free water reabsorption. Although hyponatremia most often results from excess free water, it can occur in the presence of excess solute. In this situation, TBW content is either normal or diminished but the plasma osmolality is increased. An example of this hyperosmolar-hyponatremic state is untreated hyperglycemia. Excess solute may also be due to exogenous administration or ingestion of mannitol, ethanol, methanol, or ethylene glycol. Most surgical patients with hyponatremia are euvolemic or hypervolemic. Such patients, if asymptomatic, are best treated by free water restriction, since free water overload is the cause of the condition. Hypernatremia is a less common problem in surgical patients than hyponatremia and is usually the result of excess free water loss associated with hypovolemia. Hypernatremia may also be secondary to increased total body content of sodium, which is usually related to exogenous administration of sodium. The symptoms of hypernatremia are related to the hyperosmolar state. CNS effects predominate because of cellular dehydration as water passes into the extracellular space. Once hypernatremia becomes symptomatic, it is associated with significant morbidity and mortality. Prompt treatment of hypernatremia is essential. Rapid correction, however, of hypernatremia is associated with significant risk of cerebral edema and herniation. Because chronic hypernatremia is relatively well tolerated, there are few advantages to rapidly correcting the free water deficit. Moderate degrees of hypernatremia are tolerated well, and symptoms rarely develop unless serum sodium levels exceed 160 mEq/liter. The development of symptoms also depends on the rapidity at which hypernatremia develops.


273 Which of the following statement(s) is/are true concerning abnormalities in serum potassium?

a. Hyperkalemia can occur in the otherwise normal surgical patient due to excessive intravenous potassium administration
b. The primary EKG change associated with severe hyperkalemia is peaked T-waves
c. Temporary treatment of hyperkalemia includes administration of calcium, sodium bicarbonate, or glucose and insulin
d. Alterations in membrane potentials reflected in cardiac and skeletal muscle are common results of both hypo-and hyperkalemia
e. A reduction in serum potassium of 1mEq/liter requires replacement of 40mEq of potassium
Answer: c, d

Potassium is the major intracellular cation and is a major determinant of intracellular osmolality. Because of the large differences between intracellular and extracellular potassium concentrations, a transmembrane potential is generated. Alterations in potassium concentration gradient (both hyper- and hypokalemia) have profound effects on transmembrane potential and consequently on cellular function. This is especially true for cardiac, skeletal, and smooth muscle. Extracellular potassium concentration is primarily determined by renal excretion. About 90% of ingested potassium is secreted by the urine. Hyperkalemia therefore rarely develops from excessive potassium intake in the absence of renal insufficiency, since the capacity for renal potassium excretion is large. In the surgical patient, diminished renal function is perhaps the most common problem leading to hyperkalemia. Both chronic and acute renal failure result in the deficit in potassium excretion. Hyperkalemia can also be associated with cellular disruption, such as with crush injuries or lysed erythrocytes in large hematomas or after massive blood transfusion. The clinical manifestations of hyperkalemia are primarily related to membrane depolarization. The most life-threatening manifestations are related to the cardiac effects of membrane depolarization. Mild hyperkalemia results in peaked T-waves on the EKG and may cause parethesia and weakness. More severe forms of hyperkalemia cause flattened P-waves, prolongation of the QRS complex, and deep S-waves on EKG. Ventricular fibrillation and cardiac arrest may follow. Severe hyperkalemia with EKG abnormalities requires urgent treatment. Rapid infusion of 10% to 20% calcium gluconate may reduce the effects of hyperkalemia on membrane potentials. Administration of sodium bicarbonate is another temporary measure. The increase in serum sodium antagonizes the effects of hyperkalemia on the membrane potential, whereas the increase in extracellular pH shifts potassium into the cells. Movement of potassium into the intracellular compartment can also be achieved by giving insulin and glucose.
Hypokalemia is usually caused by total body potassium depletion secondary to the decreased potassium intake, increased extra-renal potassium losses, or increased renal potassium losses. Decreased serum potassium levels may also be secondary to redistribution of potassium into the intracellular space. Symptoms of hypokalemia, like those of hyperkalemia, are manifested by disturbances in membrane potentials. As potassium levels fall below 2.5mEq/L, muscle weakness is common. The primary treatment of hypokalemia is potassium replacement. The route and rate of potassium replacement depends on the presence and severity of symptoms. A reduction in serum potassium of 1mEq/L represents a total body potassium deficiency of 100 to 200 mEq.


274 Which of the following statement(s) is/are true concerning the derangement of metabolic acidosis?

a. A major source of acid production of the body is sulfuric acid
b. Excessive loss of bicarbonate can occur with intestinal or pancreatic fistulas
c. Ketoacidosis can occur in conditions of either hyper-or hypoglycemia
d. Lactic acidosis is present when serum lactate concentration is > 2 mEq/L
e. Lactic acidosis can be associated with ethanol toxicity
Answer: a, b, c, e

Most clinically significant metabolic acidosis is related to the net loss of bicarbonate, which occurs when consumption due to either loss or titration is greater than bicarbonate generation. Under normal circumstances of ingestion of the average amount of protein in an American diet, about 70 mEq acid is generated daily. The major source of acid production is sulfuric acid from the metabolism of sulphur-containing amino acids. Increased protein intake and tissue catabolism resulting in greater metabolism of sulphur containing amino acids may lead to a generation of increased amounts of sulfuric acid. This excess acid utilizes excess bicarbonate for neutralization. Diarrhea, intestinal or pancreatic fistulas, and burns can cause metabolic acidosis secondary to loss of bicarbonate. The two most common types of organic acidosis are ketoacidosis and lactic acidosis. The abnormality primarily responsible for ketoacidosis is deficiency of insulin whether primary, as in diabetic ketoacidosis, or secondary to hypoglycemia. Under normal conditions a small amount of ketoacids is produced. During prolonged starvation, production of ketoacids increase to modest levels, providing an important source of energy to nonhepatic tissues, particularly the brain. In ketoacidosis, the ketoacid production is excessive because of insulin deficiency. In diabetic acidosis, insulin deficiency also contributes to hyperglycemia by decreasing the metabolism of glucose by extrahepatic tissue and increasing hepatic production of glucose.
Lactic acidosis can be divided into type A, caused by tissue hypoxia, and type B, caused by other mechanisms. Hypoxia, the most common cause of lactic acidosis, impairs the mitochondrial oxidation of NADH to NAD that is necessary for glycolysis. Normal serum lactate concentration is below 2 mEq/L. Lactate acidosis is secondary to hypoxemia, usually due to an increased production of lactate as well as decreased use, and serum lactate concentrations greater than 6 mEq/L. The most common cause of type B lactate acidosis is ethanol intoxication.


275 Which of the following statement(s) is/are true concerning the osmotic activity of body fluids?

a. Urea contributes to the osmolality of a solution but not its tonicity
b. The osmolality of the body remains fairly constant at approximately 289 mOsm/kg H2O
c. The two primary regulators of water balance are antidiuretic hormone and aldosterone
d. Serum sodium is the most valuable laboratory indicator of abnormal total body water content
Answer: a, b, d

Body fluids are aqueous solutions composed primarily of water and contained in different compartments of the body. The movement of water from these compartments depends on a number of physical properties, the most important of which is osmosis. According to the principles of osmosis, if two solutions are separated by semipermeable membrane, water moves across the membrane to equalize the concentration of the osmotically active particles. The osmotic activity across a semipermeable membrane is determined by the concentration of solutes on each side of the membrane. The body is capable of fine regulation of solute and water concentrations, so that osmolality remains fairly constant at an average of 289 mOsm/kg H2O. In response to small changes in cell volume, osmoreceptors in the paraventricular and supraoptic nuclei of the hypothalamus send signals to the neuronal centers that control the two primary regulators of water balance, thirst and antidiuretic hormone secretion. Changes in TBW are reflected by changes in extracellular solute concentration. Because sodium is the primary extracellular cation and potassium is the predominant intracellular cation, the serum sodium approximates the sum of the exchangeable total body sodium and exchangeable total body potassium divided by the TBW. Because total body solute content remains relatively stable over time, changes in TBW content result in inversely proportional changes in serum sodium. Thus, abnormalities in serum sodium are the indication of abnormal TBW content. In contrast to impermeable solutes that are excluded from the intracellular space, such as sodium, permeable solutes such as urea can freely cross the cell membranes. Although urea contributes to the osmolality of a solution, it has no effect on tonicity because it distributes equally across membranes, and as such does not contribute to the osmols that affects cell volume.


276 Which of the following statement(s) is/are true concerning the compensatory mechanisms and treatment of metabolic acidosis?

a. Maximal renal compensation for metabolic acidosis occurs before full respiratory compensation can occur
b. All patients with lactic acidosis should receive prompt treatment with bicarbonate
c. Potassium replacement is essential even in the face of normal or high serum potassium when treating diabetic ketoacidosis
d. Sodium bicarbonate administration should begin simultaneous with volume resuscitation in patients with hypoxia secondary to shock
Answer: c

The kidney is extremely sensitive to changes in serum bicarbonate concentration and responds by increasing net acid excretion primarily by increasing ammonia excretion. Maximal renal compensation requires 2 to 4 days. Delay in achieving maximal renal response to an increased acid load causes blood pH to decline, which stimulates hyperventilation. Although effective in promptly raising blood pH, ventilatory compensation is only partial, and full respiration compensation requires 12 to 24 hours. The major principal of treatment for mild to moderate acute metabolic acidosis is correction of the underlying cause. In surgical and trauma patients, metabolic acidosis is often the result of hypoxia secondary to inadequate tissue perfusion and subsequent lactic acidosis. Volume and/or blood resuscitation alone may be enough to correct the acidosis. Attempts to correct acidosis with exogenous bicarbonate before correction of inadequate tissue perfusion are usually unsuccessful. The use of bicarbonate for the treatment of lactic acidosis is controversial at best. In several studies the use of bicarbonate in patients with lactic acidosis does not improve clinical parameters or outcome. The correction of both acidosis and hypoglycemia of diabetic ketoacidosis is best achieved by the administration of insulin. Volume resuscitation is also required. Potassium replacement is essential, even in the face of normal or high serum potassium, and as hypokalemia develops as acidosis in hyperglycemia are corrected.


277 Which of the following statement(s) is/are true concern renal tubular acidosis?

a. Renal tubular acidosis is primarily caused by reduction in ammonia excretion
b. The renal tubular defect in renal tubular acidosis can either be at the distal or proximal renal tubule
c. In distal renal tubular acidosis associated with hyperkalemia, the defect involves increased tubular permeability with backleak of secreted sodium and potassium into the tubular cell
d. Uremic acidosis occurs independently of protein intake
Answer: a, b

The impaired ability of the kidney to excrete acid and hence generate bicarbonate may be secondary to a decrease in the number of functioning nephrons and is termed uremic acidosis or renal tubular acidosis. Renal tubular acidosis, which can occur both in acute and chronic renal failure, is primarily caused by reduction in ammonia excretion secondary to a reduction in the number of functioning proximal tubular cells. In addition, decreased proximal tubular bicarbonate reabsorption contributes to the development of acidosis. Although the onset of uremic acidosis is related to declining renal function, its appearance may be influenced by diet-dependent protein and organic anion ingestion. Renal tubular acidosis may be classified as distal or proximal, depending on the primary site of the renal tubular defect leading to acidosis. In renal tubular acidosis with hyperkalemia, the mechanism is decreased luminal negativity secondary to impaired sodium reabsorption. In distal renal tubular acidosis with hypokalemia, mechanisms including increased tubular permeability with backleak of secreted H+ into the tubular cell and reduced H+ pump activity are proposed mechanisms.


278 Which of the following statement(s) is/are true concerning the postoperative fluid management in a surgical patient?

a. Standard formulas are available that essentially can direct the therapy for all patients
b. Isotonic solutions containing potassium should be used throughout the entire postoperative period
c. Urine output should be maintained at a level greater than 0.5 ml/kg/h
d. A urine specific gravity of greater than 1.012 may indicate that the patient is dehydrated
Answer: c, d

Fluid therapy during the postoperative period should be tailored to each patient and depends on the adequacy of patient’s volume status at the completion of the operative procedure, as well as ongoing fluid losses. Maintenance fluid should be supplemented by replacement of the additional fluids needed to replace the ongoing third space loss as well as losses from various tubes and drains. In general, isotonic solution should be used for volume resuscitation during the early postoperative period. It is best not to give potassium supplements during this period unless they are specifically required as indicated by serum electrolyte measurements. Monitoring fluid status during the postoperative period is best accomplished by careful monitoring of vital signs, urine output, and central venous pressure, if necessary. Urine output is maintained at a level greater than 0.5 mL/kg/h. A urine specific gravity of greater than 1.010 to 1.012 indicates that urine is being concentrated and the patient may not be receiving adequate hydration.


279 Which of the following statement(s) is/are true concerning parenternal electrolyte solutions?

a. Lactated Ringer’s solution contains physiologic concentrations of all important electrolytes
b. Glucose is added to hypotonic saline solutions to increase their tonicity
c. About 1/2 of all exogenously administered albumin ends up in the extravascular space
d. Normal saline solution provides excessive sodium and chloride which may lead to body sodium overload
Answer: b, c, d

A number of electrolytes solutions are available for parenteral administration. Lactated Ringer’s solution is a physiologic solution containing many of the electrolytes found in plasma. The disadvantage of this solution is the relatively low sodium content (130 mEq/L) as compared to plasma. Hyponatremia can occur with extended use of lactated Ringer’s solution. Isotonic saline (0.9% or normal saline) contains 154 mEq of both sodium and chloride. The excess of both sodium and chloride can lead to electrolyte and acid-base disturbances. Infusion of large volumes of 0.9% saline can lead to total body sodium overload and hyperchloremia. The less-concentrated saline solutions are hypo-osmotic and have excess free water. In addition, 0.2% saline solution is hypotonic with respect to plasma and can result in red blood cell lysis if rapidly infused. For this reason, 5% dextrose is added to these solutions to increase the tonicity. Plasma expanders are commonly used in surgical patients. Plasma protein solutions such as 5% and 25% albumin act initially by increasing plasma oncotic pressures. Abnormalities in microvascular permeability such as those found in the pulmonary circulation in adult respiratory distress syndrome, in regional circulatory bed burns or infections, and in the systemic circulation in sepsis, may result in extravasation of these proteins into the interstitial space. About half of all exogenously administered albumin eventually ends up in the extravascular space. The half life of exogenously administered albumin is about 11 days.


280 An 11-year-old boy has experienced severe diarrhea for 10 days. He presents with decreased skin tungor, sunken eyes, orthostatic hypotension, and tachycardia. Which of the following statement(s) may be true concerning his diagnosis and treatment?

a. His hematocrit will likely be elevated
b. His BUN may be elevated out of proportion to serum creatinine
c. His serum sodium will be elevated
d. Fluid resuscitation should begin with D5/.2 normal saline because of the expected high serum sodium associated with excessive fluid loss
Answer: a, b

Chronic volume deficits may be manifested by decreased skin turgor, weight loss, sunken eyes, hypothermia, oliguria, orthostatic hypotension and tachycardia. Serum BUN and creatinine may be elevated, with a high BUN/creatinine ratio. The hematocrit may be elevated as well. Plasma sodium is not an indicator of intravascular volume, and if the loses have been isotonic, plasma sodium concentration remains normal. Fluid resuscitation for hypovolemia is initiated with an isotonic solution such as lactated Ringer’s solution. Urine flow in critically ill patients is monitored with an indwelling Foley catheter, with the goal of a urine output 0.5mL/kg/h desirable.


281 Clinical manifestations of acute metabolic acidosis include:

a. Decreased cardiac contractility
b. Decreased catecholamine secretion
c. Peripheral arteriolar dilitation
d. Shift of the oxygen-hemoglobin disassociation curve to the left
Answer: a, b, c

The major cardiovascular effects of acute metabolic acidosis are peripheral arteriolar dilitation, decreased cardiac contractility, and central venous constriction. These may lead to cardiovascular collapse and pulmonary edema. Catecholamine secretion is stimulated by metabolic acidosis and in mild cases, heart rate may be increased. In addition to these cardiovascular effects, metabolic acidosis may also affect oxygen delivery by shifting the oxygen-hemoglobin disassociation curve to the right.


282 Which of the following statement(s) is/are true concerning respiratory alkalosis?

a. Exposure to high altitudes can result in respiratory alkalosis
b. Renal compensation for respiratory alkalosis is obtained by increasing excretion of bicarbonate
c. Symptoms of respiratory alkalosis may mimic those of hypocalcemia
d. The treatment of acute respiratory alkalosis may involve a brown paper bag
Answer: a, c, d

A primary decrease in PCO2 resulting in an increase extracellular pH is referred to as respiratory alkalosis. Hyperventilation and the ensuing fall in PCO2 may be secondary to hypoxia, reflux simulation from decreased pulmonary compliance, drugs, mechanical ventilation, and other causes. The two most common causes of hypoxia resulting in respiratory alkalosis are pulmonary disease and exposure to high altitudes. Renal compensation for respiratory alkalosis is not achieved by increasing excretion of bicarbonate but by decreasing net acid excretion, primarily through the reduction in ammonia excretion and increases in organic anion excretion. Chronic respiratory alkalosis is generally asymptomatic. Acute respiratory alkalosis may cause sensations of breathlessness, dizziness, and nervousness and can result in circumoral and extremity parathesias, altered levels of consciousness, and tetany. These signs are related to decreased cerebral blood flow secondary to decreased PCO2 and decreased ionized calcium concentration secondary to increased blood pH. In acute symptomatic respiratory alkalosis rebreathing, by breathing in and out of a paper bag, can temporarily relieve the symptoms.


283 Which of the following statement(s) is/are true concerning alterations in serum magnesium?

a. Renal failure is the primary cause of hypermagnesemia
b. Hypomagnesemia may occur during prolonged periods of intravenous fluid replacement
c. Symptoms of hypomagnesemia may mimic symptoms of hypocalcemia
d. Intravenous administration of magnesium sulfate is usually the most efficient method of correction of magnesium deficiency
Answer: a, b, c, d

Renal failure is the primary cause of hypermagnesemia. Because of the kidneys ability to excrete large magnesium loads, hypermagnesemia rarely occurs if renal function remains normal. Because the kidneys are able to conserve magnesium well in states of magnesium depletion, hypomagnesemia rarely occurs from poor intake alone. The combination of low intake and increased gastrointestinal loss may lead to hypomagnesemia. Prolonged periods of intravenous fluid replacement without magnesium replacement and the chronic use of loop diuretics or other medications such as cyclosporine or aminoglycosides can also result in hypomagnesemia. Deficiencies of magnesium may present signs and symptoms similar to hypocalcemia. Hypomagnesemia may be treated by the oral administration of magnesium however large doses frequently leads to diarrhea. Correction of major deficits is therefore best managed by intravenous administration of magnesium sulfate at a dose of 50 to 100 mEq/d.

284 Muscle relaxants are a class of anesthetic agents used to prevent movement and facilitate surgical exposure. Which of the following statement(s) is/are true concerning the use of muscle relaxants in surgical procedures.

a. Succinylcholine produces rapid obvious muscle fasciculations
b. Pancuronium can be reversed by increasing the acetylcholine concentration using an anticholinesterase inhibitor (neostigmine)
c. Prolonged periods of muscle relaxation in patients requiring prolonged ventilation should be used in conjunction with analgesics and amnesic agents
d. The best clinical test for complete reversal of neuromuscular blockade is the ability of the patient to produce a large negative inspiratory force
Answer: a, b, c

Neuromuscular blocking agents can be classified as depolarizing or nondepolarizing inhibitors of the neurotransmitter, acetylcholine at the neuromuscular junction. The only noncompetitive inhibitor employed clinically is succinylcholine. This drug rapidly binds to the neuromuscular junction and produces depolarization, clinically obvious as fine muscle fasciculations occurring approximately 60 seconds after injection. All other clinically useful muscle relaxants are termed competitive inhibitors and do cause depolarization when they attach to the neuromuscular junction. Since these agents compete with acetylcholine, the block produced is in direct proportion to the concentration of the agent relative to the concentration of acetylcholine. If the concentration ratio is low enough, competitive relaxants can be “reversed” if the concentration of acetylcholine is artificially elevated. Increase of acetylcholine concentration can be achieved by giving a drug which blocks metabolism of anticholinesterase (neostigmine).
Nondepolarizing relaxants are frequently used in critically ill patients who are difficult to manage otherwise during prolonged periods of mechanical ventilation. It is imperative that these drugs be given in conjunction with analgesics and amnesic agents, since neuromuscular blocking agents have no analgesic or amnestic properties and only prevent motion of voluntary muscles. Patients may therefore be totally aware and in pain and unable to communicate. All muscles of the body do not have equal sensitivity in muscle relaxants. The diaphragm is both resistant to neuromuscular blockade while the neck and pharyngeal muscles that support the airway are most sensitive. It is possible for an intubated patient to spontaneously ventilate and even to produce a large negative inspiratory force and yet have complete airway obstruction when extubated due to effects of residual muscle relaxants on upper airway muscles. The definitive clinical test for complete reversal of neuromuscular blockade is the ability of the patient to sustain a head lift from the bed for five seconds.


285 Local anesthetics are essential agents used in current surgical practice. Which of the following statement(s) is/are true concerning the use of local anesthetic agents.

a. Complications due to excessive plasma concentration can result only from inadvertent intravascular injection of the agent
b. Bupivacaine is noted for a slow onset but long duration
c. The addition of epinephrine to a local anesthetic agent will both lower the toxicity and increase the duration of local anesthesia
d. Hypotension observed when a local anesthetic is administered in the form of a spinal epidural block, is the result of myocardial depression
Answer: b, c

Local anesthetics constitute a class of drugs which produce temporary blockage of nerve conduction by binding to neuronal sodium channels. Adverse consequences associated with the use of local anesthetics fall into three categories: acute central nervous system toxicity due to excessive plasma concentration, hemodynamic and respiratory consequences due to excessive conduction block of the sympathetic or motor nerves, and allergic reactions. Whenever a local anesthetic has been injected, there may be inadvertent intravascular injection or an overdose of the drug due to rapid uptake from the tissues. All may produce seizures. Complications can be minimized by aspirating prior to injection to avoid intravascular injection and limiting the doses to the safe range. When local anesthetics are administered for a spinal or epidural block, there will be a progressive blockade of the sympathetic nervous system which will produce systemic vasodilatation. If the block travels along the thoracolumbar region, a sympathetic blockade will result in profound systemic vasodilatation and bradycardia with resultant hypotension.
Local anesthetics are divided into two groups: esters and amides. Most commonly used agents, the amides, include lidocaine and bupivacaine. Lidocaine is noted for a fast onset of action but a short duration whereas bupivacaine has a slower onset with the duration lasting for four to 12 hours. The addition of epinephrine (100 µg) will lower the toxicity and increase the duration of the local anesthetic.


286 A 65-year-old gentleman with a history of coronary artery disease and a recent myocardial infarction requires an elective colon resection for a nonobstructing neoplasm. Which of the following statement(s) is/are true concerning the risks of general anesthetic in this patient?

a. The age of the previous infarct has no effect on the perioperative reinfarction risk
b. The incidence of reinfarction appears to stabilize after six months
c. Invasive hemodynamic monitoring has no effect on perioperative reinfarction rates
d. Reinfarction has minimal effect on mortality
e. Perioperative infarction most frequently occurs after the first 72 hours from surgery
Answer: b, d

The history of myocardial infarction is an important risk factor for general anesthesia. Large retrospective studies have found that the incidence of reinfarction is related to the time elapsed since the previous myocardial infarction. The incidence of reinfarction appears to stabilize at approximately 1% after six months, with the highest rate of reinfarction occurring in the first three months after the infarct. Mortality from reinfarction, for patients undergoing non-cardiac surgery, has been reported to be between 20–50% and usually occurs within the first 48 hours after surgery. Invasive hemodynamic monitoring with pulmonary artery catheters and aggressive pharmacologic intervention has been demonstrated to reduce reinfarction rates.


287 General anesthesia is not without risks. Which of the following statement(s) is/are true concerning the risk associated with general anesthesia.

a. Current estimates for mortality due to anesthesia alone are 1:10,000
b. Human error accounts for between 50 and 75% of anesthetic-related deaths
c. Most anesthetic-related deaths are associated with overdose of analgesic agents
d. The most common problems associated with adverse anesthetic outcomes are related to the airway
Answer: b, d

Anesthetic agents effectively obtund or completely block nearly all physiologic protective mechanisms, therefore, there is an associated risk even without a surgical procedure. Fortunately, with the advent of newer agents and monitoring techniques, it is estimated the mortality due to anesthesia alone has decreased from approximately 1:10,000 in the 1950s to as low as 1:100,000 or less for healthy patients today. It has been estimated that between 50–75% of anesthetic-related deaths are due to human error and are preventable. The most common problems associated with adverse outcomes are related to the airway: inadequate ventilation, unrecognized esophageal intubation, unrecognized extubation, and unrecognized disconnection from the ventilator.


288 Over the last decade, the routine use of both invasive and noninvasive monitoring devices has been instituted for the administration of most anesthetics. The following statement(s) is/are true concerning monitoring of the surgical patient.

a. A pulse oximeter reading will reflect changes in PaO2 only below 80 mm Hg
b. Monitoring of end tidal CO2 will reflect changes in ventilation but not cardiac output
c. Intermittent, noninvasive systemic blood pressure monitoring using an oscillometric blood pressure cuff has essentially replaced clinical measurement by auscultation
d. Pulmonary arterial catheter monitoring is generally reserved for critically ill patients with significant left ventricular dysfunction
Answer: a, c, d

Pulse oximetry continuously, noninvasively and inexpensively provides arterial hemoglobin saturation and peripheral pulse determination. It must be remembered, however, that a pulse oximeter measures oxygen saturation and not arterial oxygen tension (PaO2). The PaO2 must drop below 80 mm Hg before any significant change in oxygen saturation will occur. End tidal CO2 monitoring reflects metabolism (the production of CO2), circulation (blood flow to the lungs), and ventilation (respiratory rate in an intact ventilatory circuit). It can be used as a surveillance monitor for both the respiratory circuit and the cardiovascular system. Any acute decrease in cardiac output will decrease output to the lung and increase alveolar dead space, causing an acute drop in end tidal CO2.
Hemodynamic stability can be monitored in a variety of methods, the most basic of which is systemic arterial blood pressure measure. Intermittent, noninvasive measure of systemic blood pressure with an oscillometric blood pressure cuff has become the standard in the operating room with an accuracy equal to that of clinical measurement by auscultation. When tighter control is required in patients with significant hypertension, serious heart disease, or in patients who may suffer acute blood loss, invasive arterial monitoring is employed. In patients with left ventricular dysfunction who are undergoing extended surgical procedures with significant fluid shifts and potential blood loss, central venous pressure monitoring is frequently used, with pulmonary arterial catheter monitoring reserved for more critically ill patients and for those with significant left ventricular dysfunction.

289 Correct statement(s) concerning complications occurring in the post-anesthetic care unit include which of the following?

a. The use of nitrous oxide has been well documented to increase the incidence of postoperative nausea
b. Perioperative myocardial ischemia is usually easily diagnosed in the early postoperative period
c. Hypothermia results in a deleterious effect on drug metabolism therefore delaying recovery from anesthesia
d. The serotonin antagonist, odansetron, holds promise as the superior antiemetic agent in the perioperative period
Answer: c, d

Twenty-four percent of patients experience a post-anesthetic care unit complication. Nausea, vomiting and airway support comprise 70% of these complications. The need to maintain airway support is by far the most common respiratory complication. Hypothermia has a deleterious effect on altering drug metabolism and delaying recovery. Nausea and vomiting are rarely unifactorial and cause considerable discomfort to the patient. There is little evidence to favor one anesthetic or anesthetic technique over another. Nitrous oxide does not appear to increase incidence of nausea in well documented studies. The new serotonin antagonist, odansetron, has been shown in several studies to be superior to other agents as a perioperative antiemetic agent.
Perioperative myocardial ischemia is an extremely important complication but difficult to recognize. Diagnosis is complicated by the fact that only 10—30% of patients suffering documented myocardial infarction will have pain and that postoperative EKG changes are often nonspecific. One must therefore look for secondary indications of on-going ischemia such as hypotension, arrhythmias, elevated filling pressures, or postoperative oliguria.


290 Patient-controlled analgesia ( PCA) is a commonly used technique for postoperative analgesia. The following statement(s) is/are true for the use of PCA.

a. Satisfactory pain relief is provided by the administration of higher narcotic doses
b. The technique is not applicable in the semiconscious or uncooperative patient
c. PCA is as safe as conventional intramuscular administration of pain medication
d. Excessive administration of narcotic medication can be limited by a lockout duration which controls administration of the narcotic
Answer: b, c, d

The technique of patient-controlled analgesia is based on investigations that small intravenous bolus doses of narcotic on demand can provide patients with improved pain relief at the same or less total narcotic dose. The system requires some degree of sophistication and a conscious patient who has been instructed in the technique. Numerous studies have demonstrated that PCA is as safe as conventional IM medication. The patient can be restricted from receiving excessive agents via setting a lockout interval duration of several minutes during which time a dose of narcotic cannot be successfully administered. In addition, limits to the total hourly dose can be set.

291 Narcotics are commonly used in the administration of general anesthesia. Which of the following statement(s) is/are true concerning this class of agents.

a. Narcotics have both profound analgesic and amnestic properties
b. Narcotics can cause hypotension by direct myocardial depressive effects
c. Naloxone should be used routinely for the reversal of narcotic analgesia
d. Acutely injured hypovolemic patients are at significant risk for decreased blood pressure with the use of narcotic analgesics
e. Propofol is a new intravenous short-acting narcotic used frequently in the outpatient setting
Answer: d

Narcotics and synthetic analogues belong in the class of drugs called opioids. Narcotics produce profound analgesia and respiratory depression. They have no amnesic properties, no myocardial depressive effects, and no muscle relaxant properties. Narcotics may produce significant hemodynamic effects indirectly through the release of histamine and/or blunting of the patient’s sympathetic vascular tone due to analgesic properties. Acutely injured patients may be hypovolemic and in pain, with high sympathetic tone and peripheral resistance. Therefore, such patients can experience a dramatic drop in systemic blood pressure with minimal doses of opioids. All opioids can be reversed with naloxone. Naloxone reversal, however, can be dangerous because the agent acutely reverses not only the analgesic effects of the opioid but also analgesics effects of native opioids. Naloxone treatment has been associated with acute pulmonary edema and myocardial ischemia and should not be used electively to reverse the effects of narcotic. Propofol is a lipid-soluble substitute isopropyl phenol non-narcotic agent that produces rapid induction of anesthesia followed by awakening in four to eight minutes.


292 Anesthetic techniques used in the management of patients with significant pulmonary disease include:

a. Intubation at a deep level of anesthesia
b. Choice of an anesthetic agent which produces bronchodilatation
c. The use of epidural analgesia for postoperative pain control
d. Perioperative use of intermittent positive pressure breathing
Answer: a, b, c

Patients with significant pulmonary diseases require special anesthetic techniques. Obstructive pulmonary disease can either be chronic (COPD) or acute (asthma). In either case, the reversible component of obstruction should be reversed prior to elective surgery. In patients with reactive airway disease, the endotracheal tube may induce severe bronchospasm. Even in patients who are well treated preoperatively, reactive bronchospasm may complicate anesthetic induction and emergence from anesthesia. The principal method used to prevent or diminish this “foreign body” induced bronchospasm is intubation of the patient at a deep level of anesthesia when reflexes are blunted. The classic way of managing a patient with severe asthma is to induce with an agent that produces bronchodilatation and to ventilate the patient with an inhalation agent until deeply anesthetized prior to laryngoscopy and intubation. The patient should be extubated while spontaneously ventilating, but with the inhalation agent still in effect, bringing the patient to consciousness while ventilating by mask.
Because of the potential adverse effects of systemic narcotics on respiratory drive, the use of epidural narcotics and local anesthetics for postoperative pain control has become very popular. These techniques allow the patient to be extubated earlier, and patients with intrathoracic and upper abdominal surgery, help restore pulmonary function toward preoperative values. Preoperative use of intermittent positive pressure breathing has not been demonstrated to decrease the incidence of postoperative pulmonary complications.


293 Which of the factors listed below will adversely affect the risk of perioperative cardiac complications and reinfarction in the patient described above?

a. Greater than five premature ventricular beats per minute on EKG rhythm strip
b. The anesthetic technique used
c. Withdrawal of medical therapy with beta blockers and topical nitrates
d. Length of surgical procedure less than three hours
e. Known three vessel coronary artery disease
Answer: a, c, e

The incidence of reinfarction is increased in patients undergoing intrathoracic or intra-abdominal procedures lasting longer than three hours. The site of surgery or anesthetic technique have not been shown to change the incidence of reinfarction if the procedure is less than three hours in duration. Patients with known three-vessel or left main coronary artery disease are at increased risk, while those who have undergone prior coronary artery bypass grafting are of substantially decreased risk of reinfarction. Prophylactic therapy with beta blockers, calcium channel agents, and nitrates has not been proven beneficial; however, withdrawal of these agents has been associated with perioperative ischemia, myocardial infarction, and death. CHF is the single most important factor predicting postoperative cardiac morbidity. Rhythm disturbances, particularly frequent premature ventricular beats, more than five beats/minute, are also independently associated with an increased risk of perioperative cardiac complications.

44 comments:

  1. I've been absent for some time, but now I remember why I used to love this website. Thank you, I will try and check back more frequently. How frequently you update your web site?
    Walex OTH-11129 Instant Hand Sanitizer - 800 ml Refill, (Pack of 12)

    ReplyDelete
  2. Free Medical MCQs here: http://www.medmcqs.com

    ReplyDelete
  3. Thank you for taking the time to publish this information very useful! I've been looking for books of this nature for a way too long. I'm just glad that I found yours. Looking forward for your next post. Thanks :)

    TOSHIBA PLT-1204BT

    ReplyDelete
  4. Very intersting post…. I enjoyed reading your informative article and considering the points. TOSHIBA PLT-805AT

    ReplyDelete
  5. Thank you for taking the time to publish this information very useful! I've been looking for books of this nature for a way too long. I'm just glad that I found yours. Looking forward for your next post. Thanks :)

    purchase blood pressure monitor

    ReplyDelete
  6. You made this kind of fascinating piece to peruse, giving each subject illumination for us to pick up learning. A debt of gratitude is to ensure that imparting the such data to us to peruse thiscolorectal surgeons Houston

    ReplyDelete
  7. What is your reference here? Thank you

    ReplyDelete
  8. Great Article it its really informative and innovative keep us posted with new updates. its was really valuable. thanks a lot. ias app

    ReplyDelete
  9. it's really nice and meanful. it's really cool blog. Linking is very useful thing.you have really helped lots of people who visit blog and provide them usefull information. currency conversion

    ReplyDelete
  10. Everyone out there,I am here to give my testimony about a Herbalist doctor who helped me . I was infected with HERPES SIMPLEX VIRUS in 2016, i went to many hospitals for cure but there was no solution, so I was thinking how can I get a solution out so that my body can be okay. One day I was in the river side thinking where I can go to get solution. so a lady walked to me telling me why am I so sad and i open up all to her telling her my problem, she told me that she can help me out, she introduce me to a doctor who uses herbal medication to cure HERPES SIMPLEX VIRUS and gave me his email, so i mail him. He told me all the things I need to do and also give me instructions to take, which I followed properly. Before I knew what is happening after two weeks the HERPES SIMPLEX VIRUS that was in my body got vanished . so if you are also heart broken and also need a help, you can also email him at drgodwinsolutionhome@gmail.com or whatsapp him number: 2348054615060 Contact him today a

    ReplyDelete
  11. This is an excellent Blog. Please do check out my blog too Go Doc Web

    ReplyDelete
  12. https://www.nursingpath.in/2016/09/medical-surgical-nursing-mcq.html?showComment=1602346543787#c215522206188295876

    ReplyDelete
  13. Are you looking for Piles, Hernia, Fistula, and Wight Loss Surgery in Jaipur make an appointment with Best GI Surgeon in Jaipur Dr. Vinay Kumar Mahala. For appointment call on 094149 82717
    GI Surgeon in Jaipur

    ReplyDelete
  14. My husband was diagnosed with early onset Parkinson's disease at 57.his symptoms were shuffling of feet,slurred speech, low volume speech, degradation of hand writing, horrible driving skills, right arm held at 45 degree angle, things were tough for me, but now he finally free from the disease with the help of total cure ultimate health home, he now walks properly and all symptoms has reversed, he had trouble with balance especially at night, getting into the shower and exiting it is difficult,getting into bed is also another thing he finds impossible.we had to find a better solution for his condition which has really helped him a lot,the biggest helped we had was ultimatehealthhome they walked us through the proper steps,am highly recommended this ultimatehealthhome@gmail.com to anyone who needs help.

    ReplyDelete
  15. Thank you for post this article. Elastic tape used to treat muscle and joint related injuries.

    ReplyDelete
  16. Thank you for sharing this blog. skin graft products

    ReplyDelete
  17. Very nice article, thank you for sharing with us
    Reach at Jainsons Lights to urge the Pulse Oximeter Wholesale in Delhi at an inexpensive price. Pulse oximeters are used to measure the amount of oxygenated blood (SpO2) in your body, giving medical professionals an inspiration of what quantity of oxygen your body is carrying to vital organs and tissues.

    ReplyDelete
  18. Thank you for sharing this article.
    bed sores
    diabetic foot ulcer

    ReplyDelete
  19. Thankyou for sharing this article. From this blog i know about how to care about wounds by using diabetic foot ulcer.

    ReplyDelete
  20. Thank you for sharing this article. From this it is possible to learn about the effects of injury and the methods of repairing them.
    woundcare supplies near me
    purchase medicine online

    ReplyDelete
  21. Very interesting article, I really appreciate it.
    Are you in search of a UVC Disinfection Box Supplier? If yes, then you'll visit Jainsons Lights. It’s a number one firm that has various other forms of medical-related equipment like IR Thermometer, 3 Ply mask, UVC Disinfection Tube, UVC Disinfection Bag, Pulse Oximeter, Oxygen Concentrator, and lots of more.

    ReplyDelete
  22. Nice article, I really appreciate it.
    If you're bored with looking for the simplest IR Thermometer Suppliers in Delhi, then we propose you visit Jainsons Lights directly and find your desire medical-related products at a reasonable price. Here, all the professional experts always able to help their clients so they feel comfortable and relaxed.

    ReplyDelete
  23. BiPAP may be a sort of mask that's applied to the face. One thing to stay in mind while installing it's that the BiPAP machine should be installed properly so that the air cannot escape. Also, the lungs can work properly. If you want to know about Bipap Machine Cost in Delhi, then visit the website of Jainsons Lights.

    ReplyDelete
  24. This article is very nice, thank you for sharing with us.
    If you're seeking a reliable UVC Disinfection Box Supplier, then you'll choose Jainsons Lights. At this company, you'll buy rather more medical-related equipment like a Digital Thermometer, Oxygen Concentrator, UVC Disinfection Tube, Ventilator Machine, and lots more. Over to the web site you get the entire detail of all our products.

    ReplyDelete
  25. This article is very nice, thank you for sharing with us.
    If you people think to buy for Oxygen Machine for Home Use, then within the market various supplier available that helps you in providing this machine at the affordable price. The demand for Oxygen machines has also increased rapidly, which could be used especially in patients facing home isolation and in hospitals facing oxygen shortage.

    ReplyDelete
  26. Injury care is unique in the medical field. If the wounds in our body are not properly maintained the consequences will be much worse. Take care about your wounds by nanocrystalline silver gel uses.
    synersol ag nanocrystalline silver gel

    ReplyDelete
  27. Injury care is unique in the medical field. If the wounds in our body are not properly maintained the consequences will be much worse. Take care about your wounds by
    seeskin plus collagen sponge dressing
    synerheal collagen sponge dressing

    ReplyDelete
  28. The way a scar looks is often determined by how effectively the wound heals. While scars from surgery or over joints such as the knees and elbows are difficult to avoid, scars from minor cuts and scrapes can be reduced in appearance by treating the wound appropriately at home.
    stomahesive paste by convatec
    vac granufoam by kci

    ReplyDelete
  29. DR EMU WHO HELP PEOPLE IN ANY TYPE OF LOTTERY NUMBERS
    It is a very hard situation when playing the lottery and never won, or keep winning low fund not up to 100 bucks, i have been a victim of such a tough life, the biggest fund i have ever won was 100 bucks, and i have been playing lottery for almost 12 years now, things suddenly change the moment i came across a secret online, a testimony of a spell caster called dr emu, who help people in any type of lottery numbers, i was not easily convinced, but i decided to give try, now i am a proud lottery winner with the help of dr emu, i won $1,000.0000.00 and i am making this known to every one out there who have been trying all day to win the lottery, believe me this is the only way to win the lottery.

    Dr Emu can also help you fix this issues

    (1)Ex back.
    (2)Herbal cure & Spiritual healing.
    (3)You want to be promoted in your office.
    (4)Pregnancy spell.
    (5)Win a court case.

    Contact him on email Emutemple@gmail.com
    What’s app +2347012841542
    Website Https://emutemple.wordpress.com/
    Facebook page Https://web.facebook.com/Emu-Temple-104891335203341

    ReplyDelete
  30. When your skin is harmed, whether by accident or as a result of surgery, your body attempts to heal the wound. A scar may appear while your skin heals, as this is a normal part of the healing process.
    duoderm extra thin by convatec
    stomahesive paste by convatec

    ReplyDelete
  31. Jainsons Lights is the leading firm is famous for providing the Oxygen Concentrator Delhi to their clients at an affordable price. Oxygen concentrators are devices that concentrate oxygen. To produce the economical source of oxygen in industrial processes also, these devices are being employed. Thanks to the portability they're gaining importance within the medical field.

    ReplyDelete
  32. The most important medical device at the time of covid-19 spread is infrared thermometer, which you will be able to use to take your body temperature from safe distancing. If you're trying to find the IR Thermometer Suppliers in Delhi, then you'll be able to reach the platform of Jainsons Lights.

    ReplyDelete
  33. Would you wish to grasp about Ventilator Machine Price in Delhi? If yes, then check the official website of Jainsons Lights. The ventilator machines in most hospitals are getting used to treat people, but doctors also say that they still need more machines. The ventilator machine is usually used for brief periods, like during surgery when you're under anesthesia.

    ReplyDelete
  34. Finding reliable Bipap Machine Dealers in Delhi isn't so simple, you wish to figure hard for locating the simplest one. Jainsons Light is a reputed firm that is famous for providing the simplest quality medical equipment to their clients at an affordable price. If you wish highest quality medical products, then visit the corporate website and acquire all the products delivered to your doorstep.

    ReplyDelete
  35. People who are suffering from a heavy illness and medical problem that make it difficult for them to breathe, and the medical doctor may suggest using an Oxygen Concentrator to require a deep breath. If you people wish to understand the precise Oxygen Concentrator Price in Delhi, then don’t get upset. Visit the official website of Jainsons Lights. It’s the foremost company that famous for providing the simplest quality medical-related equipment to their clients at a low cost.

    ReplyDelete
  36. After a long time, I read a very beautiful and very important article that I enjoyed reading. I have found that this article has many important points. Thanks .pls visit our website personal injury treatment Surprise az

    ReplyDelete
  37. Thank you for sharing this. Colldrez D treatment for burns is a sterile, translucent, flexible dressing, designed to heal partial thickness burns, bed sores and skin transplants.

    ReplyDelete
  38. This comment has been removed by the author.

    ReplyDelete

Related Posts Plugin for WordPress, Blogger...
Related Posts Plugin for WordPress, Blogger...